Sunteți pe pagina 1din 94

Anul XI, Nr.

2 Iulie – Decembrie 2009

RECREAŢII
MATEMATICE
REVISTĂ DE MATEMATICĂ PENTRU ELEVI ŞI PROFESORI

10 ani de la apariţia
revistei “Recreaţii Matematice”

e i   1

Asociaţia “Recreaţii Matematice”


IAŞI - 2009
Semnificaţia formulei de pe copertă:
i
Într-o formă concisă, formula e  1 leagă cele patru ramuri fundamentale
ale matematicii:

ARITMETICA reprezentată de 1
GEOMETRIA reprezentată de 
ALGEBRA reprezentată de i
ANALIZA MATEMATICĂ reprezentată de e

Redacţia revistei :
Petru ASAFTEI, Dumitru BĂTINEŢU-GIURGIU (Bucureşti), Temistocle BÎRSAN, Dan
BRÂNZEI, Alexandru CĂRĂUŞU, Constantin CHIRILĂ, Eugenia COHAL, Adrian
CORDUNEANU, Mihai CRĂCIUN (Paşcani), Paraschiva GALIA, Paul GEORGESCU,
Mihai HAIVAS, Gheorghe IUREA, Lucian-Georges Lăduncă, Mircea LUPAN, Gabriel
MÎRŞANU, Alexandru NEGRESCU (student, Iaşi), Gabriel POPA, Dan POPESCU
(Suceava), Florin POPOVICI (Braşov), Maria RACU, Neculai ROMAN (Mirceşti), Ioan
SĂCĂLEANU (Hârlău), Ioan ŞERDEAN (Orăştie), Dan TIBA (Bucureşti), Marian TETIVA
(Bârlad), Lucian TUŢESCU (Craiova), Adrian ZANOSCHI, Titu ZVONARU (Comăneşti).

Materialele vor fi trimise la una dintre adresele e-mail : t-birsan@yahoo.com


profgpopa@yahoo.co.uk
vpgeo@go.com

COPYRIGHT © 2008, ASOCIAŢIA “RECREAŢII MATEMATICE”. Toate drepturile aparţin Asociaţiei


“Recreaţii Matematice”. Reproducerea integrală sau parţială a textului sau a ilustraţiilor din
această revistă este posibilă numai cu acordul prealabil scris al acesteia.

TIPĂRITĂ LA SL&F IMPEX IAŞI


Bd. Carol I, nr. 3-5
Tel. 0788 498933
E-mail: simonaslf@yahoo.com

ISSN 1582 - 1765


Către cititori
după zece ani de apariţie a revistei
Cascada cântă:
Îmi revărs cu bucurie apele,
dacă un strop din ele ajunge ı̂nsetatului
Rabindranath Tagore

În toamna anului 1999 apare la Iaşi primul număr al revistei Recreaţii Matematice.
Continuându-şi apariţia ı̂n mod constant cu două numere anual, revista a atins zece
ani de existenţă ı̂n primăvara acestui an. Revista este gândită de iniţiatorii ei să fie
o continuatoare a vechii reviste Recreaţii Ştiinţifice (1883–1888), de la care a preluat
şi numele, puţin modificat, noua revistă având un conţinut strict matematic.
Revista Recreaţii Ştiinţifice este prima revistă ştiinţifică din ţară care s-a adresat
tineretului cu chestiuni din toate ramurile ştiinţei, dar cu un conţinut predominant
matematic; ı̂n cuvântul de ı̂nceput al celui de-al şaselea an de apariţie se spune, cu
modestie, dar şi cu mândrie: Credem că noi am tras cea ı̂ntăi brazdă [...] Brazda′ i
mică şi ı̂ngustă, dar există!
Este o mare ı̂ndrăzneală să-ţi propui a fi continuator al unei reviste care a făcut
operă de pionerat ı̂n cultura românească şi a avut o contribuţie importantă ı̂n educaţia
matematică a tineretului. Preluând scopurile generoase ale creatorilor Recreaţiilor
Ştiinţifice, actuale şi astăzi, dorinţa noastră supremă este de a asigura noii reviste un
nivel de calitate şi de utilitate la ı̂nălţimea celui avut de ”străbuna” sa.
Acum, la ı̂mplinirea a zece ani de apariţie, Recreaţiile Matematice este o revistă
considerată printre primele publicaţii de gen din ı̂ntreaga ţară. Fondatorii revis-
tei Recreaţii Matematice sunt (ı̂n ordine alfabetică): T. Bı̂rsan, C. Calistru, Al.
Cărăuşu, C. Cocea, A. Corduneanu, Gh. Iurea, care au format primul comitet
de redacţie. În timp, li s-au alăturat alţi profesori şi elevi. De Alin Spumă şi
Mihai Gârtan, dispăruţi din rândurile noastre, ne vom aminti cu pietate, dar şi cu
recunoştinţă pentru aportul lor entuziast la apariţia revistei ı̂n perioada de ı̂nceput
a ei. Cu nr. 2/2001 intră ı̂n redacţie Gabriel Popa, care devine ı̂n scurt timp unul
dintre principalii realizatori ai revistei.
Revista este distribuită şi are colaboratori, elevi şi profesori ı̂n toată ţara: Bârlad,
Braşov, Bucureşti, Craiova, Galaţi, Hârlău, Orăştie, Paşcani, Piteşti, Rm. Vâlcea,
Satu Mare, Sf. Gheorghe (Tulcea), Suceava, Timişoara, Vaslui etc. Menţionăm câţiva
colaboratori din ţară ataşaţi Recreaţiilor Matematice: D. Popescu, F. Popovici, I.
Şerdean, M. Tetiva, D. Tiba, L. Tuţescu, T. Zvonaru ş.a.
Structura revistei comportă trei părţi mari: I) evenimente din lumea matematicii
(conferinţe, aniversări, comemorări etc.), II) partea ”teoretică” cu rubricile: articole
şi note, nota elevului, chestiuni metodice, istoria matematicii, corespondenţe ş.a.,
III) partea aplicativă cu rubricile: concursuri şi examene, probleme şi soluţii, pa-
gina rezolvitorilor. Mai sunt presărate ı̂n cuprinsul numerelor de revistă amuzamente
matematice, diverse informaţii, premii şi elevi premiaţi.
Se găsesc prezentate ı̂n paginile revistei figuri de mari matematicieni: P. Fermat,
N.H. Abel, H. Poincaré, L. Euler, A.N. Kolmogorov, W.R. Hamilton ş.a., ilustre nume
ale matematicii româneşti: Spiru Haret, Gh. Vrănceanu, Al. Myller, Gr. C. Moisil,

85
M. Haimovici ş.a., probleme celebre ale matematicii din toate timpurile: postulatul V
al lui Euclid, construcţii cu rigla şi compasul, problema celor patru culori, conjectura
lui Poincaré ş.a., instituţii şi reviste care au contribuit la progresul ı̂nvăţământului
şi cercetării matematice româneşti: Seminarul Matematic ”Al. Myller” din Iaşi, Ob-
servatorul astronomic din Iaşi, revista Recreaţii Ştiinţifice, Revista Ştiinţifică ”V.
Adamachi” ş.a., evenimente importante: Congresele internaţionale ale matematicie-
nilor români din 2003 şi 2007, cât şi reflectarea altor aspecte ale vieţii matematice.
Revista Recreaţii Ştiinţifice, pe care ı̂ncercăm să o continuăm, a fost sărbătorită
ı̂n 2003, la 120 de ani de la apariţia sa, prin publicarea unor materiale omagiale, dar
şi a unor articole selectate din numerele acestei vechi reviste. A fost sărbătorită mai
amplu ı̂n 2008, la 125 de ani de la apariţie, ı̂n cadrul unui simpozion desfăşurat ı̂n
aula filialei Iaşi a Academiei Române. O importantă acţiune a fost, reeditarea celor
şase tomuri de Recreaţii Ştiinţifice prin redarea textului originar cu mijloace moderne
şi prin producerea unei variante electronice a lui de către firma Kepler Systèmes
d′ Information (director Marinela Ghigea).
În rubricile părţii ”teoretice” a revistei au fost publicate ı̂n aceşti 10 ani, un număr
de 216 articole: 121 note matematice, 26 note ale elevilor, 34 chestiuni metodice, 10
”corespondenţe” (articole primite din afara ţării), 9 articole adresate claselor primare
şi 16 articole repartizate ı̂n alte rubrici.
Datorită strădaniilor unor membri ai redacţiei revistei, au fost atraşi, ca rezolvitori
sau colaboratori cu note matematice, elevi talentaţi, olimpici, care, fără ı̂ndoială, se
vor afirma ı̂n scurt timp ca valoroşi cercetători ı̂n domeniul matematicii: Gabriel
Dospinescu, Adrian Zahariuc, Marius Pachiţariu, Irina Mustaţă, Vlad Emanuel,
Cezar Lupu, Iurie Boreico (Chişinău), Marius Tiba, recompensaţi cu premii ı̂n bani
şi diplome.
La rubrica Corespondenţe articolele au apărut ı̂n limba ı̂n care au fost redactate
(română, franceză şi engleză) şi provin de la corespondenţi din ţările: Republica
Moldova, Germania, Kazahstan, Franţa şi China.
Date statistice sumare dau o imagine asupra părţii de probleme şi soluţii (toate
problemele au primit soluţii după un an de la publicare): ciclul primar–173 probleme,
gimnaziu–432, liceu–400, de tip G (avansate pentru gimnaziu)–165, de tip L (avansate
pentru liceu)–165. La aceastea trebuie adăugate şi problemele date la diferite concur-
suri naţionale care sunt prezentate ı̂n revistă ediţie de ediţie: Concursul ”Al. Myller”,
Concursul ”Florica T. Câmpan”, Concursul ”Recreaţii Matematice” ş.a. Pentru trei
apariţii la rubrica Pagina rezolvitorilor, elevii harnici sunt premiaţi cu câte o diplomă
şi cărţi; ı̂n acest interval de 10 ani au fost premiaţi 234 de elevi rezolvitori, cei mai
mulţi fiind din ciclul primar.
Eforturile colectivului de redacţie au fost uşurate datorită colaborării unui număr
mare de profesori din ı̂ntreaga ţară, care prin materialele oferite au contribuit la
ridicarea nivelului revistei.
Revista Recreaţii Matematice este recenzată ı̂n G.M.-A şi ı̂n revista de referate
Zentralblatt für Didaktik der Mathematik (ZDM). Pagina web a revistei poate fi
vizitată la adresa: http://www.recreatiimatematice.ro

Redacţia revistei

86
NECULAI GHEORGHIU

(1930-2009)

În luna ianuarie 2009, Profesorul nostru,


Neculai Gheorghiu a plecat dintre noi pentru
totdeauna. Va sta de acum ı̂ncolo la masa um-
brelor.
Scriu aceste rânduri, nu numai ı̂n calitate de
decan al Facultăţii de matematică din Iaşi, fa-
cultate pe care Profesorul Gheorghiu a slujit-o cu
dăruire şi pasiune o viaţă, ci mai ales ı̂n calitate
de discipol al său.
Am urmat două cursuri ţinute de domnia sa,
cel de analiză matematică din semestrul I al anului
I şi cel de analiză funcţională din anul al III-lea.
În plus, Profesorul Gheorghiu a fost ı̂ndrumătorul
ştiinţific al lucrării mele de licenţă. Ca asistent
universitar, am făcut mulţi ani la rând seminarul
la cursul de analiză funcţională din anul al III-lea.
Perioada studenţiei mele a reprezentat o etapă
de vârf a carierei Profesorului. Astfel, ı̂n anul 1973 a apărut, ı̂n colaborare cu prof.
Teodor Precupanu, cursul litografiat Analiză matematică, publicat ı̂n 1976 la Editura
didactică şi pedagogică, iar ı̂n 1974 a apărut, la Editura Academiei Române, cartea
Introducere ı̂n analiză funcţională. În aceeaşi perioadă, 1972-1976, a fost prodecan al
Facultăţii de matematică a Universităţii ”Al.I. Cuza” Iaşi, punând ı̂n slujba acesteia
calităţile sale de bun organizator şi om cu gândire limpede şi de perspectivă. Anterior,
fusese decanul Facultăţii de matematică a Institutului Pedagogic de 3 ani din Iaşi,
unde a fost colaborator apropiat al prof. Ilie Popa, rectorul institutului, şi a militat
neobosit pentru o calitate superioară a ı̂nvăţământului. A făcut parte din comitetul
de conducere al Seminarului matematic ”Al. Myller” din Iaşi.
Profesorul Neculai Gheorghiu a fost ı̂nzestrat cu un talent didactic de excepţie.
Lecţiile sale erau de o claritate, de o rigoare şi de o naturaleţe deosebite. Nu am
predat decât lucruri simple, obişnuia să spună Profesorul; ı̂n realitate, concepte cu
un ı̂nalt grad de abstractizare şi teorii complexe deveneau simple şi transparente
datorită harului său pedagogic. Ele au contribuit decisiv la orientarea mea spre
domeniul analizei matematice. Nu puţini au fost şi mai sunt aceia care, la rândul
lor, au afirmat şi afirmă cu recunoştinţă că, precumpănitor, şi-au modelat gândirea
matematică tocmai datorită acestor lecţii.
Avea şi un mod original de a se comporta cu studenţii şi cu cei din jur. Era
deschis, direct, sever şi uneori dur, dar aceste asperităţi ale firii sale erau completate
şi compensate de bogăţia sufletului său şi omenia sa. Faţă de studenţi dovedea largi
disponibilităţi de a-i asculta şi ı̂nţelege, dar şi de a-i ajuta, ı̂n măsura ı̂n care o

87
putea face. Era ı̂ntotdeauna corect şi cinstit. Toate acestea au făcut ca Profesorul
Neculai Gheorghiu să fie foarte mult apreciat şi iubit de multe generaţii de studenţi.
Aproape de fiecare dată când eram undeva unde se găseau foşti studenţi ai săi, apărea
ı̂ntrebarea Ce mai face Profesorul Gheorghiu? sau, mai direct şi mai sugestiv, Ce mai
face Nae?, ca şi cum Nae era un fost coleg sau un prieten comun.
Avea nostalgia locului natal; vorbea cu afecţiune de Mileanca (fostul judeţ Doro-
hoi), satul ı̂n care s-a născut şi a urmat clasele primare şi nu uita să adauge cu mândrie
că la Mileanca se află cel mai bun cernoziom. Îşi amintea cu plăcere de şcolile pe care
le-a urmat şi de faptul că a fost propunător de probleme la Gazeta Matematică (toate
de geometrie, ţinea să precizeze).
O dovadă a posibilităţilor ı̂ntinse de comunicare a Profesorului Neculai
Gheorghiu cu semenii săi din domenii diverse de activitate este implicarea sa ı̂n
viaţa sportivă a Iaşului, la un moment dat având funcţii de conducere ı̂n baschetul şi
fotbalul ieşean.
S-a preocupat şi de ı̂nvăţământul matematic preuniversitar. În repetate rânduri a
onorat ca preşedinte comisia judeţeană a Olimpiadei naţionale de matematică, a ţinut
prelegeri pentru elevi, a propus probleme la concursuri şi olimpiade şi este coautor
la o frumoasă culegere de probleme pentru elevi intitulată Matematici elementare;
probleme de sinteză (Editura Junimea, Iaşi, 1983). Evident, ca orice culegere de
probleme bine scrisă şi aceasta conţine rezolvări integrale şi multe comentarii utile.
Există acolo o superbă problemă propusă de Profesorul Gheorghiu la etapa finală a
Olimpiadei de matematică din 1963.
Profesorul Neculai Gheorghiu avea o inteligenţă sclipitoare. A avut rezultate
notabile şi ı̂n activitatea de cercetare ştiinţifică, domeniul său predilect de preocupări
fiind acela al ecuaţiilor diferenţiale ordinare. Teza sa de doctorat, sub conducerea prof.
dr. doc. Ilie Popa, cu titlul Despre comportarea asimptotică a soluţiilor ecuaţiilor
diferenţiale de ordinul doi a fost foarte bine apreciată. Rezultatele cu totul deosebite
din activitatea didactică ı̂mpreună cu rezultatele din activitatea de cercetare au făcut
ca Profesorul Gheorghiu să avanseze rapid ı̂n cariera universitară, astfel că la 40 de
ani era deja profesor universitar.
Profesorul Neculai Gheorghiu continuă să fie prezent printre noi şi după dis-
pariţia sa fizică; este de găsit ı̂n rafturile de cărţi ale Seminarului Matematic ”Al.
Myller”, dar şi păstrat cu recunoştinţă şi iubire ı̂n inimile numeroaselor promoţii de
studenţi care l-au avut ca dascăl.
Va rămâne mereu ı̂n amintirea noastră ca un profesor reprezentativ al Facultăţii
de matematică a universităţii ieşene, cu o contribuţie importantă ı̂n progresul ı̂nvă-
ţământului matematic românesc!

Prof. dr. Ovidiu CÂRJĂ


Decan al Facultăţii de Matematică
Universitatea ”Al.I. Cuza” Iaşi

88
Conjectura Beal pentru polinoame
Temistocle BÎRSAN 1 , Gabriel DOSPINESCU 2

Abstract. This paper deals with the solution of equation (1) in the set of polynomials with
integer coefficients and it has an informative aim. As well as in the case of Fermat′ s Conjecture, the
solution to Beal′ s Conjecture (i.e., the claim that Eq. (1) with p, q, r- integer numbers greater than
2 has no solution with x, y, z positive and mutually prime) in the set C [X]is elementary (Proposition
1). The triples (p, q, r) with p, q, r in N∗ such that equation (1) has solutions in C [X] are: (1, q, r),
(2, 2, r), (2, 3, 3), (2, 3, 4), (2, 3, 5).
Keywords: polynomials, Beal′ s Conjecture, Fermat′ s Conjecture, regular polyhedra.
MSC 2000: 11D41.

În 1966, Andrew Beal instituie un premiu pentru demonstrarea sau infirmarea
aşa-numitei acum Conjecturi Beal [1]:
Ecuaţia

(1) xp + y q = z r ,

unde p, q, r sunt numere ı̂ntregi mai mari ca 2, nu are nici o soluţie cu x, y, z ı̂ntregi
pozitivi şi relativ primi.
Pentru p = q = r (= n) ecuaţia devine

(2) xn + y n = z n ,

ecuaţie care a fost subiect de preocupări pentru lumea matematică ı̂ncă din anti-
chitate şi mai este şi acum, deşi a fost realizată rezolvarea ei completă. Şcoala lui
Pitagora a demonstrat că ecuaţia x2 + y 2 = z 2 admite o infinitate de triplete (x, y, z)
formate cu numere ı̂ntregi, pozitive şi prime ı̂ntre ele ce o verifică. Pierre Fermat a
afirmat că ecuaţia (2) pentru n ≥ 3 nu admite nici o soluţie cu numere x, y, z ı̂ntregi
şi nenule şi a notat pe marginea unei pagini a Aritmeticii lui Diofant că posedă o
demonstraţie minunată a acestui fapt. Generaţii de matematicieni, aflaţi pe urmele
acestei demonstraţii, n-au reuşit să o găsească sau să lămurească care ar fi putut
să fie aceasta, ceea ce a făcut pe unii să presupună că Fermat a greşit pe parcursul
demonstraţiei sale. În sfârşit, ı̂n 1995, Andrew Wiles demonstrează Marea teo-
remă a lui Fermat, aşa cum istoria matematicii reţine afirmaţia lui Pierre Fermat [6];
demonstraţia lui Wiles este, ı̂nsă, accesibilă unui cerc foarte restrâns de specialişti.
În acest context, apare cu atât mai uimitor şi remarcabil faptul că rezolvarea
ecuaţiei (2) ı̂n mulţimea C [X] a polinoamelor cu coeficienţi complecşi este elementară,
1 Prof. dr., Catedra de matematică, Univ. Tehnică ”Gh. Asachi”, Iaşi
2 Student, École Normale Supérieure, Paris

89
accesibilă unui elev de liceu. Mai precis, cu ajutorul teoremei Mason - Stothers,
enunţată mai jos, se poate dovedi afirmaţia următoare:
Dacă n ≥ 3, atunci ecuaţia (2) nu are soluţii ı̂n C [X] cu polinoame neconstante
şi relativ prime.
O prezentare a acestor fapte este făcută ı̂n [4]; cititorii revistei sunt informaţi
asupra acestui subiect ı̂n [2].
În această ordine de idei, se impune de la sine ı̂nlocuirea ecuaţiei (2) cu (1) şi
rezolvarea acesteia din urmă ı̂n mulţimea C [X]. Este tocmai ceea ce ne propunem.
Lăsăm, ı̂nsă, cititorilor plăcerea de a rezolva Conjectura lui Beal propriu-zisă (ı̂n Z)
(informaţii asupra premiului oferit de A. Beal sunt date ı̂n [1]).
Vom apela şi ı̂n acest caz la
Teorema Mason - Stothers ([4], [2]). Fie f, g, h ∈ C [X] neconstante şi relativ
prime. Dacă are loc egalitatea f + g = h, atunci

(3) max {deg f, deg g, deg h} ≤ n0 (f gh) − 1

Q
k
mi
(pentru f ∈ C [X] cu descompunerea f (X) = α (X − ai ) , se notează deg f =
i=1
m1 + m2 + · · · + mk – gradul lui f şi n0 (f ) = k – numărul rădăcinilor sale distincte).
Demonstraţia este elementară şi poate fi găsită ı̂n [2].
Propoziţia 1 (Conjectura Beal pentru polinoame). Dacă p, q, r ∈ Z şi
p ≥ 3, q ≥ 3, r ≥ 3, atunci ecuaţia (1) nu are soluţii ı̂n C [X] cu polinoame necon-
stante şi relativ prime.
Demonstraţie. Procedăm ca şi ı̂n cazul ecuaţiei (2). Presupunem că ecuaţia
(1) cu exponenţii p, q, r satisfăcând condiţiile din enunţ admite o soluţie (f, g, h) cu
polinoame neconstante relativ prime. Ca urmare, putem aplica Teorema Mason –
p q r
Stothers polinoamelor (f (X)) , (g (X)) , (h (X)) şi scrie
p p q r
deg (f (X)) ≤ n0 (f (X)) (g (X)) (h (X)) − 1,

de unde
p deg f (X) ≤ n0 (f (X) g (X) h (X)) − 1
şi, deci,
p deg f (X) ≤ n0 (f (X)) + n0 (g (X)) + n0 (h (X)) − 1.
Scriind şi inegalităţile analoage acesteia relativ la polinoamele g şi h şi adunându-le
membru cu membru, obţinem relaţia

(p − 3) deg f (X) + (q − 3) deg g (X) + (r − 3) deg h (X) ≤ −3,

care este falsă, căci p ≥ 3, q ≥ 3, r ≥ 3, şi demonstraţia este ı̂ncheiată.


Înainte de a vedea ce se ı̂ntâmplă atunci când unul (cel puţin) dintre exponenţii
ecuaţiei (1) este mai mic ca 3, vom face câteva

90
Observaţii. 1) În prezenţa egalităţii f + g = h, faptul că polinoamele f, g, h sunt
prime ı̂ntre ele este echivalent cu condiţia ca f, g, h să fie relativ prime două câte
două.
2) Dacă εp1 = 1, εq2 = 1 şi εr3 = 1 şi (x, y, z) este o soluţie ı̂n C [X] a ecuaţiei
(1), atunci sunt soluţii ale acestei ecuaţii şi tripletele: (ε1 x, ε2 y, ε3 z), (x, ε2 y, ε3 z),
(x, y, ε3 z) etc. Vom face abstracţie de aceste soluţii derivate ale unei soluţii găsite.
3) Fără a restrânge generalitatea, putem considera că ı̂n tripleta (p, q, r) a exponen-
ţilor ecuaţiei (1) avem

(4) p ≤ q ≤ r.

Într-adevăr, putem lua p ≤ q schimbând, eventual, x şi y ı̂ntre ele ı̂n ecuaţia (1).
p
Dacă r ≤ p, scriem (1) sub forma z r + (ε1 x) = y q , unde εp1 = −1. Dacă p ≤ r ≤ q,
r q
punem (1) sub forma xp + (ε2 z) = (ε3 y) , unde εr2 = εq3 = −1.
Conform Propoziţiei 1 şi ţinând seama de (4), rezultă că p ∈ {1, 2}.
I Cazul (1,q,r ) este banal: soluţia generală a ecuaţiei

(5) x + yq = zr

este dată de

(6) x = hr − g q , y = g, z = h, ∀g, h ∈ C [X] ,

pentru orice exponenţi q, r ∈ N∗ .


II Cazul (2,q,r ). Cu teorema Mason - Stothers vom obţine limitări importante
ı̂n privinţa exponenţilor q şi r. Fie (x, y, z) o soluţie ı̂n C [X] a ecuaţiei x2 + y q = z r
(cu x, y, z neconstante şi relativ prime) şi fie a = deg x, b = deg y, c = deg z. Teorema
amintită ne spune că
€ Š
max {2a, qb, rc} ≤ n0 x2 y q z r − 1,

de unde
max {2a, qb, rc} ≤ a + b + c − 1,
deci avem

(7) a<b+c−1 şi max {qb, rc} ≤ 2 (b + c − 1) .


1 q
Cum max {qb, rc} ≥ (qb + rc) ≥ (b + c), combinând cu a doua relaţie din (7) vom
2 2
q
obţine (b + c) ≤ 2 (b + c − 1), ceea ce conduce la q ∈ {2, 3}.
2
II.1 Subcazul (2,2,r ), cu r ≥ 2. Ecuaţia

(8) x2 + y 2 = z r

se mai scrie
(x + iy) (x − iy) = z r

91
şi cum x + iy şi x − iy sunt polinoame prime ı̂ntre ele rezultă că fiecare este puterea de
exponent r a unui polinom: x + iy = f r şi x − iy = g r . În final, ecuaţia x2 + y 2 = z r
are soluţia dată de
1 r 1
(9) x= (f + g r ) , y= (f r − g r ) , z = f g, ∀f, g ∈ C [X] .
2 2i
Verificarea faptului că (9) este o soluţie a ecuaţiei (8) este imediată.
II.2 Subcazul (2,3,r ), cu r ≥ 3. Din (7), avem max {3b, rc} ≤ 2 (b + c − 1), din
care rezultă că
b < 2 (c − 1) şi rc < 6 (c − 1) ,
deci r ∈ {3, 4, 5}. Aşadar, au rămas trei situaţii de analizat: (2, 3, 3), (2, 3, 4) şi
(2, 3, 5).
II.2.1 Ecuaţia x2 + y 3 = z 3 . Vom vedea că rezolvarea acestei ecuaţii se reduce
la un caz anterior studiat. Într-adevăr, scriem ecuaţia ı̂n forma
€ Š
(10) (z − y) (z − εy) z − ε2 y = x2 ,

unde ε2 + ε + 1 = 0. Întrucât z − y, z − εy şi z − ε2 y sunt relativ prime două câte


două, din egalitatea precedentă deducem că fiecare dintre acestea este pătratul unui
polinom:
z − y = f 2 , z − εy = g 2 , z − ε2 y = h2 .
Pentru ca acest sistem liniar ı̂n z şi y să fie compatibil, impunem polinoamelor f , g,
h condiţia
−εf 2 + (1 + ε) g 2 = h2 .
Aceasta, ı̂nsă, se reduce după substituţii evidente la ecuaţia x2 + y 2 = z 2 , care se
rezolvă conform cazului II.1.
II.2.2 Ecuaţia x2 + y 3 = z 4 . Procedăm ca ı̂n cazul precedent. Scriem ecuaţia ı̂n
discuţie sub forma
€ Š€ Š
(11) z2 − x z2 + x = y3 .

Cum z 2 − x şi z 2 + x sunt relativ prime, urmează că ele sunt cuburi de polinoame:

z2 − x = f 3, z2 + x = g3 .

Astfel, suntem conduşi la ecuaţia 2z 2 = f 3 + g 3 care se reduce la rândul ei la ecuaţia


x2 + y 3 = z 3 ı̂ntâlnită ı̂n subcazul II.2.1. Practic, soluţiile ecuaţiei x2 + y 3 = z 4 vor
fi obţinute parametric şi se vor exprima ı̂n funcţie de soluţiile cazului (2, 2, r).
II.2.3 Ecuaţia x2 + y 3 = z 5 . Rezolvarea acestei ecuaţii pare să fie deosebit de
dificilă. În lipsa unei descompuneri de tipul (10) sau (11), nu putem proceda ca mai
sus.

92
Stabilim doar faptul că ecuaţia are soluţii. Se poate verifica direct că tripleta
(x, y, z) dată de

x = X 30 + 522X 25 − 10005X 20 − 10005X 10 − 522X 5 + 1,


(12) y = −X 20 + 228X 15 − 494X 10 − 228X 5 − 1,
√ € Š
z = 1728 X 11 + 11X 6 − X
5

este o soluţie a acestei ecuaţii ([3] , [5]).


Observaţie. În [3], Felix Klein pune ı̂n evidenţă legătura strânsă care există
ı̂ntre poliedrele regulate şi soluţiile ı̂n C(X) ale ecuaţiei (1). Astfel, cazul (2, 3, 3)
este legat de tetraedrul regulat, cazul (2, 3, 4) de cub şi octogonul regulat, iar cazul
(2, 3, 5), cu soluţia (12), de dodecaedrul şi de icosaedrul regulat.
Această legătură este menţionată şi discutată ı̂n [5].
În concluzie, tripletele de exponenţi (p, q, r), p, q, r ∈ N∗ , pentru care ecuaţia (1)
are soluţii ı̂n C [X] sunt: (1, q, r), (2, 2, r), (2, 3, 3), (2, 3, 4), (2, 3, 5) şi toate per-
mutările acestora.

Bibliografie
1. *** - Beals Conjecture, The New Zeland Math. Mag., 35(1998), no.2, 38.
2. T. Bı̂rsan - Marea teoremă a lui Fermat pentru polinoame, RecMat - 1/2004, 5-9.
3. F. Klein - Vorlesungen über das Ikosaeder und die Auflösung der Gleichungen vom
fünften Graden, Teubner, Leipsig, 1884.
4. S. Lang - Math Talks for Undergraduates, Springer, 1999.
5. V. V. Prasolov - Essays on Numbers and Figures, Amer. Math. Soc., Mathematical
World, v. 16, 2000.
6. A. Wiles - Modular elliptic curves and Fermat ′ s Last Theorem, Annals of Math.,
142(1995), 443-551.

În 1962 Bachet de Méziriac publică o versiune latină a Aritmeticii ce includea


peste o sută de probleme şi avea margini largi ale textului. Pe paginile unui astfel
de exemplar, Pierre Fermat ı̂şi nota soluţiile, comentariile şi rezultatele proprii. Pe
marginea Cărţii a II-a Fermat notă afirmaţia :

ecuaţia xn + y n = z n nu are soluţii ı̂n numere ı̂ntregi şi nenule pentru n ≥ 3.

cunoscută acum ca Marea Teoremă a lui Fermat. Apoi a scris comentariul:

(continuare la pagina 20)

93
Teorema lui Brouwer - un caz particular elementar
Cornelia-Livia BEJAN 1

Abstract. By following the ideas developed by T. Traynor in [2], an elementary proof of


Brouwer′ s fixed point theorem is presented for the restricted case of continuously differentiable
functions.
Keywords: closed ball, ball, fixed point, continuously differentiable function.
MSC 2000: 54H25.

Teorema de punct fix a lui Brouwer are o istorie lungă. Ideile ce conduc la


demonstraţia acesteia se găsesc la Henri Poincaré ı̂nainte de 1886. L.E.J. Brouwer
a demonstrat teorema pentru n = 3 ı̂n 1909. În 1910 J. Hadamard dă prima
demonstraţie pentru n arbitrar, iar Brouwer dă o alta ı̂n 1912.
În [2], autorul a dat o prezentare elementară a teoremei de punct fix a lui Brouwer.
În această notă, adaptând demonstraţia elementară dată ı̂n [2], ne propunem să
prezentăm un caz particular al teoremei lui Brouwer, ı̂ntr-o formă şi mai accesibilă
profesorilor şi studenţilor.
Peste tot vom nota cu D discul unitate din plan şi cu C frontiera sa, adică cercul.
Teorema de punct fix a lui Brouwer afirmă că orice aplicaţie continuă f : D → D
admite măcar un punct fix, adică există măcar un element x ∈ D astfel ı̂ncât f (x) = x.
Altfel spus, dacă deformăm ı̂n mod continuu discul unitate ı̂n el ı̂nsuşi, atunci există
măcar un punct care nu-şi schimbă poziţia.
Vom prezenta un caz particular al acestei teoreme (ipoteze ı̂ntărite) şi anume:
Teoremă. Dacă f : D → D este o funcţie cu derivatele de ordinul ı̂ntâi continue,
atunci ea admite măcar un punct fix.
Lemă. Nu există nici o aplicaţie f : D → C cu derivatele de ordinul ı̂ntâi continue
care să lase fixe toate punctele cercului, adică f (x) = x, ∀x ∈ C.
Demonstraţie. Presupunem că ar exista o astfel de aplicaţie f şi notăm g(x) =
f (x) − x, ∀x ∈ D. Constatăm că g este o funcţie cu derivatele de ordin ı̂ntâi continue
şi, ı̂n consecinţă, majorate (ı̂n normă) de o constantă k. Din teorema valorii medii
avem ∥g(x) − g(y)∥ ≤ k∥x − y∥, ∀x, y ∈ D, adică distanţa ı̂ntre g(x) şi g(y) este
majorată de distanţa dintre punctele x şi y, multiplicată cu k.
Definim aplicaţia ft (x) = x + t · g(x) = (1 − t)x + tf (x), ∀t ∈ [0, 1]. Se verifică
faptul că ft lasă fixe punctele cercului, ∀t ∈ [0, 1]. Arătăm că ft este injectivă pentru
0 ≤ t < k1 . Într-adevăr, dacă ft (x) = ft (y), atunci ∥x − y∥ = t∥g(x) − g(y)∥ ≤
tk∥x − y∥, de unde obţinem x = y ı̂ntrucât tk < 1.
În plus, pentru fiecare t ∈ [0, 1] fixat, avem ft′ (x) = I +tg ′ (x), unde I este aplicaţia
identică a planului. Pentru 0 ≤ t < k1 se vede că aplicaţia ft duce discul D ı̂n el ı̂nsuşi,
adică ft (D) = D. În continuare să scriem:
Z
1
π = Aria(D) = Aria(ft (D)) = detft′ (x)dx, 0 ≤ t < .
D k
1 Prof.dr., Catedra de matematică, Univ. Tehnică ”Gh. Asachi”, Iaşi

94
Deci membrul drept al relaţiei precedente este constant ı̂n raport cu t. Mai mult,
cantitatea de sub integrală fiind un polinom ı̂n t, rezultă că prin integrare se obţine
tot un polinom ı̂n t. Din faptul că membrul drept este pe de o parte o constantă
şi pe de altă parte un polinom ı̂n t, urmează că valoarea sa este o constantă pentru
orice t ∈ [0, 1], nu numai pentru t ∈ [0, k1 ). Trecând la limită pentru t → 1 ı̂n relaţia
precedentă şi ţinând cont de f (D) = C, găsim:
π = Aria D = lim Aria(ft (D)) = Aria(f (D)) = Aria C = 0,
t→1
ceea ce este fals. Contradicţia ne arată că Lema este adevărată.
Demonstraţia Teoremei. Presupunem că aplicaţia din ipoteza teoremei nu are
puncte fixe. Atunci din inegalitatea Cauchy-Buniakowski-Schwarz se obţine
f (x) · x ≤ ∥x∥∥f (x)∥ ≤ 1, ∀x ∈ D.
Egalitatea s-ar atinge când x şi f (x) ar fi liniar dependente. Cum membrul drept ≤ 1,
dacă membrul stâng f (x)·x ar fi = 1, am avea egalitate, ı̂n care caz am avea f (x) = αx
cu α ∈ R şi ∥x∥ = 1. Deci α = αx · x = 1, de unde f (x) = αx = x; deducem că f ar
avea puncte fixe, ceea ce contrazice presupunerea făcută. În concluzie, egalitatea nu
se atinge şi deci avem x · f (x) < 1, ∀x ∈ D.
1−x·x
Construim o aplicaţie h(x) = x − f (x), ∀x ∈ D. Această aplicaţie este
1 − x · f (x)
bine definită ı̂ntrucât 1 − x · f (x) > 0. Evident, aplicaţia f are derivatele de ordinul
ı̂ntâi continue, iar h(x) = x, ∀x ∈ C. Pe de altă parte, avem:
Caz 1. Dacă (x · f (x))x = (x · x)f (x), atunci
x − (x · f (x))x − f (x) + (x · x)f (x) x − f (x)
h(x) = = ̸= 0,
1 − x · f (x) 1 − x · f (x)
deoarece am presupus că f nu are puncte fixe.
Caz 2. Dacă (x·f (x))x ̸= (x·x)f (x) rezultă că x şi f (x) nu sunt liniar dependente.
După modul cum este definită aplicaţia h, deducem că h(x) ̸= 0 căci ı̂n caz contrar x
şi f (x) ar fi liniar dependente.
În concluzie, constatăm că h nu se anulează şi deci putem defini o aplicaţie H(x) =
h(x)
, ∀x ∈ D. Se vede că H : D → C are derivatele de ordin ı̂ntâi continue şi
∥h(x)∥
fixează punctele cercului, adică H(x) = x, ∀x ∈ C. În baza Lemei, ı̂nsă, am ajuns la
o absurditate, ı̂ntrucât o astfel de aplicaţie H nu există. Demonstraţia este ı̂ncheiată.
Observaţie. O generalizare de la plan la spaţiul cu n dimensiuni se face urmând
pas cu pas calea parcursă mai sus. În liceu, elevii ı̂ntâlnesc teorema lui Brouwer ı̂n
cazul unidimensional: o funcţie f : [0, 1] → [0, 1] continuă are cel puţin un punct fix.

Bibliografie
1. L.E.J. Brouwer - Über Abbildungen von Mannigfaltigkeiten, Math. Ann., 71(1912),
97-115.
2. T. Traynor - An Easy Analitic Proof of Brouwer′ s Fixed Point Theorem, Atti Sem.
Mat. Fis. Univ. Modena, XLIV (1996), 479-483.

95
Exponentul de triangularitate al unui triunghi
Dorin MĂRGHIDANU1

Abstract. The following problem is investigated: if a, b, c denote the lengths of the sides of
a triangle, it is required to determine the values of the real and positive exponents α such that
the powers aα , bα , cα can still be the side lengths of a triangle. It is introduced the notion of
triangularity exponent t and it is proved that t = 2 for the right-angled triangles, t ∈(1, 2) for the
obtuse-angled triangles and t > 2 for the acute-angled triangles (Proposition 2).
Keywords: triangularity exponent.
2000 MSC: 51M15.

În această notă dăm un răspuns la următoarea ı̂ntrebare:


Dacă a, b, c sunt lungimile laturilor unui triunghi, pentru care numere α reale şi
pozitive puterile aα , bα , ca pot forma de asemenea un triunghi?
În prima parte a notei prezentăm instrumentul algebric de lucru – o extindere a
inegalităţii 11.19 din [1], p.99, iar ı̂n cea de-a doua introducem exponentul de trian-
gularitate al unui triunghi, noţiune necesară rezolvării problemei propuse.
1. La fel cum a fost demonstrată inegalitatea 11.19 din [1], putem stabili şi
Propozţia 1. Dacă pentru numerele reale strict pozitive a, a1 , a2 , . . . , an există
un număr real α > 0 astfel ı̂ncât aα = aα α α
1 + a2 + . . . + an , atunci au loc:
β β
(1) 1) a > a1 + a2 + . . . + an , ∀β ∈ R, β > α;
β β

(2) 2) aβ < aβ1 + aβ2 + . . . + aβn , ∀β ∈ R, 0 < β < α.


Demonstraţie. Din relaţia de condiţie rezultă că a > a1 , a > a2 , . . . , a > an .
1) β > α implică aβ−α > aβ−α 1 , . . . , aβ−α > aβ−α
n şi avem

aβ = aβ−α · aα = aβ−α (aα α


1 + . . . + an ) = a
β−α
· aα
1 + ... + a
β−α
· aan
> aβ−α
1 · aα β−α
1 + . . . + an · aα β β
n = a1 + . . . + an .

2) β < α implică aβ−α < aβ−α1 , . . . , aβ−α < aβ−α


n şi, inversând semnul de inegali-
tate ı̂n calculul de la punctul 1), obţinem (2).
Observaţii. 1. Relaţia (1) rămâne valabilă şi ı̂n condiţia aα > aα α α
1 + a2 + . . . + an .
2. Pentru n = 2, α = 2 şi β ∈ N, β > 2, punctul 1) revine la afirmaţia 11.19 [1].
Corolar. Dacă a, b, c sunt lungimile laturilor unui triunghi şi este verificată con-
diţia aα = bα +cα , atunci (aβ , bβ , cβ ) formează un triunghi dacă şi numai dacă β < α.
Vom indica două aplicaţii directe, ı̂n geometrie, ale rezultatelor precedente.
Aplicaţia 1. Într-un paralelipiped dreptunghic de muchii a, b, c şi diagonală mare
D, avem Dp > ap + bp + cp , ∀p ∈ R, p > 2.
Demonstraţie. Fie d diagonala feţei de laturi a, b. Conform punctului 1) al
Propoziţiei 1 (cu n = 2, α = 2), aplicat de două ı̂n cazul particular al triunghiului
dreptunghic avem Dp > dp + cp > ap + bp + cp .
1 Prof. dr., Liceul Teoretic din Corabia, d.marghidanu@gmail.com

96
Aplicaţia 2. Fie OABC un tetraedru tridreptunghic cu vârful O şi OA = a,
OB = b, OC = c. Atunci AB p + BC p + CAp > 2(ap + bp + cp ), ∀p ∈ R, p > 2.
Demonstraţie. Cu Propoziţia 1, aplicată la triunghiurile dreptunghice OAB,
OBC şi OCA, obţinem relaţiile AB p > ap + bp , BC p > bp + cp şi CAp > cp + ap care,
adunate, conduc la inegalitatea din enunţ.
2. Vom introduce o noţiune, cu rol decisiv, prin următoarea
Definiţie. Numărul real pozitiv t se numeşte exponent de triangularitate al tri-
unghului ABC cu laturi de lungimi a, b, c, dacă tripleta (as , bs , cs ) formează un tri-
unghi pentru orice s ∈ R∗+ , s < t, şi nu formează un triunghi pentru s ≥ t.
Rezolvarea problemei propuse este dată de
Propoziţia 2. Sunt adevărate următoarele afirmaţii:
1) t = 2, dacă triunghiul este dreptunghic;
2) t ∈ (1, 2), dacă este obtuzunghic;
3) t > 2, dacă este ascuţiunghic; ı̂n acest caz, t este număr real (finit), dacă
triunghiul nu-i isoscel sau dacă este isoscel şi unghiul opus bazei sale este strict mai
mare de 60◦ şi t = +∞, dacă triunghiul este isoscel şi are unghiul opus bazei mai mic
sau egal ca 60◦ .
Demonstraţie. Fie ABC cu a = max{a, b, c}. Considerăm funcţia continuă
f (x) = ax − bx − cx , x > 0.
1) Triunghiul fiind dreptunghic, avem a2 = b2 + c2 . Conform Corolarului de mai
sus, deducem că t = 2.
2) În acest caz, f (1) = a − b − c < 0 şi f (2) = a2 − b2 − c2 > 0 (triunghiul ABC
fiind obtuzunghic). Rezultă că există o valoare t∈(1, 2) (unică, conform Propoziţiei
1) astfel ı̂ncât f (t) = 0, adică at = bt + ct . Din nou apelând la Corolar, deducem că
t este exponentul de triangularitate al triunghiului.
3) Avem f (2) = a2 − b2 − c2 < 0 (triunghi ascuţiunghic) şi f (∞) = lim f (x) =
•  ‹x ˜
 c x
x→∞
b
lim a 1 −
x
− = +∞, dacă triunghiul nu-i isoscel sau dacă este isoscel
x→∞ a a
şi baza sa este mai mare ca laturile sale (echivalent, măsura unghiului din vârf este
strict cuprinsă ı̂ntre 60◦ şi 90◦ ). Deci, există t ∈ (2, ∞) unic astfel ı̂ncât at = bt + ct .
Conchidem că t astfel găsit este exponentul de triangularitate ı̂n subcazul considerat.
În sfârşit, dacă triunghiul este isoscel cu baza mai mică sau cel mult egală cu
laturile sale (echivalent, unghiul din vârf are măsura mai mică sau cel mult 60◦ ),
atunci se constată direct că puterile de exponent α ale laturilor acestuia formează un
triunghi de acelaşi tip, oricare ar fi α ∈ R. Aşadar, ı̂n acest subcaz avem t = +∞.
Consideraţiile precedente
√ √sugerează
√ examinarea triunghiurilor care verifică condiţii
ca a3 = b3 + c3 (sau a = b + c etc.), aşa cum s-a făcut ı̂n cazul triunghiurilor
dreptunghice (a2 = b2 + c2 ) sau altor triunghuiuri speciale (triunghiuri mediane,
triunghiuri cu laturi ı̂n progresie aritmetică etc.).

Bibliografie
1. O. Bottema, R.Z. Djordjević, R.R. Janić, D.S. Mitrinović, P.M. Vasić –
Geometric inequalities, Wolters-Noordhoff, Groningen, 1969.

97
Asupra unui şir de integrale Riemann
Dan POPESCU 1

Abstract. This Note is an extension of a former paper–[3] in the reference list. The author
remarks that many problems proposed as topics to various contests are direct consequences of Propo-
sitions 1 and 2 in the note.
Keywords: continuous function, periodic function, Rimann integral.
MSC 2000: 26A42.

În cele ce urmează, sunt prezentate două condiţii suficiente de convergenţă a şirului
real de integrale  
Z b
f (x) g (nx) dx ,
a n∈N

unde f şi g sunt funcţii integrabile Riemann care asigură corectitudinea definirii
şirului.
Un prim rezultat este prezentat ı̂n [3]:
Propoziţia 1. Fie f : [0, T ] → R o funcţie integrabilă Riemann şi g : [0, ∞) → R
o funcţie cu perioada T >0, astfel ı̂ncât restricţia g|[0,T ] este integrabilă Riemann.
Atunci
Z T Z T  Z T 
1
(1) lim f (x) g (nx) dx = f (x) dx g (x) dx .
n→∞ 0 T 0 0

O aplicaţie directă a acestui rezultat este următoarea:


Fie f : R → (0, ∞) o funcţie continuă cu perioada 1. Atunci
Z 1 Z 1 2
lim f (x) · f (nx) dx = f (x) .
n→∞ 0 0

(Cristinel Mortici, etapa judeţeană, 2003)


În lucrarea [2], se propune tot o consecinţă directă a rezultatului (1):
Fie funcţia f : R → R continuă şi periodică cu perioada T > 0. Să se demonstreze
că, pentru a, b ∈ R, a < b, avem
Z b Z
b−a T
lim f (nx) dx = f (x) dx.
x→∞ a T 0

Tot ca o consecinţă a rezultatului (1), este prezentată următoarea problemă:


Fie funcţia continuă f : [0, 1] → R. Să se demonstreze că şirul (an )n≥1 , definit
Z 1
2
de relaţia an = {nx} f (x) dx, este convergent şi să i se afle limita, ı̂n funcţia f ,
0
unde {x} = x − max {k ∈ Z; k ≤ x} ,∀x ∈ R.
(Octavian Purcaru, Lista scurtă, O.N.M., 2003)
1 Profesor, Colegiul Naţional ”Ştefan cel Mare”, Suceava

98
2
Demonstraţie. Cum funcţia g : R → Rg (x) = {x} este periodică cu perioada
principală 1, au loc:
Z 1 Z 1  Z 1  Z 1
2 1
an = lim f (x) g (nx) dx = f (x) dx {x} dx = f (x) dx,
n→∞ 0 0 0 3 0

Z 1 Z 1
2
deoarece {x} dx = x2 dx, funcţiile de integrat fiind egale pe [0, 1) .
0 0

În lucrarea [1], apare următoarea problemă, semnată de Mihail Bencze:


Z 1
Să se calculeze lim t2 {nt} dt, unde {x}are semnificaţia de mai sus.
n→∞ 0

Un al doilea rezultat util:


Propoziţia 2. Fie funcţiile continue g : [0, ∞) → R şi f : [0, a] → R, unde a >0.
Dacă lim g (x) = L ∈ R, atunci
x→∞
Z a Z a
(2) lim g (nx) f (x) dx = L f (x) dx.
n→∞ 0 0

Demonstraţie. Într-adevăr, dacă h (x) = g (x) − L, ∀x ∈ [0, ∞), atunci


Z a Z a Z a Z a
g (nx) f (x) dx = (h (nx) + L) f (x) dx = h (nx) f (x) dx + L f (x) dx.
0 0 0 0

Notând nx = t, se obţine:
Z a Z na  ‹
1 t
h (nx) f (x) dx = h (t) f dt.
0 n 0 n

Cum ∃b > 0 astfel ı̂ncât |f (x)| ≤ b, ∀x ∈ [0, a] ,


Z Z Z
a b na ab na

h (nx) f (x) dx ≤ |h (t)| dt = |h (t)| dt.
n 0 na 0
0

Dacă H este o primitivă pentru |h| pe intervalul [0, ∞) ,


Rx
|h (t)| dt H (x) − H (0)
lim 0
= lim = lim |h (x)| = 0,
x→∞ x x→∞ x x→∞
Z a
ceea ce asigură că lim h (nx) f (x) dx = 0.
x→∞ 0

Observaţie. Relaţia (2) se poate scrie şi


Z a  Z a ‹Z a
′ 1
(2 ) lim g (nx) f (x) dx = lim g (x) dx f (x) dx,
n→∞ 0 a→∞ a 0 0

99
ceea ce justifică prezentarea celor două şiruri de integrale Riemann ı̂mpreună .
O aplicaţie a acestui rezultat:
Fie funcţiile f : [0, ∞) → R şi g : [0, 1] → R, astfel ca lim f (x) = L ∈ R. Să se
x→∞
demonstreze că Z n Z 1
1  x 
lim f (x) g dx = L g (x) dx.
n→∞ n 0 n 0

(Laurenţiu Panaitopol, etapa judeţeană, 2003)


Demonstraţie. Cu substituţia x = nt, limita devine
Z 1 Z 1
lim f (nt) g (t) dt = L g (x) dx
n→∞ 0 0

şi finalizarea este clară.


Propunem cititorului următoarele exerciţii:
1. Să se demonstreze că
Z π
2
lim e−x cos nx dx =  π.
n→∞ 0 1 − e−π
Z 1
2. Să se calculeze lim {nx} [x] dx, justificându-se ı̂ntâi existenţa limitei, unde
n→∞ 0
[x] = max {k ∈ Z|k ≤ x} şi {x} = x − [x] , ∀x ∈ R.
3. Să se demonstreze că, pentru orice funcţie continuă f : [0, a] → R, are loc
relaţia Z a
lim e−nx f (x) dx = 0.
n→∞ 0

Bibliografie
1. D.M. Bătineţu - Giurgiu ş.a. – Analiză matematică. Probleme pentru clasa a
XII-a, Editura Matrix, 2004.
2. R. Miculescu ş.a. – Probleme de calcul integral, Editura GIL, 2005.
3. D. Popescu, F. Popovici – O generalizare a lemei lui Riemann, Recreaţii Matem-
atice, Iaşi, 4(2002), nr.1, 12-13.

100
Dualitatea unor sume combinatoriale
Andrei VERNESCU 1

Abstract. The main result consist in the combinatorial identity (6), where Ωn is the notation
in (1). Some connections of this identity with other combinatorial identities of similar kind, that are
known or are expected to be established, are formulated.
Keywords: combinatorial identity.
MSC 2000: 05A19.

1. În lucrarea [1], din numărul 2/2007 al acestei reviste, s-a aplicat o ideee foarte
(2k − 1)!!
elegantă pentru calculul unei sume care conţine expresiile , k = 1, 2, 3, . . . , n.
(2k)!!
În nota de faţă venim ı̂n continuarea lucrării citate, prezentând calculul bazat
pe aceeaşi idee al altei sume, care ar putea fi considerată, din punctul de vedere al
mijloacelor folosite, ca fiind duala celei precedente.
Pentru facilitarea calculelor care vor urma, vom utiliza notaţia
(2n − 1)!!
(1) Ωn = (n = 1, 2, 3, . . .).
(2n)!!
(Am introdus această notaţie ı̂n manuscrisul din 1987 al cărţii [11], am publicat-o
pentru prima dată ı̂n articolul [6], am folosit-o ı̂n toate ediţiile culegerii de probleme
[7], ca şi ı̂ntr-o serie de lucrări: [8], [9], [11] şi [13].)
Tot ı̂n scopul scrierii mai simple a formulelor şi calculelor care vor urma, este
necesar să definim Ωn şi pentru n = 0. Pentru aceasta, ı̂n cazul n ≥ 1, să amplificăm
(2n)!
fracţia din partea dreaptă a egalităţii (1) cu (2n)!! = 2n n!. Obţinem Ωn = n 2 .
(2 n!)
Ca această formulă să poată fi prelungită şi pentru n = 0, convenim să punem prin
definiţie

(2) Ω0 = 1.

Cu această notaţie, identitatea care formează rezultatul principal al articolului


citat [1] se scrie concentrat astfel:
X
n
(3) (−1)k Cnk Ωk = Ωn .
k=0

Ea poate fi considerată din două puncte de vedere: ca formulă de ı̂nsumare com-


binatorială, precum şi ca identitate referitoare la expresiile Ωk .

2. Obţinerea identităţii (3) se efectuează ı̂n [1] utilizând integralele:


Z π/2 Z π/2
(4) In = n
cos x dx = sinn x dx,
0 0
1 Conf. dr., Departamentul de Ştiinţe, Univ. ”Valahia”, Târgovişte

101
π
pentru care I0 = , I1 = 1 şi pe care, folosind notaţia (1) ı̂mpreună cu definiţia (2),
2
putem să le mai scriem sub forma:
8
π
>
< Ωk , dacă n = 2k
2
(5) In =
>
:
1 1
· , dacă n = 2k + 1.
Ωk 2k + 1
k−1
(Găsirea, pe baza formulei de recurenţă Ik = Ik−2 , k ≥ 2, a expresiilor integralei
k
In – scrise ı̂nsă fără a folosi expresia Ωk – este reamintită ı̂n [1].)
P Astfel, cu utilizarea notaţiei menţionate pentru Ωn şi a simbolului de ı̂nsumare
, putem sintetiza obţinerea rezultatului principal din [1] astfel:
Z π/2 Z π/2 € Šn
π
Ωn = I2n = cos2n x dx = 1 − sin2 x dx =
2 0 0
Z ! ‚Z Œ
π/2 X
n X
n π/2
k 2k 2k
= (−1) Cnk sin x dx = (−1) k
Cnk sin x dx =
0 k=0 k=0 0

X
n
πX
n
= (−1)k Cnk I2k = (−1)n Cnk Ωk ,
2
k=0 k=0
adică
πX
n
π
Ωn = (−1)k Cnk Ωk ,
2 2
k=0

de unde (3).
Putem aplica acum exact aceeaşi idee de calcul pornind de la integrala (4) de ordin
impar, I2n+1 . Vom obţine:
Z π/2 Z π/2 € Šn
1 1 t=sin x
· = I2n+1 = cos2n+1 x dx = 1 − sin2 x cos x dx ===
Ωn 2n + 1 0 0
Z Z !
t=sin x
1 € Šn 1 X
n
=
== 1−t 2
dt = (−1) k
Cnk t2k dt =
0 0 k=0
Z 
X
n 1 X
n
Cnk
k
= (−1) Cnk 2k
t dt = (−1)k ,
0 2k + 1
k=0 k=0
adică
X
n
Cnk 1 1
(6) (−1)k = · .
2k + 1 Ωn 2n + 1
k=0

Această identitate, care constituie rezultatul principal al lucrării noastre, nu se


mai referă la expresiile Ωk ı̂n interiorul sumei, ci conţine pe Ωn doar la rezultat, fiind

102
(spre deosebire de (3), care admite două interpretări) doar o identitate combinatorială.
Dar, din punct de vedere al procedeului de deducere folosit, ea constituie o identitate
duală celei din [1], adică (3).
Însă, ca de multe ori ı̂n domeniul identităţilor combinatoriale, identitatea găsită
nu este nouă! Într-adevăr, ı̂n cartea lui H. W. Gould [2], la pag. 6, este prezentată
identitatea  
X n
k n x 1
(−1) = x+n (x ∈ R)
k x+k n
k=0
 
α
ı̂n care notaţia consacrată , pentru α ∈ R şi k ∈ N∗ , are semnificaţia:
k
 
α α(α − 1)(α − 2) · . . . · (α − k + 1)
= ,
k 1 · 2 · 3 · ... · k
   
α def n
cu == 1, iar dacă α = n ∈ N şi k ≤ n se obţin combinările, = Cnk .
0 k
1
Introducând ı̂n identitatea lui Gould x = , se obţine (6).
2
3. Prin prisma identităţilor combinatoriale, egalitatea (6) ocupă poziţia a patra
din următoarea succesiune de formule:

def
X
n
Cnk 2n+1 − 1 def
X n
Ck 1
Sn === = ; Sn(a) ==
= (−1)k n = ;
k+1 n+1 k+1 n+1
k=0 k=0
def
X
n
Cnk def
Xn
Cnk 1 1
Tn ==
= = f (n); Tn(a) ==
= (−1)k = · ,
2k + 1 2k + 1 Ωn 2n + 1
k=0 k=0

unde a desemnează o sumă alternată, iar f (n) este o expresie neprecizată.


(a)
Găsirea sumelor Sn şi Sn este binecunoscută (o cale fiind integrarea identităţilor
X
n X
n
(1+t)n = Cnk tk , respectiv (1−t)n = (−1)k Cnk tk , pe intervalul [0, 1]). Obţinerea
k=0 k=0
(a)
sumei Tn a format obiectul prezentei note, expus ı̂n secţiunea 2 şi astfel, se mai pune
problema calculului sumei Tn . O altă problemă de studiu ar putea-o constitui calculul
X
n
unei sume asemănătoare cu cea din (3), dar nealternată, anume Cnk Ωk .
k=0

Bibliografie
1. A. Corduneanu, Gh. Costovici – Un şir strâns legat de şirul lui Wallis, Recreaţii
Matematice, Anul IX, Nr. 2 iulie-decembrie 2007, 95-96.
2. H. W. Gould – Combinatorial identities, Morgantown Printing and Binding, Co.,
1972.
3. R. L. Graham, D. E. Knuth, O. Patashnik – Concrete Mathematics, Addison
Wesley Longman, Reading MA, 1994.

103
4. D. E. Knuth – The art of Computer Programming, Vol. 1, Addison Wesley Longman,
Reading, MA 1977 (traducerea la Ed. Teora, Bucureşti).
5. J. Riordan – Combinatorial Identities, J. Wiley & Sons, New York, 1968.
6. L. Tóth, A. Vernescu – Dezvoltarea asimptotică a şirului lui Wallis, G. M.-A, 11
(1990), Nr. 1, 26-29.
7. A. Vernescu – Analiză matematică. Probleme. Calcul diferenţial, Editura Pantheon,
Bucureşti, 1991.
8. A. Vernescu – Ordinul de convergenţă al şirului lui Wallis, G. M.-A, 12 (1991), 7-8.
9. A. Vernescu – Asupra unui tip de relaţie de recurenţă, Revista Matematică din
Timişoara (seria a IV-a), nr. 4/2003, 8-14.
10. A. Vernescu – Şiruri de Numere Reale, Ed. Univ. Bucureşti, 2004.
11. A. Vernescu – Numărul e şi matematica exponenţialei, Ed. Univ. Bucureşti, 2004.
12. A. Vernescu – The natural proof of the inequalities of Wallis type, Libertas Mathe-
matica, 24 (2004), 183-190.
13. A. Vernescu, C. Mortici – New results in discrete asymptotic analysis, General
Mathematics, to appear.

(continuare de la pagina 9)
Mă aflu ı̂n posesia unei demonstraţii minunate a acestei afirmaţii,
dar marginea paginii este prea strâmtă pentru a o cuprinde.
Care avea să fie o provocare timp de peste 350 de ani pentru multe generaţii de
matematicieni.
Abia ı̂n 1995 Andrew Wiles a demonstrat afirmaţia, punând capăt provocării lui
Fermat. Drumul parcurs de lumea matematică până la acest final a fost presărat de
tentative de demonstraţie, eşecuri, descoperiri epocale, drame şi tragedii individuale,
dezvăluiri senzaţionale ı̂n mass-media etc.
N-au lipsit nici glumele pe marginea provocării lui Fermat:

În staţia de metrou de pe Eighth Street, New York a apărut inscripţia:


xn + y n = z n : nu există soluţie. Am descoperit o demonstraţie cu
adevărat remarcabilă a acestui fapt dar nu pot s-o scriu fiindcă-mi
vine trenul.
Sau catrenul:
”Pe untul meu sunt multe litere scrise”,
Supărat un client la o masă răcnise;
”N-am avut loc, răspunse piccolo-ul Pierre,
Nici pe margine, nici pe rafturile din frigider.”

(Simon Singh–Marea Teoremă a lui Fermat, Humanitas, Bucureşti, 2005.)

104
Asupra problemei C.O: 5004 din G.M.
1
Dan Mihai MOCANU

Abstract. In this Note, the problem C.O.: 5004 of the journal G.M. – 2/2009, p.103 is gene-
ralized and a couple of simple results in a triangle are established; they deal with isogonal lines and
angle trisectors.
Keywords: altitudes, angle bisector, isogonal lines, angle trisectors, symmedians.
MSC 2000: 51F20.

Scopul prezentei note este de a stabili câteva rezultate A


ı̂ntr-un triunghi, legate de cevienele izogonale care sunt ı̂n
acelaşi timp şi trisectoare.
Fie ABC un triunghi oarecare şi A1 , A2 ∈ (BC). O .. H
Se spune că cevienele (AA1 , (AA2 sunt izogonale dacă
Ö1 ≡ CAA
BAA Ö2 şi că sunt trisectoare ale unghiului A b dacă
Ö1 ≡ A
BAA × Ö
1 AA2 ≡ A2 AC.
B A1 A2 C
Afirmaţia următoare are o demonstraţie imediată.

Propoziţie. Dacă (AA1 şi (AA2 sunt ceviene izogonale şi una din ele este bisec-
b
toarea unghiului dintre o latură şi cealaltă, atunci ele sunt trisectoarele unghiului A.
Demonstraţie. Să presupunem că (AA1 este bisectoarea unghiului BAA2 (la Ö
fel demonstrăm dacă (AA2 ar fi bisectoarea unghiului A Ö Ö
1 AC). Avem, deci, BAA1 ≡
×
A Ö Ö
1 AA2 . Cum BAA1 ≡ A2 AC (AA1 şi AA2 sunt izogonale), rezultă că BAA1 ≡
Ö
× Ö
A1 AA2 ≡ A2 AC, ceea ce trebuia demonstrat.

Consecinţa 1. (C.O.: 5004 din G.M.-2/2009, p.102). În triunghiul ascuţitunghic


ABC se consideră ı̂nălţimea AH. Să se arate că, dacă bisectoarea unghiului BAHÕ
trece prin centrul cercului circumscris triunghiului ABC, atunci semidreapta (AH
Õ
este trisectoare a unghiului BAC.
Laura Constantinescu
Demonstraţie. Se ştie că ı̂nălţimea coborâtă din A şi diametrul ce are A ca o
extremitate a sa sunt izogonale. Dar, prin ipoteză, acest diametru este bisectoarea
Õ Conform propoziţiei precedente (AH este trisectoare a unghiului A).
unghiului BAH. b

Consecinţa 2. Dacă AH este ı̂nălţime, O este centrul cercului circumscris tri-


Õ atunci (AO
unghiului ascuţitunghic ABC, iar (AH este bisectoarea unghiului OAC,
Õ
este trisectoare a unghiului BAC.
Demonstraţie. Se procedează ca ı̂n Consecinţa 1.
Observaţie. Dacă una dintre (AH sau (AO este trisectoare, atunci şi cealaltă
este trisectoare.
1 Elev, cl. a IX-a, Colegiul Naţional, Iaşi

105
Următoarele rezultate pot fi privite ca reciproce ale consecinţelor de mai sus.
Reciprocă (a Consecinţei 1). Dacă (AA2 este o trisectoare a triunghiului ascu-
Ö2 , atunci A2 este ı̂nălţime.
ţit-unghic ABC şi (AO este bisectoarea unghiului BAA
Demonstraţie. Din condiţiile ipotezei, rezultă că (AA2 şi (AO fac cu laturile
A
triunghiului unghiuri de măsură . Atunci şi (AH, ca izogonală a lui (AO, face cu
3
A
AC un unghi de măsură . Ca urmare (AA2 coincide cu (AH.
3
Reciprocă (a Consecinţei 2). Dacă (AA1 este o trisectoare a triunghiului ascuţit-
unghic ABC şi (AH este bisectoarea unghiului A Ö 1 AC, atunci O ∈ (AA1 .
Demonstraţie. La fel ca pentru reciproca precedentă.
Observaţie. În loc de ”ı̂nălţime-diametru”, putem lua o altă pereche de ceviene
izogonale; de exemplu, ”mediană-simediană”. Trecerea rezultatelor de mai sus la
noua pereche se face cu uşurinţă.
În ı̂ncheiere, vom rezolva o problemă strâns legată de cele de mai sus.
Problemă. Fie un triunghi ABC, AB > AC. Fie (AM ) mediana relativ la
latura (BC) şi (AN ) simediana corespunzătoare. Arătaţi că, dacă (AM sau (AN
Õ atunci AB = 2 cos A .
este trisectoare a unghiului BAC,
AC 3
Demonstraţie. Ambele vor fi trisectoare. Cu ajutorul ariilor, avem

Ö = AC · AM sin CAM
AB · AM sin BAM Ö,

de unde
AB Ö
sin CAM sin 2A A
3
= = = 2 cos .
AC Ö
sin BAM sin A
3
3

În toamna anului 1999 a apărut primul număr al revistei Recreaţii Matematice.
În anul acesta se ı̂mplinesc 10 ani de la apariţia acesteia.

Scrieţi numărul 2009 cu ajutorul numarului 10 şi folosind numai operaţiile de


adunare şi ı̂mpărţire !
Care este numărul maxim de operaţii cu care puteţi face acest lucru ?
Dar cel minim ?
(Nu se acceptă termeni nuli !)

Notă. Răspunsurile la aceste ı̂ntrebări sunt date la pag.

106
Matrices à coefficients dans un corps fini
Adrien REISNER1

Abstract. It is considered the set Ap of matrices of order 2 with their entries in Zp , defined by
Ap = {a = λM + µI; λ, µ ∈ Zp }, and some properties of this set are presented (Theorems 1,3,6,7,9).
Keywords: unitary ring, the order of an element, the field Zp , isomorphism.
MSC 2000: 15A33.

A étant un anneau unitaire d′ éléments neutre e on appelle ordre d′ un élément


inversible a de A le plus petit entier positif n del que an = e; dans ce cas l′ ensemble
Ga = {e, a, a2 , . . . , an−1 } forme un sous-groupe du groupe G des éléments inversibles
de l′ anneau A: l′ ordre n de l′ élément a ∈ G est le cardinal du ce sous-groupe Ga .
Pour toute matrice carrée M à coefficients dans un corps on désigne par T et ∆
respectivement les deux applications suivantes: T :M →T (M )=trace de la matrice M,
∆ : M → ∆(M )=déterminant de la matrice M .
p désignant un nombre premier strictement supérieur à 3, on considère le corps
fini Zp des classes résiduelles modulo p. M et I étant les deux matrices suivantes à
coefficients dans Zp :
   
b
4 b 1 b
1 b 0
M= , I= b b ,
−b1 b0 0 1
on considère l′ ensemble Ap des matrices d′ ordre 2 à coefficinets dans Zp défini par:

Ap = {a = λM + µI; λ, µ ∈ Zp }.

Théorème 1. L′ ensemble Ap est un anneau commutatif unitaire pour les opérations


usuelles. De plus: Card Ap = p2 .
Démonstration. On a immédiatement M 2 = b 4M − I; on en déduit compte tenu
de la structure de l′ ensemble M2 (Zp ) que Ap est un algèbre associative et unitaire.
Enfin, la commutativité se vérifie directement. D′ autre part, (I, M ) étant une base
de l′ algèbre Ap , on a Ap ≃ Z2p et par suite: Card Ap = (Card Zp )2 = p2 .
La proposition suivante se vérifie immédiatement par le calcul.

Proposition 2. Pour toute matrice A = λM + µI ∈ Ap , on a:


a) T (A) = b
2(b
2λ + µ), ∆(A) = λ2 + µ2 + b 4λµ;
b) T (A ) = T 2 (A) − 2∆(A).
2

En particulier, T (A2 ) = b
2(b
7λ2 + µ2 + b
4λµ).

Théorème 3. Pour A ∈ Ap , deux quelconques des conditions suivantes impliquent


la troisième: a) T (A) = b
0, b) ∆(A) = b
1, c) A est une matrice d′ ordre 4.
1 Centre de Calcul E.N.S.T., Paris; e-mail: adrien.reisner@enst.fr

107
Démonstration. a) + b) ⇒ c) Le théorème de Cayley-Hamilton appliquée à la
matrice A donne:
A2 − T (A)A + ∆(A)I = O.
Il vient alors avec les hypothèses a) et b): A2 = −I et A4 = I. L′ ordre de la matrice
A devise donc 4 et comme A2 ̸= I (p étant impair), 2 n′ est pas multiple de cet ordre.
Finalement, A est d′ ordre 4.
a) + c) ⇒ b) Supposant les conditions a) et c) vérifiées, on a:

A2 = −∆(A)I et I = A4 = ∆2 (A)I.

On en déduit: ∆2 (A) = b
1, ∆(A) ̸= −b
1 soit ∆(A) = b
1.
b) + c) ⇒ a) Compte tenu de b) et c), A2 = T (A)A − I d′ où

I = A4 = T 2 (A)A2 − b
2T (A)A + I = T (A)[T 2 (A) − b
2]A + [b
1 − T 2 (A)]I.

On peut distinguer deux cas: I A et I sout liés, i.e. A = µI. Dans ce cas, A2 = µ2 I,
A4 = µ4 I, d′ oú, A étant d′ ordre 4: µ2 ̸= b
1, µ4 = b1, µ2 = −b1 et A2 = −I, T (A)A = 0.
Si T (A) ̸= 0, alors A = O d où T (A) = 0, impossible. Par suite T (A) = b
b ′ b 0. II {A, I}
est une famille libre. Dans ce cas l′ égalité I = T (A)[T 2 (A) − b 2]A + [b
1 − T 2 (A)]I
implique: b1=b 1 − T 2 (A) d′ oú T (A) = b0. Le théorème est ainsi démontré.
On considère la suite des entiers {Yk }k≥0 définie par Y0 = 2 et Yk+1 = 2Yk2 − 1,
k ≥ 0, dont les premiers terms sont: Y0 = 2, Y1 = 7, Y2 = 97, Y3 = 18817, . . .
k
Théorème 4. Pour tout k ≥ 0, on a 2Yk ∈ T (M 2 ).
Démonstration. Par récurrence sur k. Pour k = 0 la propriété se vérifie trivi-
alment puisque 2Y0 = 4 ∈ b 4 = T (M ). Supposant la propriété vérifiée à l′ ordre k,
démontrons-là pour k + 1. On a immédiatment d′ une part: 2Yk+1 = 4Yk2 − 2 =
(2Yk )2 − 2 et d′ autre part T (M 2 ) = T [(M 2 )2 ] = T 2 (M 2 ) − b
k+1 k k
2, compte tenu
de l′ assertion b) de la Proposition 2 et puisque ∆(M ) = b 1. Donc, compte tenu de
l′ hypothèse de récurrence: 2Yk+1 = (2Yk )2 − 2 ∈ T (M 2 ).
k+1

Théorème 5. La matrice M est d′ ordre 2k si et seulement si p|Yk−2 .


Supposons la matrice M d′ ordre 2k , i.e. M 2 = I. On a:
k
Démonstration.
 
c
15 b
4
M2 = et par suite l′ ordre de M ne divise pas 2 soit k ≥ 2. En posant
−b4 −b 1
on a, M étant d′ ordre 2k : A2 = M 2
k−2 k−1 k
alors A = M 2 ̸= I, A4 = M 2 = I.
A ∈ Ap est donc une matrice d′ ordre 4 vérifiant aussi ∆(A) = b 1. Compte tenu du
théorème 3, b) + c) ⇒ a), il vient alors: T (A) = b 0 soit d′ après le théorème précédent:
2Yk−2 ∈ T (A) = b 0, i.e. p divise 2Yk−2 et finalement p étant impair p|Yk−2 .
Réciproquement, avec les mêmes notations si p|Yk−2 i.e. Yk−2 ≡ 0 (mod p), alors
T (A) = b0. Mais, comme ∆(A) = b 1, le théorème 3, a) + b) ⇒ c), montre que la matrice
A est d′ ordre 4 soit M 2 = I et M 2 = A2 ̸= I. L′ ordre de M divisant 2k mais non
k k−1

2k−1 est donc égal à 2k .

108
Théorème 6. Les deux assertions suivantes sont équivalentes:
a) b
3 n′ est pas le carré d′ un élément de Zp ;
b) Ap est un corps.
Démonstration. a) ⇒ b) Si b 3 n′ est pas le carré d′ un élément de Zp , alors pour
A ∈ Ap :
∆(A) = b
0 ⇒ A = O.

En effet, supposons ∆(A) = b 0. Pour A ∈ Ap on a: ∆(A) = b 0 = (λ + b


2µ)2 − b
3µ2 .
µ ̸= b0 impliquerait b 3 = (λµ + b
−1
2) ce qui est exclut par hypothèse. Donc µ = b
2
0 et
∆(A) = b 0 = λ2 soit: λ = b0 et finalement: A = O. Donc l′ ensemble Ap est formé de
la matrice nulle et de matrices inversibles dans M2 (Zp ).
A ̸= O étant une matrice de Ap , l′ application Ap → Ap définie par X → AX est
linéaire et injective, donc surjective (dim Ap = 2). Pour A ̸= O il existe B ∈ Ap telle
que: AB = 1 soit A−1 = B ∈ Ap : les inverse des matrices non nulles de Ap sont dans
Ap .
b) ⇒ a) Nous allons montrer que, non a) ⇒ non b). Supposons qu′ il existe a ∈ Zp
tel que a2 = b 3. Dans ce cas: ∆(A) = [λ + (b 2 − a)µ][λ + (b
2 + a)µ]. Pour la matrice
A = (a − 2)M + I(a ̸= 2 puisque 2 − 3 = 1) on a ∆(A) = b
b b b2 b b 0 et (Cayley-Hamilton)
A(A − T (A)I) = O avec A ̸= O et A ̸= T (A)I. On en déduit que Ap n′ est pas intègre
et par suite que Ap n′ est pas un corps.

Théorème 7. En supposant que b 3 est un carré dans Zp :


a) M est semblable à une matrice diagonale;
b) Ap est isomorphe à l′ anneau produit Zp × Zp ;
c) dans Ap il y a p − 1 éléments de déterminant b 1 et (p − 1)2 éléments inversibles.
Démonstration. a) L′ équation caractéristique de la matrice M s′ écrit X 2 − b 4X +
b
1 = (x − b 2)2 − a2 = 0 où a est tel que a2 = b3. M ayant deux valeurs propre distinctes
λ1 = b 2 + a et λ2 = b 2 − a, on en déduit
 que la matrice
 M est diagonalisable i.e. il
b2+a b
0 −1
existe P ∈ GL2 (Zp ) telle que M = b b
P .
0 2−a
b) L′ application R → P −1 RP qui est un automorphisme pour la structure d′ anneau,
transforme toute matrice A = λM +µI de Ap en une matrice diagonale. Or l′ ensemble
Dp des matrices diagonales de M2 (Zp ) ayant comme cardinal p2 (je rappelle que Card
Ap = p2 ), l′ application Ap → Dp , A → P −1 AP est donc un isomorphisme.
 
α b 0
D′ autre part, l′ application Zp × Zp → Dp , (α, β) → b , étant de façon
0 β
immédiate un isomorphism, on en déduit que: Ap ≃ Zp × Zp (cet isomorphisme est
même un isomorphisme d′ algèbre).
c) Compte tenu de l′ isomorphisme précédent, comme ∆(A) = ∆(P −1 AP ), le
nombre de matrices A ∈ Ap telles que ∆(A) = b 1 est égal aux nombre de couple (α, β)
vérifiant αβ = b 1. Il y en a p − 1 (choisir d′ abord α).
La démonstration de l′ implication a) ⇒ b) du Théoreme 6 a montré que ∆(A) ̸=
b
0 ⇒ A−1 ∈ Ap . On en déduit que le nombre de matrices A inversibles de Ap (i.e. le

109
nombre des matrices A ∈ Ap telles que ∆(A) ̸= b 0) est égal aux nombre des couples
b
(α, β) ∈ Zp × Zp vérifiant αβ = 0 soit (p − 1) .
2

Corollaire 8. Dans le cas où b 3 est un carré dans Zp :


a) l′ ordre de la matrice M divise p − 1;
b) si p|Yk−2 , alors 2k |p − 1.
Démonstration. a) L′ ensemble des matrices de Ap de déterminant b 1 forment un
sous-groupe multiplicatif de A∗p , groupe des matrices inversibles de Ap (si A ∈ Ap et
∆(A) ̸= b 0, alors A−1 ∈ Ap –voir a) ⇒ b) du théorème 6). Compte tenu de l′ assertion
c) du théorème précédent ce sous-groupe est fini de cardinal p − 1. Or la matrice
M appartient à ce sous-groupe. L′ ordre de M divise par suite p − 1 (ordre de ce
sous-groupe).
b) Si p divise Yk−2 , alors la matrice M est d′ ordre 2k d′ aprés le Théorème 5.

L assertion a) de ce corollaire permet alors de conclure.

Théorème 9. En supposant que b 3 n′ est pas un carré dans Zp :


a) ∆ est un homomorphisme du groupe multiplicatif des éléments non nuls de Ap
dans celui des éléments non nuls de Zp ;
b) il existe k tel que p − 1 = Card (Im ∆) et Card Ker∆ = k(p + 1);
c) il existent p + 1 éléments de déterminant b 1 dans Ap .
Démonstration. a) Ap étant un corps d′ après le Théorème 6, l′ assertion a) est
évidente.
b) On en déduit l′ isomorphisme Im ∆ ≃ A∗p /Ker∆ et par suite: Card A∗p = p2 −1 =
(Card Im ∆)(Card Ker ∆). De plus, Im ∆ est un sous-groupe de Z∗p : il existe k tel
que p − 1 = k Card (Im ∆), d′ où Card Ker ∆ = k(p + 1). Notons que 1 ≤ k ≤ p − 1.
c) Les matrices A de Ker ∆ vérifient ∆(A) = b 1. Il s′ agit de montrer que Card
b
Ker ∆ = p + 1. L égalité ∆(A) = 1 entraı̂ne, compte tenu de l′ assertion a) de la

Proposition 2: λ2 + µ2 + b 4λµ − b
1=b 0 (1). λ ∈ Zp étant donné, il existe donc 0, 1 ou
2 éléments µ ∈ Zp tels que ∆(A) = b 1 (Zp étant un corps), donc il existe au plus 2p
couples (λ, µ) vérifiant l′ équation (1). D′ autre part, le nombre de tels couples est égal
à-voir b)-k(p + 1) = Card Ker ∆. On en déduit finalement que k = 1 et par suite
Card Ker ∆ = p + 1.

Corollaire 10. Dans le cas où b 3 n′ est pas un carré dans Zp :



a) l ordre de la matrice M divise p + 1;
b) si p|Yk−2 , alors 2k |p + 1.
Démonstration. a) La matrice M de déterminant b 1 appartient au sous-groupe
Ker ∆. L′ ordre de M divise par suite l′ ordre p + 1 de ce sous-groupe Ker ∆ -voir
l′ assertion c) du Théorème 9.
b) De même qu′ au Corollaire 8, si p divise Yk−2 , alors la matrice M est d′ ordre
2 d aprés le Théorème 5. L′ assertion a) de ce corollaire permet alors de conclure.
k ′

Remarque. Dans le cas où b


3 n′ est pas un carré dans Zp , l′ ordre de toute matrice
b
A de Ap vérifiant ∆(A) = 1 divise p + 1 - ordre de sous-groupe Ker ∆. Par suite
∀A ∈ Ap : Ap+1 = I.

110
O metodă de rezolvare a problemelor
Maria MIHEŢ1
În această notă vom evidenţia o strategie importantă de rezolvare a problemelor:
demonstraţia prin exprimarea ı̂n două moduri a unor mărimi. Această metodă s-a
folosit ı̂ncă ı̂n primele clase de gimnaziu, pentru obţinerea unor ecuaţii. Am ı̂ntâlnit-o,
de asemenea, ı̂n demonstraţia formulei pentru suma unghiurilor unui poligon convex:
cu ajutorul unui punct O din interiorul poligonului se triangulează poligonul şi se
exprimă ı̂n două moduri suma unghiurilor triunghiurilor obţinute. Prin exprimarea
ı̂n două moduri a ariei sau volumului se pot afla anumite distanţe (ne amintim, de
exemplu, cum se poate afla ı̂nălţimea din vârful unghiului drept ı̂ntr-un triunghi
dreptunghic prin exprimarea ı̂n două moduri a ariei triunghiului), iar multe identităţi
combinatoriale pot fi demonstrate folosind numărarea ı̂n două moduri.
Am exemplificat metoda prin câteva probleme tip, iar la sfârşitul lucrării am
ı̂ntocmit o listă de probleme ı̂nsoţite de indicaţii de rezolvare. Am dori ca cititorii să
ı̂ncerce mai ı̂ntı̂i să le rezolve fără a apela la indicaţii.
Această notă se adresează ı̂n special elevilor din clasele VI-VIII. Ea poate fi ı̂nsă
completată cu multe exemple de nivel liceal.
Problema 1. Fie q un număr real, iar n ∈ N, n ≥ 2. Să se calculeze, ı̂n funcţie
de q, suma Sn = 1 + q + q 2 + . . . + q n .
Soluţie. Exprimăm Sn+1 ı̂n două moduri: mai ı̂ntâi Sn+1 = Sn + q n+1 , iar, pe
de altă parte, Sn+1 = qSn + 1. Rezultă că Sn + q n+1 = qSn + 1, de unde obţinem că
(∗) (q − 1)Sn = q n+1 − 1.
q n+1 − 1
Astfel, dacă q ̸= 1, atunci Sn = . Dacă q = 1, prin ı̂nlocuire ı̂n Sn se
q−1
obţine direct că Sn = n + 1. Aşadar,
8
<n + 1, dacă q = 1;
Sn = q n+1 − 1
: ̸ 1.
, dacă q =
q−1
Problema 2. Fie n ≥ 2 un număr natural şi Dn = {d1 , . . . , dk } mulţimea divizo-
d1 + . . . + dk
rilor naturali ai lui n. Demonstraţi că 1 1 este un număr natural.
d1 + . . . + dk
Soluţie. Observăm că d §este un divizor
ª
al lui n dacă şi numai dacă nd este divizor
n n
al lui n. Mai mult, mulţimea ,..., este egală cu mulţimea Dn . Scriind că suma
d1 dk
d1 + . . . + dk
elementelor ı̂n aceste mulţimi este aceeaşi, obţinem egalitatea 1 1 = n.
d1 + . . . + dk
1 Profesor, Şcoala cu clasele I-VIII nr. 24, Timişoara

111
Problema 3. Să se arate că ı̂n orice triunghi ABC au loc relaţiile:
a b c
a) = = (teorema sinusurilor );
sin A sin B sin C
BD AB
b) = unde D este piciorul bisectoarei unghiului A (teorema bisectoarei ).
CD AC
b c
Soluţie. a) Vom arăta că = ; pentru aceasta, fie M mijlocul laturii
sin B sin C
[BC]. Considerând triunghiurile ABM şi ACM cu ı̂nălţimile din A, obţinem că
SABM = SACM .
Pe de altă parte, 2SABM = AB · BM · sin ∠(ABC), iar 2SACM = AC · CM ·
b c
sin ∠(ACB). Cum CM = BM , rezultă că b sin C = c sin B, adică = .
sin B sin C
SABD BD · ha BD
b) Considerăm triunghiurile ABD şi ACD. Atunci = = .
SACD CD · ha CD
Pe de altă parte, deoarece D se află pe bisectoarea unghiului A, ı̂nălţimile din D
SABD AB
ale triunghiurilor ABD şi ACD sunt egale; prin urmare = . Egalând
SACD AC
rapoartele, obţinem teorema bisectoarei.
În cele ce urmează, propunem celor interesaţi o serie de probleme care pot fi
rezolvate folosind această strategie.
1) Vasul A conţine apă, iar vasul B conţine alcool pur. Se toarnă ı̂n A un pahar
plin de alcool din B, apoi se scoate din A un pahar de amestec, care se toarnă ı̂n B.
Ce relaţie există ı̂ntre cantitatea de alcool din A şi cantitatea de apă din B?
Indicaţie. Exprimaţi ı̂n două moduri cantitatea de apă care lipseşte din A după
cele două operaţii.
2) Doi călători au plecat ı̂n acelaşi moment din localităţile A şi B, fiecare de-
plasându-se spre localitatea celuilalt cu viteză constantă. Ei s-au ı̂ntâlnit la ora 13 şi,
continuându-şi drumul, primul a ajuns ı̂n B la ora 17, iar cel de-al doilea ı̂n A la ora
21. La ce oră au plecat cei doi ı̂n călătorie?
Indicaţie. Calculaţi ı̂n două moduri distanţele parcurse de cei doi călători până la
ı̂ntâlnire.
3) O motonavă a plecat ı̂ntr-o cursă pe mare. Când motonava s-a ı̂ndepărtat cu
180 mile de ţărm, a fost trimis după ea un hidroavion cu un mesaj urgent. Viteza
hidroavionului este de zece ori mai mare decât viteza motonavei. La ce distanţa de
ţărm a fost ajunsă motonava?
Indicaţie. Exprimaţi ı̂n două moduri timpul până la ı̂ntâlnire.
4) 98 de numere naturale consecutive a1 < a2 < . . . < a98 au suma 137. Aflaţi
a2 + a4 + a6 + . . . + a98 .
Indicaţie. Exprimaţi ı̂n două moduri a1 + a2 + a3 + . . . + a98 .
5) În fiecare din pătrăţelele unui tabel dreptunghiular cu 4 linii şi 5 coloane scriem
câte un număr natural, astfel ı̂ncât suma numerelor de pe fiecare linie să fie aceeaşi şi
suma numerelor de pe fiecare coloană să fie aceeaşi (nu neapărat egală cu cea de pe
linii). Fie S suma numerelor din tabel. a) Putem avea S = 30? b) Daţi două exemple
de tabele cu S = 20. c) Aflaţi toate tabelele cu S < 20.
Indicaţie. Calculaţi S ı̂n două moduri.
6) Există poligoane convexe cu mai mult de trei unghiuri ascuţite?

112
Indicaţie. Calculaţi ı̂n două moduri suma unghiurilor poligonului.
7) Un număr natural n ≥ 2 cu număr impar de divizori este pătrat perfect.
Indicaţie. Scrieţi
n ı̂n două moduri mulţimea divizorilor lui n şi grupaţi divizorii ı̂n
no
perechi de forma d, .
d
8) Fie {d1 , . . . , dk } mulţimea divizorilor naturali ai numărului natural n ≥ 2.
Demonstraţi că (d1 d2 . . . dk )2 = nk .
Indicaţie. Scrieţi ı̂n două moduri mulţimea divizorilor lui n.
9) Demonstraţi că dacă m ≥ 2, n ≥ 2 sunt numere naturale, iar m divide n, atunci
2m − 1 divide pe 2n − 1.
Indicaţie. Calculaţi ı̂n două moduri 1 + 2 + .§. . + 2n−1 . ª
1 1
10) Aflaţi cea mai mică valoare pentru max a + , b + , când a, b ∈ (0, ∞).
b a
1 1
Indicaţie. Evaluaţi ı̂n două moduri suma dintre a + şi b + .
b a
11) Pe latura (BC) a triunghiului ABC, se consideră punctele D, E astfel ı̂ncât
BD BE AB 2
∠BAD ≡ ∠CAE. Demonstraţi că · = (Steiner ).
DC EC AC 2
SABD SABE
Indicaţie. Exprimaţi ı̂n două moduri şi .
SACD SACE

12) În triunghiul ascuţitunghic ABC, ı̂nălţimea AA şi mediana CM au aceeaşi
lungime. Aflaţi m(∠M CB).
Indicaţie. Exprimaţi ı̂n două moduri aria triunghiului M CB.
13) Fie n ∈ N. Demonstraţi că
1 1 1 1 1 1 1 1
1− + − +. . .+ − = + +. . .+ (Botez-Catalan).
2 3 4 2n − 1 2n n+1 n+2 2n
1 1 1 1 1 1
Indicaţie. Calculaţi ı̂n două moduri suma: 1+ 2 + 3 +. . .+ n + n+1 + n+2 +. . .+ 2n .
14) Produsul a două numere de forma x − 2y , x, y ∈ N, este un număr de aceeşi
2 2

formă. √ √ √ √
Indicaţie. Exprimaţi ı̂n două moduri (a + 2b)(c + 2d)(a − 2b)(c − 2d).
15) Fie k ≥ 3 un număr natural impar. Arătaţi că dacă x1 , x2 , . . . , xk sunt
numere ı̂ntregi astfel ı̂ncât |x1 − x2 | = |x2 − x3 | = . . . = |xk−1 − xk | = |xk − x1 |, iar
x1 + x2 + . . . + xk = m, atunci k ı̂l divide pe m.
Indicaţie. Exprimaţi ı̂n două moduri suma (x1 − x2 ) + (x2 − x3 ) + . . . + (xk−1 −
xk ) + (xk − x1 ).
16) Aflaţi numerele N = a0 a1 . . . a9 de 10 cifre (ı̂n baza 10) cu proprietatea că a0
este numărul cifrelor de 0 ale lui N , a1 este numărul de 1-uri din scrierea lui N , . . . ,
a9 este numărul cifrelor de 9 ale lui N.
Indicaţie. Calculaţi ı̂n două moduri suma cifrelor lui N .
17) Numerele a1 , a2 , . . . , an aparţin mulţimii {−1, 1}, iar a1 a2 + a2 a3 + . . . +
an−1 an + an a1 = 0. Arătaţi că n este multiplu de 4.
Indicaţie. Calculaţi ı̂n două moduri produsul (a1 a2 )(a2 a3 ) . . . (an a1 ).
18) Un tablou cu m linii şi n coloane are toate elementele egale cu 1 sau cu −1,
iar produsul elementelor de pe fiecare linie şi fiecare coloană este −1. Demonstraţi că
m şi n au aceeaşi paritate.
Indicaţie. Calculaţi ı̂n două moduri produsul elementelor din tablou.

113
19) Într-un tablou cu 25 de linii şi 25 de coloane pentru fiecare i, j ∈ {1, . . . , 25}
scriem la intersecţia dintre linia i şi coloana j numărul 1, dacă i divide pe j şi 0, dacă
i nu divide pe j. Demonstraţi că numărul zerourilor din tablou este par.
Indicaţie. Calculaţi ı̂n două moduri suma elementelor tabloului.
20) La un turneu de şah au participat 14 şahişti, fiecare jucând câte o partidă cu
fiecare din ceilalţi 13. La sfârşitul turneului s-a constatat că suma punctelor obţinute
de primii trei clasaţi a fost egală cu suma punctelor ultimilor 9. Ştiind că meciul
dintre şahiştii de pe locurile 4 şi 5 nu s-a terminat remiză, aflaţi câte puncte a obţinut
şahistul clasat pe locul 4 (pentru victorie se primeşte 1p, pentru remiză 12 p, iar pentru
ı̂nfrângere niciun punct).
Indicaţie. Calculaţi ı̂n două moduri suma punctelor primilor trei şahişti.
21) Putem scrie pe tablă 17 numere reale astfel ı̂ncât suma oricăror 7 dintre ele
să fie strict pozitivă, iar suma oricăror 11 să fie strict negativă?
Indicaţie. Dacă x1 , . . . , x17 sunt cele 17 numere, calculaţi ı̂n două moduri suma
elementelor din tabloul (cu 11 linii şi 7 coloane):

x1 x2 . . . x7
x2 x3 . . . x8
..................
x11 x12 . . . x17 .

22) La o adunare a zeilor ı̂n Olimp, fiecare zeu a primit ı̂n cupa sa aceeaşi cantitate
de ambrozie şi are voie să toarne din cupa sa ı̂n cupa altui zeu, ı̂nsă doar o cantitatea
de ambrozie egală cu cea pe care o are celălalt zeu. La sfârşitul adunării, toată
ambrozia a ajuns ı̂n cupa lui Zeus. Demonstraţi că numărul participanţilor a fost o
putere a lui 2.
Indicaţie. Scrieţi ı̂n două moduri cantitatea iniţială de ambrozie din cupa lui Zeus.

Bibliografie
1. A. Engel - Probleme de matematică – strategii de rezolvare, GIL, Zalău, 2006.
2. M. Miheţ, E. Obădeanu - Teste şi probleme comentate pentru Concursul ”Traian
Lalescu” (gimnaziu), 19 (1988).
3. L. Niculescu - Teme de algebră pentru gimnaziu, Ed. Cardinal, 1993.
4. G. Polya - Cum rezolvăm o problemă?, Ed. Ştiinţifică, 1965.
5. *** - Colecţia RMT, 1996-2009.

Vizitaţi noua pagina web a revistei:

http://www.recreatiimatematice.ro

114
Metoda identificării
Silviu BOGA1

Ne ocupăm ı̂n cele ce urmează de demonstrarea unor identităţi ce permit calculul


unor sume sau al unor produse, prin formule de tipul

X
n Y
n
ak = bn , ∀n ∈ N∗ , respectiv ak = bn , ∀n ∈ N∗ .
k=1 k=1

În ambele situaţii, cele mai populare strategii de abordare sunt următoarele:
• demonstrarea relatiei prin raţionamente sintetic-constructive; considerăm că a-
ceastă metodă este superioară calitativ faţă de oricare alta, ı̂nsă presupunem o anume
dexteritate din partea rezolvitorilor ı̂n utilizarea unor artificii de calcul şi raţionament;
• demonstrarea relaţiei prin inducţie matematică – este metoda aleasă de majori-
tatea rezolvitorilor, alegere motivată de gradul de accesibilitate al raţionamentului şi
calculelor ı̂n acest caz, comparativ cu aplicarea strategiilor de tip sintetic-constructiv.
Vom oferi cititorului spre comparare un procedeu mai puţin cunoscut, pe care
l-am denumit metoda identificării, exprimat prin următoarea
X
n
Propoziţie. a) Dacă a1 = b1 , bk −bk−1 = ak , ∀k ≥ 2, atunci ak = bn , ∀n ≥ 1.
k=1
bk Y
n
b) Dacă a1 = b1 şi = ak , ∀k ≥ 2, atunci ak = bn , ∀n ≥ 1.
bk−1
k=1
X
n X
n
Demonstraţie. Într-adevăr, ak = b1 + (bk − bk−1 ) = b1 + (b2 − b1 ) + (b3 −
k=1 k=2
b2 ) + ... + (bn − bn−1 ) = bn , la sumare efectuând aşa-numitele reduceri telescopice.
Faptul că metoda identificării nu apare printre căile frecvent aplicate pentru veri-
ficarea unor identităţi este explicabil prin aceea că celelalte metode au o arie de apli-
cabilitate incomparabil mai vastă. În ceea ce priveşte ı̂nsă eleganţa raţionamentului
şi simplitatea, următorul exemplu cu rezolvări comparative va convinge, probabil, de
avantajele metodei identificării.
X
n
Problema 1. Demonstraţi că (k 2 + 1)k! = n(n + 1)!, ∀n ∈ N∗ .
k=1
Soluţia 1 (prin raţionamentele sintetic-constructive). Dacă ak = (k 2 + 1)k! este
termenul general al sumei, căutăm a, b, c astfel ı̂ncât ak = a · k! + b(k + 1)! + c(k + 2)!;
după calcule similare celor din metoda coeficienţilor nedeterminaţi, găsim a = 2, b =
−3, c = 1, prin urmare ak = 2 · k! − 3 · (k + 1)! + (k + 2)! Sumând, obţinem că
X
n
ak = 2 · 1! + (2 − 3) · 2! + (1 − 3)(n + 1)! + (n + 2)! = n · (n + 1)! şi identitaea este
k=1
astfel demonstrată.
1 Profesor, Colegiul Tehnic ”I.C. Ştefănescu”, Iaşi

115
Soluţia 2 (prin inducţie matematică). Identitatea se verifică pentru n = 1
(obţinem 2 = 2, afirmaţie adevărată). Presupunem că egalitatea din enunţ are loc
pentru n (oarecare) şi demonstam că este adevărată şi pentru n + 1. Avem

X
n+1 X
n
(k 2 + 1)k! = (k 2 + 1)k! + ((n + 1)2 + 1)(n + 1)! =
k=1 k=1
= n(n + 1)! + (n2 + 2n + 2)(n + 1)! =
= (n + 1)!(n2 + 3n + 2) = (n + 1)!(n + 2)(n + 1) = (n + 1)(n + 2)!

Conform principiului inducţiei complete, identitatea are loc pentru orice n ∈ N∗ .


Soluţia 3 (prin metoda identificării ). Identitatea este de tipul celei din Propoziţia
1, ak = (k 2 + 1)k! şi bn = n(n + 1)!. Observăm că a1 = b1 (= 2), iar bk − bk−1 =
X
n
k(k + 1)! − (k − 1)k! = (k 2 + 1)k! = ak , ∀k ≥ 2. Rezultă că ak = bn = n(n + 1)!
k=1
Prezentăm ı̂ncă două probleme, ı̂n rezolvarea cărora vom folosi direct metoda
identificării:
Yn  ‹
2 n+3
Problema 2. Demonstraţi că 1− 2 = , ∀n ∈ N∗ .
k + 3k + 2 3(n + 1)
k=1
Y
n
2
Soluţie. Identitatea este de tipul ak = bn , ∀n ≥ 1, cu ak = 1 − şi
k 2 + 3k + 2
k=1
n+3 bk k+3 k+2 k 2 + 3k
bn = . Se observă că a1 = b1 , iar = : = =
3(n + 1) bk−1 3(k + 1) 3k (k + 1)(k + 2)
2
1− 2 = ak , ∀k ≥ 2. Conform Propoziţiei 1, identitatea este demonstrată.
k + 3k + 2
(n + 1)x
X 2 · sin
sin nx
n
Problema 3. Demonstraţi că sin kx = 2 , ∀n ∈ N∗ .
x
k=1 sin
2
Soluţie.

Cu notaţiile din Propoziţia 1, avem ‹
evident că a1 = b1 , iar bk − bk−1 =
1 kx (k + 1)x (k − 1)x kx 1 x kx
sin sin − sin sin = 2 · 2 · sin 2 cos 2 =
sin kx
sin x2 2 2 2 2 sin x2
kx kx
2 sin cos = sin kx = ak , ∀k ≥ 2, prin urmare are loc cerinţa problemei.
2 2
Propunem cititorului stabilirea următoarelor identităţi valabile pentru ∀n ∈ N∗ :
Xn
n(4n2 − 1) Xn
k2 n(n + 1)
4. (2k − 1)2 = , 5. = ,
3 (2k − 1)(2k + 1) 2(2n + 1)
k=1 k=1
Yn  ‹ Y
n
p−1 p(n + 1) sin 2n+1 x
6. 1+ = , 7. cos 2k x = .
k(k + p) n+p 2n · sin 2x
k=1 k=1

Bibliografie
1. L. Panaitopol - Inducţia matematică, Gil, Zalău, 2005.
2. Gh. Rizescu - Sume şi produse, Ed. Sigma, Bucureşti, 1999.

116
Bisectoarele exterioare
nu sunt ca bisectoarele interioare
Dumitru MIHALACHE şi Marian TETIVA1

Abstract. It is known the fact that an analogous result to the Steiner-Lehmus Theorem for the
exterior bisectors does not hold. The authors of this article present this result in a convenient way
for the reader who wants to see how a case study should be performed.
Keywords: angle bisector, exterior angle bisector, Steiner-Lehmus Theorem.
MSC 2000: 97C20.

Încă din clasa a şasea elevii ı̂nvaţă despre triunghiul isoscel. Mai precis, ei ı̂nvaţă
ı̂n primul rı̂nd că dacă un triunghi are două laturi congruente, atunci şi unghiurile
opuse acelor laturi sunt congruente. Mai află că, ı̂ntr-un asemenea triunghi, medianele
corespunzătoare laturilor congruente sunt congruente şi acelaşi lucru se ı̂ntâmplă cu
ı̂nălţimile corespunzătoare acestor laturi, şi tot congruente sunt şi bisectoarele unghiu-
rilor congruente. De prisos să mai spunem că privim toate aceste linii importante ale
triunghiului ca pe nişte segmente, toată lumea ı̂nţelege la ce ne referim. De asemenea,
este vorba de bisectoarele unghiurilor interioare ale triunghiului, deoarece despre cele
ale unghiurilor exterioare nu prea se mai vorbeşte ı̂n noile programe. Totuşi n-ar fi
greu de demonstrat şi congruenţa a două dintre ele dacă le definim corespunzător şi
triunghiul este isoscel, dar nu echilateral. Vom avea nevoie de această noţiune, aşa că
explicăm acum la ce ne referim. Să spunem că triunghiul ABC are laturile [AB] şi
[AC] de lungimi diferite şi să considerăm bisectoarele unghiurilor sale exterioare care
au vârful ı̂n A (acestea fiind unghiurile adiacente şi suplementare cu unghiul interior
care are vârful ı̂n A), deocamdată ca semidrepte (se ştie că sunt semidrepte opuse).
Una din ele intersectează dreapta BC; dacă AB > AC vedeţi uşor că intersecţia M
este astfel ı̂ncât C se află ı̂ntre B şi M . Vom numi atunci bisectoare exterioară din A a
triunghiului ABC segmentul [AM ]. E clar că, dacă AB = AC, bisectoarea exterioară
nu poate fi considerată ı̂n sensul acestei definiţii.
Nu mult după aceea, sau chiar deodată, elevii află că şi reciprocele acestor afirmaţii
sunt adevărate. Dar, găsindu-se la ı̂nceputul studiului geometriei, le va fi mult mai
greu să le justifice. Faptul că un triunghi cu două mediane congruente (sau cu două
ı̂nălţimi congruente) este isoscel nu prea poate fi demonstrat cu una, cu două, deoarece
necesită cazuri speciale de congruenţă. Iar despre a arăta că un triunghi cu două bi-
sectoare congruente este isoscel nici nu poate fi vorba atâta vreme cât el face obiectul
unei renumite teoreme care a pus la grea ı̂ncercare chiar mari matematicieni. Dar este
adevărat, ceea ce nu se mai poate afirma ı̂n cazul bisectoarelor exterioare (şi despre
asta vom vorbi mai departe). Profesorul C.L. Lehmus, ı̂n 1840, a cerut pentru prima
dată o soluţie sintetică a problemei, care a fost găsită, printre alţii, de marele geometru
Jacob Steiner (se pare că Lehmus ı̂nsuşi avea să găsească ulterior o demonstraţie);
1 Profesori, Colegiul Naţional ”Gheorghe Roşca Codreanu”, Bârlad

117
teorema a rămas cunoscută cu aceste nume: teorema Steiner-Lehmus. Mai multe
despre istoria ei şi demonstraţia ı̂n cadrul geometriei euclidiene clasice se găsesc, de ex-
emplu, pe internet la adresa http://forumgeom.fau.edu/FG2005volume5/FG200525.pdf
[2] (ca să dăm un exemplu la ı̂ndemâna cititorilor mai tineri; altminteri, orice carte
serioasă de geometrie euclidiană elementară cuprinde şi această teoremă).
Noi vom ı̂ncepe cu o demonstraţie prin calcul a acestui renumit rezultat, probabil
foarte cunoscută cititorilor. Vom considera un triunghi oarecare ABC ı̂n care vom
folosi notaţiile obişnuite: a, b, c sunt lungimile laturilor (opuse vârfurilor A, B, re-
spectiv C), la este lungimea bisectoarei obişnuite (interioare) a unghiului cu vârful ı̂n
A. Formula
a2 bc
la2 = bc −
(b + c)2
care furnizează lungimea acestei bisectoare este notorie, nu mai insistăm asupra ei.
Ca şi asupra următorului calcul, pe care-l recomandăm celor ce nu-l cunosc ı̂ncă.
Exerciţiul 1. Să se arate că, cu notaţiile de mai sus, avem
 
la2 − lb2 ab(a2 + b2 + c2 + ab + 2ac + 2bc)
= (b − a) 1 + .
c (a + c)2 (b + c)2

Evident, de aici se obţine imediat teorema Steiner-Lehmus şi chiar mai mult, se
vede că avem la < lb ⇔ a > b (deoarece expresia din paranteză este strict pozitivă).
Această proprietate este valabilă şi pentru mediane sau ı̂nălţimi, adică ma < mb ⇔
a > b şi ha < hb ⇔ a > b (dacă notăm ma şi ha lungimea medianei, respectiv
ı̂nălţimii din A), dar vom vedea că pentru bisectoarele exterioare nu este câtuşi de
puţin adevărată. Colateral, vă puteţi ocupa şi de
Exerciţiul 2. Demonstraţi că ma < mb ⇔ a > b şi ha < hb ⇔ a > b.
Acum să vorbim şi despre bisectoarele exterioare. Vom notă cu ea lungimea bi-
sectoarei exterioare din A (dacă b ̸= c). Formula

a2 bc
e2a = − bc
(b − c)2

este şi ea destul de cunoscută (dacă n-o ştiţi, o puteţi obţine fie din relaţia lui Stewart,
fie dintr-un calcul cu arii, folosind teorema bisectoarei exterioare pentru a exprima ı̂n
funcţie de a, b, c lungimile segmentelor determinate pe [BC] de bisectoare, respectiv
folosind expresia lui cos(A/2)). Cu ajutorul ei obţinem imediat, la fel ca mai sus
 
e2a − e2b ab(a2 + b2 + c2 + ab − 2ac − 2bc)
= (a − b) 1 + .
c (a − c)2 (b − c)2

Desigur, putem considera ea şi eb (şi calcula această expresie) numai ı̂ntr-un triunghi
neisoscel, deci mai departe presupunem că oricare două dintre a, b, c sunt distincte.
Exerciţiul 3. Continuaţi acest calcul pentru a obţine

e2a − e2b (b − a)(a + b − c)(c3 − (a + b)c2 + 3abc − ab(a + b))


= .
c (a − c)2 (b − c)2

118
E clar acum că nu avem cum să obţinem egalitatea laturilor din egalitatea bisec-
toarelor exterioare, deşi acesta este unul din cazuri: dacă presupunem ea = eb rezultă
că unul din factorii de la numărător trebuie să fie zero. Dintre aceşti factori doar de
a + b − c putem fi siguri că este nenul (pozitiv pentru a, b, c lungimile laturilor unui
triunghi). Rămâne totuşi al treilea factor, care ne permite să demonstrăm următoarea
Propoziţie. Pentru orice numere reale pozitive şi distincte a şi b există c pozitiv
şi diferit de a şi de b astfel ı̂ncât a, b, c pot fi laturile unui triunghi, iar ı̂n acest
triunghi ea = eb .
Demonstraţie. Să presupunem, de exemplu, că a < b. Pentru funcţia continuă
f definită prin
f (x) = x3 − (a + b)x2 + 3abx − ab(a + b), ∀x ∈ R,
avem f (b − a) = −2a3 < 0 şi f (b) = ab(b − a) > 0, deci există c ∈ (b − a, b) astfel
ı̂ncât f (c) = 0. Acest c este diferit şi de a, deoarece f (a) = ab(a − b) < 0; pentru că
b > a, b > c şi c > b − a implică b < a + c, triunghiul cu laturile a, b şi c există şi este
neisoscel. Exprimarea de mai sus a diferenţei e2a − e2b ne arată că, ı̂n acest triunghi,
ea = eb , ceea ce ı̂ncheie demonstraţia.
Se mai observă că semnul acestei diferenţe nu poate fi stabilit numai ı̂n funcţie de
semnul lui b − a, de aceea un analog al proprietăţii la < lb ⇒ a > b nu funcţionează
pentru bisectoarele exterioare. În plus, deşi tocmai am arătat că există triunghiuri
neisoscele cu două bisectoare exterioare congruente, cititorul va vedea uşor că nu se
poate ca toate bisectoarele exterioare să aibă lungimi egale.
Exerciţiul 4. Să se arate că ı̂ntr-un triunghi neisoscel, nu se poate ca toate cele
trei bisectoare exterioare să fie congruente.
În sfârşit, să ne mai punem o ı̂ntrebare firească: putem da un exemplu concret
de asemenea triunghi (neisoscel, dar care are două bisectoare exterioare congruente)?
Căci una e să spui: există un astfel de triunghi, şi alta e dacă afirmi răspicat că
triunghiul cu laturile de anumite lungimi (bine precizate) are respectiva proprietate.
Avem un asemenea exemplu! √ √
Exerciţiul 5. Să se arate că ı̂n triunghiul cu laturile a = 5 − 7, b = 5 + 7 şi
c = 6 avem ea = eb .
Mai mult, putem găsi toate triunghiurile cu această proprietate.
Propoziţie. Un triunghi neisoscel ABC are proprietatea ea = eb dacă şi numai
dacă există un număr p din intervalul (0, 1) şi un număr pozitiv c astfel ı̂ncât lungimile
laturilor sale să fie date de
c  È 
BC = 3p + 1 − (1 − p)(p2 + 3p + 4) ,
2(p + 1)
c  È 
AC = 3p + 1 + (1 − p)(p2 + 3p + 4) şi AB = c.
2(p + 1)
Demonstraţie. Desigur, ca să avem ea = eb ı̂ntr-un triunghi neisoscel trebuie ca
ultimul factor de la numărătorul expresiei e2a − e2b să fie zero, adică trebuie să aibă loc
egalitatea
c3 − (a + b)c2 + 3abc − ab(a + b) = 0.

119
Cu notaţiile x = a/c şi y = b/c aceasta se mai scrie (după ı̂mpărţirea cu c3 ):

1 − (x + y) + 3xy − xy(x + y) = 0 ⇔ 1 − s + 3p − sp = 0,

dacă punem s = x + y şi p = xy. Desigur, x şi y sunt soluţiile ecuaţiei de gradul al
doilea t2 − st + p = 0, iar dacă ştim pe x şi pe y, cunoaştem şi laturile triunghiului:
BC = cx, AC = cy şi BC = c. Relaţia de mai sus ne furnizează s = (3p + 1)/(p + 1)
şi un calcul rapid ne arată că discriminantul ecuaţiei de gradul al doilea este atunci
 ‹2
3p + 1 (1 − p)(4p2 + 3p + 1)
∆ = s2 − 4p = − 4p =
p+1 (p + 1)2

care va fi pozitiv dacă şi numai dacă p ∈ (0, 1) (de la bun ı̂nceput e clar că p trebuie
să fie pozitiv, iar din ∆ > 0 rezultă că p < 1). Dacă ∆ > 0, ecuaţia va avea
două soluţii pozitive şi distincte care conduc exact la formulele din enunţ pentru
lungimile laturilor. Interesant este că aceste valori ale lui x şi y produc ı̂ntotdeauna
(fără să fie nevoie de vreo condiţie suplimentară) lungimile laturilor unui triunghi
(ı̂n care, credem că e foarte clar, ea = eb ). Într-adevăr, ordinea lor se vede că este
a < c < b, ceea ce corespunde inegalităţilor x < 1 < y (care rezultă repede din
(x − 1)(y − 1) = p(p − 1)/(p + 1) < 0), caz ı̂n care mai trebuie să verificăm doar
b < a + c ⇔ b − a < c ⇔ y − x < 1. Dar
√ 4p3
y−x= ∆=1− <1
(p + 1)2

şi demonstraţia este ı̂ncheiată.


Noi am considerat p = 1/2 şi c = 6 pentru a obţine exemplul din Exerciţiul 5, dar
cititorul are acum la dispoziţie formulele generale care dau lungimile laturilor acestor
triunghiuri şi ı̂şi poate construi oricâte exemple doreşte. Întrebare (inevitabilă): găsim
triunghiuri cu toate lungimile laturilor numere ı̂ntregi ı̂n care ea = eb ? (Nu cunoaştem
răspunsul la acestă ı̂ntrebare.)
Probabil aţi observat că titlul, aşa glumeţ-aluziv cum este el spune exact ce
am vrut noi să evidenţiem ı̂n această notă: proprietăţile bisectoarelor interioare
(la = lb ⇒ a = b, respectiv la < lb ⇒ a > b care au loc şi pentru celelalte linii
importante obişnuite ale triunghiului) nu se păstrează deloc pentru bisectoarele ex-
terioare. Lăsăm ı̂n seama cititorului interesat (şi mai bun prieten cu geometria eu-
clidiană clasică) să explice pur geometric aceste fapte aparent stranii. Menţionăm că
existenţa triunghiurilor neisoscele cu două bisectoare exterioare congruente e cunos-
cută de mult: de exemplu, se vorbeşte despre ele ı̂n [1], la pagina 235 (unde sunt
numite triunghiuri pseudoisoscele).

Bibliografie
1. F. G.-M. – Exercices de géométrie, 1912.
2. K.R.S. Sastry – A Gergonne Analogue of the Steiner - Lehmus Theorem, Forum
Geometricorum, 5 (2005), 191-195.

120
Metoda falsei ipoteze - variante de utilizare
D.M. BĂTINEŢU-GIURGIU 1

În clasele primare elevii ı̂ntâlnesc mărimi care sunt dependente una de alta. De
exemplu, dacă ı̂n produsul F1 · F2 creşte (descreşte) unul din factori de un număr de
ori, acelaşi lucru se ı̂ntâmplă şi cu produsul, iar dacă unul din factori creşte (descreşte)
de un număr de ori şi produsul rămâne neschimbat, atunci celălalt factor descreşte
(creşte) de acelaşi număr de ori.
Problemele ı̂n care apar astfel de mărimi se rezolvă uneori cu metoda falsei ipoteze,
care comportă parcurgerea următoarelor etape: 1) se face o ipoteză arbitrară asupra
mărimilor de aflat, ce va fi, ı̂n general, ı̂n contradicţie cu datele problemei; 2) se
analizează sursele eşecului şi erorii comise şi se trag concluziile corespunzătoare; 3) se
modifică ipoteza (adică valorile date arbitrar mărimilor) pe baza concluziilor punctului
precedent, ı̂n scopul obţinerii soluţiei; 4) se verifică corectitudinea soluţiei găsite.
Deoarece ı̂n programa claselor primare nu mai este inclusă această metodă, dăm
un exempul de problemă pe care vom ilustra această cale de rezolvare.
Problemă. Un producător a vândut la piaţă 1200 kg mere de 2 lei/kg, 3 lei/kg şi
4 lei/kg, pe care a ı̂ncasat 3300 lei. Ştiind că producătorul a avut cantităţi egale de
mere de 2 lei/kg şi 3 lei/kg, să se afle câte kilograme de mere de 2 lei/kg, 3 lei/kg şi
4 lei/kg a vândut producătorul.
Prezentăm şase variante de rezolvare, utilizând de fiecare dată metoda falsei
ipoteze, cu o ipoteză sau cu două ipoteze arbitrare.
Varianta 1. Presupunem că toate merele sunt de 4 lei/kg. În această ipoteză
se ı̂ncasează 4 × 1200 = 4800 lei. Suma reală este depăşită cu 4800 − 3300 = 1500
lei. Această depăşire apare din faptul că am ı̂nlocuit x kilograme mere de 2 lei/kg şi
x kilograme mere de 3 lei/kg cu 2x kilograme mere de 4 lei/kg. Înlocuind 1 kg de
mere de 2 lei/kg şi 1 kg de mere de 3 lei/kg cu 2 kg mere de 4 lei/kg, suma reală
se măreşte cu 8 − (2 + 3) = 3 lei. Avem un număr de 1500 : 3 = 500 ı̂nlocuiri, ceea
ce ı̂nseamnă că avem 500 kg mere de 2 lei/kg, 500 kg de mere de 3 lei/kg şi 200 kg
mere de 4 lei/kg. Se verifică uşor că aceste cantităţi ı̂ndeplinesc condiţiile problemei:
500 × 2 + 500 × 2 + 200 × 4 = 3300 lei.
Varianta 2. Presupunem că avem numai mere de 2 lei/kg şi de 3 lei/kg. În această
ipoteză suma ı̂ncasată de producător este 600 × 2 + 600 × 3 = 3000 lei. Diferenţa
dintre suma reală şi cea din presupunere este 3300 − 3000 = 300 lei. Această diferenţă
1 Profesor, Colegiul Naţional ”Matei Basarab”, Bucureşti

121
provine din faptul că, de x ori am ı̂nlocuit câte 2 kg de mere de 4 lei/kg cu 1 kg de
mere de 2 lei/kg şi 1 kg de mere de 3 lei/kg, adică de câte ori se cuprinde (8 − 2 − 3) ı̂n
300, de unde aflăm că x = 100. Înseamnă că producătorul a vândut 2 × 100 = 200 kg
mere de 4 lei/kg, 500 kg de mere de 2 lei/kg şi 500 kg de mere de 3 lei/kg. Verificarea
se face la fel.
Varianta 3. Presupunem că cele trei categorii de mere sunt ı̂n cantităţi egale. În
această ipoteză suma ı̂ncasată este 400 × 2 + 400 × 3 + 400 × 4 = 3600 lei. Diferenţa
dintre suma presupusă şi cea reală este 3600 − 3300 = 300 lei. Evident, cantităţile nu
pot fi egale. Trebuie să ı̂nlocuim de x ori câte 2 kg mere de 4 lei/kg cu 1 kg de mere
de 2 lei/kg şi 1 kg de mere de 3 lei/kg. Obţinem x = 300 : (8 − 2 − 3) = 100, ceea ce
ı̂nseamnă că trebuie să micşorăm cantitatea de mere de 4 lei/kg cu 2 × 100 = 200 kg.
Am aflat că producătorul a vândut 200 kg mere de 4 lei/kg, 500 kg mere de 2 lei/kg
şi 500 kg mere de 3 lei/kg.
Varianta 4. Observăm ı̂ntâi că producătorul a vândut cantităţi de mere expri-
mate prin numere naturale pare. Să presupunem că producătorul a vândut 500 kg
mere de 4 lei/kg. În acest caz suma ı̂ncasată este 350 × 2 + 350 × 3 + 500 × 4 = 3750
lei. Diferenţa dintre suma din presupunere şi cea reală este 3750 − 3300 = 450 lei.
Evident, cantitatea de mere de 4 lei/kg trebuie micşorată. Înlocuim de x ori câte 2 kg
de mere de 4 lei/kg cu 1 kg de mere de 2 lei/kg şi 1 kg mere de 3 lei/kg, diminuând
suma presupusă cu 8−(2+3) = 3 lei. Cantitatea de mere de 4 lei/kg trebuie micşorată
cu 450 : 3 × 2 = 300 kg, ceea ce ı̂nseamnă că producătorul a vândut 200 kg mere de
4 lei/kg, 500 kg mere de 2 lei/kg şi 500 kg mere de 3 lei/kg.
Varianta 5. Presupunem că producătorul a vândut 300 kg mere de 2 lei/kg,
300 kg mere de 3 lei/kg şi 600 kg mere de 4 lei/kg. În această ipoteză producătorul
ı̂ncasează 300 × 2 + 300 × 3 + 600 × 4 = 3900 lei. Diferenţa dintre suma presupusă şi
cea reală este de 3900 − 3300 = 600. Trebuie să ı̂nlocuim de x ori câte 2 kg de mere
de 4 lei/kg cu 1 kg mere de 2 lei/kg şi 1 kg mere de 3 lei/kg, iar suma ı̂ncasată ı̂n
această ipoteză se micşorează cu 8 − (2 + 3) = 3 lei. Cantitatea de mere de 4 lei/kg
trebuie micşorată cu 600 : 3 × 2 = 400 kg, ceea ce ı̂nseamnă că producătorul a vândut
200 kg mere de 4 lei/kg, 500 kg mere de 2 lei/kg şi 500 kg mere de 3 lei/kg.
Varianta 6. Prima ipoteză: producătorul a vândut 320 kg mere de 2 lei/kg, 320
kg mere de 3 lei/kg şi 560 kg mere de 4 lei/kg. Suma ı̂ncasată ı̂n prima ipoteză este
320 × 2 + 320 × 3 + 560 × 4 = 3840 lei. Ipoteza a doua: producătorul a vândut 340 kg
mere de 2 lei/kg, 340 kg mere de 3 lei/kg şi 520 kg mere de 4 lei/kg. În această ipoteză
suma ı̂ncasată este 340 × 2 + 340 × 3 + 520 × 4 = 3780 lei. Se observă că la o creştere
de 40 kg mere de 4 lei/kg, prima sumă presupusă se micşorează cu 3840 − 3780 = 60
lei. Prima sumă presupusă trebuie să scadă cu 3840 − 3300 = 540 lei. Cantitatea de
560 kg mere de 4 lei/kg trebuie să scadă cu 540 : 60 × 40 = 360 kg, ceea ce ı̂nseamnă
că producătorul a vândut 200 kg mere de 4 lei/kg, 500 kg mere de 2 lei/kg şi 500 kg
mere de 3 lei/kg. Verificarea este imediată.

122
Liceul Teoretic ”Garabet Ibrăileanu”1
La 110 ani de la ı̂nfiinţare

La prima vedere Liceul Teoretic ”Garabet Ibrăileanu”, cu arhitectura sa uşor


confundabilă cu aceea a şcolilor ridicate ı̂n ”epoca de aur”, pare o instituţie şcolară
de dată recentă. Nimic mai fals. Liceul acesta are mai bine de 100 de ani de la
ı̂nfiinţare şi istoria sa reflectă convulsiile prin care a trecut societatea românească,
drama intelectualităţii şi chiar ”drama psihologiei” pentru a prelua titlul unei cărţi
de Vasile Pavelcu, fost profesor la această şcoală.
Legea ı̂nvăţământului secundar promulgată de Spiru Haret cerea candidaţilor la
profesorat să fi trecut un examen de capacitate. Dar pentru a se putea prezenta la
acest examen candidaţii trebuiau să frecventeze un Seminar Pedagogic, să parcurgă
modulul pedagogic, am spune noi astăzi, şi implicit să facă un număr de ore de
practică pedagogică. De aceea s-a creat Seminarul Pedagogic Universitar cu
propria şcoală de aplicaţie. Aceasta se ı̂ntâmpla la 1 noiembrie 1899, localul şcolii
fiind casa Aslan de la vale de Universitatea nouă din Copou. Şcoala de aplicaţie avea
un dublu rol: pentru studenţi sau alsolvenţi era locul unde ı̂şi desfăşurau practica
pedagogică, pentru profesorii universitari era locul ı̂n care ı̂şi puneau ı̂n practică
metodele moderne de predare, un laborator de pedagogie experimentală. Se observă
din capul locului coerenţa care exista ı̂ntre activitatea didactică, pregatirea pedagogică
a viitorilor profesori şi activitatea de cercetare a universitarilor. Nu este de mirare că
de aici au plecat majoritatea cadrelor didactice din liceele Moldovei. Mulţi absolvenţi
1 N. R. Pentru informaţii mai ample recomandăm cartea Magda Negrea – Liceul de Aplicaţie

al Semninarului Pedagogic Universitar – Iaşi cu un studiu introductiv de prof.dr. Adrian


Neculau.

123
ai şcolii de aplicaţie au revenit la SPU ca practicanţi, iar unii chiar ca profesori. De
asemenea unii profesori ai SPU au pus bazele unor şcoli asemănătoare la Cernăuţi
(Constantin Narly)) şi Cluj sau a Universităţii din Chişinău (Constantin Fedeleş).
Liceul de Aplicaţie al SPU a funcţionat cu succes până la reforma ı̂nvă-
ţământului din 1948. O istorie de jumătate de veac marcată de personalitatea celor
doi directori, ambii pedagogi de frunte ai şcolii ieşene: Ioan Găvănescul şi Ştefan
Bârsănescu. Ioan Găvănescul, pedagog cu studii ı̂n Germania, a pus bazele unui
regulament organic al instituţiei. ı̂nainte de deschiderea şcolii a convocat profesorii ı̂n
şase conferinţe ı̂n care a fixat normele şi principiile generale didactice şi de ordine şi
disciplină. Ideea călăuzitoare a fost ”educaţia ca scop, instrucţia ca mijloc”. Şcoala
trebuie să dezvolte simţul practic şi spontaneitatea mentală. De aceea ı̂n predare se va
folosi metoda activă, intuiţia şi corelaţia ı̂ntre obiectele de studiu. Educaţia nu trebuie
să fie teoretică şi intelectuală ci ”practică şi voluntară”, elevul, asemeni lui Robin-
son Crusoe, să ştie ı̂n orice ı̂mprejurare ”a găsi mijloacele practice pentru a ı̂nvinge
orice greutate”. Pe lângă activitatea teoretică, elevul era antrenat ı̂n multe activităţi
practice: cartonaj, modelaj, tâmplărie. La SPU din Iaşi s-a introdus pentru prima
dată lucrul manual la o şcoală de cultură generală. Ioan Găvănescul a fost director
până ı̂n 1932, când a ieşit la pensie, dar a continuat să ţină conferinţe, era membru
al partidului conservator, a avut convingeri de dreapta. La venirea comuniştilor la
putere, deşi se apropia de 90 de ani a fost arestat şi moare ı̂n ı̂nchisoare ı̂n 1951.
Între 1932 şi 1938 director este Paul Nicorescu, istoric, fiind ajutat de Vasile
Pavelcu, responsabil cu practica pedagogică şi de E. Briul care răspundea de ac-
tivitatea diriginţilor şi disciplina şcolară. Sub directoratul lui Paul Nicorescu se con-
struieşte aripa de vest a şcolii, care există şi azi la adresa din Toma Cozma, nr. 2.
Ştefan Bârsănescu, care ı̂i urmează ca director, continuă să construiască ı̂n ciuda
vremurilor de război. Sub coordonarea lui Ştefan Bârsănescu se construieşte latura
din centru şi se toarnă fundaţia pentru aripa est care ar fi ı̂nchis acest amsamblu.
ı̂n această activitate, Ştefan Bârsănescu este secundat de directorul administrativ
I. Rick şi de directorul adjunct D. Gafiţanu.
Odată cu venirea lui Ştefan Bârsănescu la conducerea Seminarului Pedagogic Uni-
versitar, ı̂ncepe o perioadă de dezvoltare distinctă a acestei instituţii. Cu studii la
Universitatea din Iaşi şi două doctorate ı̂n Germania, Ştefan Bârsănescu are toate
atributele unui mare pedagog. Spre deosebire de Ioan Găvănescul, care punea accent
pe formarea caracterului şi pregătirea practică, Ştefan Bârsănescu pune accent pe
lecţie, pe transmiterea de valori. Totul gravitează ı̂n jurul ideii de cultură. Pentru
Ştefan Bârsănescu un ”om cultivat” este acela care se conduce ı̂n activitatea sa după
”valorile veşnice” ı̂mbogăţindu-şi şi ordonându-şi năzuinţele după adevăr (ştiinţă),
bine(morală) şi frumos(artă). În privinţa studenţilor practicanţi, Ştefan Bârsănescu
ı̂i roagă pe profesori ”să-i câştige ca pe nişte prieteni, să nu fie descurajaţi, să se simtă
ca ı̂ntr-o mare familie. Practicanţii asistau la un număr impresionant de ore până
erau lăsaţi să facă lecţii de probă. Toate lecţiile erau discutate. Fiecare practicant ı̂şi
ı̂ntocmea un dosar consistent cu observaţii, rapoarte, planuri de lecţii, un adevărat
ghid cu care pornea ı̂n activitatea la catedră.
Dar ı̂n martie 1944 tot Iaşul pleacă ı̂n refugiu. Tot ce s-a putut demonta şi ambala
a fost pus ı̂n 14 vagoane şi transportat ı̂n Banat. Iaşul est bombardat, trei bombe

124
căzând şi pe Liceul de aplicaţie. În martie 1945, Liceul revine din refugiu şi pe 9 aprilie
se ı̂ncep cursurile. Zece vagoane din cele 14 sunt recuperate, se fac reparaţii, ı̂ncât
la 1 oct. 1945 clădirea adăposteşte atât Liceul de aplicaţie cât şi clase de la Liceul
Internat şi de la ”Oltea Doamna”. Se fac demersuri pentru a continua construcţia
aripei de est la care de mulţi ani era turnată fundaţia. Totul părea ı̂ncă posibil,
dar reforma ı̂nvăţământului din 1948 face totul imposibil. Seminarul Pedagogic se
desfiinţează, Liceul de aplicaţie ı̂şi schimbă această menire şi devine Liceul Clasic.
Se poate spune că ı̂ncepe drama psihologiei şi pedagogiei. Disciplinile acestea aproape
că dispar din ı̂nvăţământul universitar, practica pedagogică se reduce ca număr de
ore şi ca importanţă, dar, ı̂n mod paradoxal, se face cu precădere tot la fostul Liceu de
aplicaţie. Ştefan Bârsănescu este ı̂ndepărtat atât de la liceu cât şi de la Universitate.
Abia dacă ocupă un post de bibliotecar. Este reprimit la catedră ı̂n 1957. Avea
nu numai cultură, ci şi har ı̂şi aminteşte studentul de atunci Adrian Neculau, acum
profesor de psihologia grupului social şi fervent admirator al S.P.U. de altădată.
Liceul de aplicaţie fusese doar de băieţi, Liceul Clasic este mixt şi are 12 clase. Din
1953 se numeşte Şcoala Medie Clasică, apoi din anul următor se numeste Şcoala
Medie nr. 2 de fete. Toate aceste schimbări, adevărate convulsii pentru ı̂nvăţământ
ı̂n general şi pentru liceul de care ne ocupăm ı̂n special, precum şi politizarea ı̂n exces
a lecţiilor au drept scop ruperea de istoria şcolii, uitarea bunei tradiţii. Din anul 1956,
şcolile medii devin mixte şi se numără din nou, liceul nostru semicentenar devenind
Şcoala Medie nr. 4. Pentru ca uitarea să se aştearnă şi mai bine pe istoria şcolii, ı̂n
loc să se termine de construit aripa est, se ı̂ncepe construirea unui nou local pe strada
Oastei nr. 1. Şcoala Medie nr. 4, care ı̂ntre timp a căpătat şi numele de Liceul
”Garabet Ibrăileanu”, se mută ı̂n noul local ı̂n 1962. În localul vechi continuă să
funcţioneze internatul pentru mai mulţi ani, apoi aici se instalează dispensarul pentru
elevi şi studenţi. Vis-à-vis, ı̂n vechea şcoală de ı̂nvăţătoare, funcţionează Facultatea
de psihologie şi pedagogie, adică fostul SPU. E doar ironia sorţii sau contratimpul
tragic care ne marchează pe noi, românii? Vechea clădire din Toma Cozma nr. 2
aşteaptă şi acum să fie terminată şi să servească drept şcoală ı̂ntre marile şcoli ale
Iaşului: Universitate, Facultatea de psihologie, Liceul Internat şi nu departe Liceul
Naţional şi fosta ”Oltea Doamna”.
În noul local din dealul Copoului, liceul are profilul real - umanist şi din 1967
devine şcoală pilot pentru ı̂nvăţarea limbilor străine, ı̂n speţă limba franceză, din
ciclul primar. În 1977 ı̂şi schimbă iar numele şi profilul, devenind Liceul Industrial
Hidrotehnic. Dar pe fiecare an de studiu se păstrează o clasă de reală şi una de
filologie. În 1990 revine la profilul real - umanist. Pe baza tradiţiei de studii clasice,
la fiecare an de studiu a existat până ı̂n 2004 o clasă de limbi clasice (limba latină şi
greaca veche). Actualmente liceul este teoretic şi are clase de informatică, ştiinţe ale
naturii, filologie.
Povestea Liceului ”Garabet Ibrăileanu” e povestea noastră a tuturor, mereu
”sub vremuri”. Şi totuşi spiritul Liceului de aplicaţie, sămânţa bună semănată de
acei dascăli de demult, cu dragoste de ţară şi de profesie nu a pierit şi aducerea ı̂n
memorie a unui secol de şcoală ne ı̂ncurajează şi ne obligă.

Prof. Magda NEGREA

125
LIDIA COHAL
(1930-2009)
Evenimente a căror ı̂mpletire este mai presus de dorinţe şi raţiuni ne obligă la
oarece zăbavă. Pe lângă numeroasele, nobilele şi acaparantele obligaţii ale profesorilor
de matematică, ni se relevă una mai rar evidenţiată, dar pe care o găsim plină de
semnificaţii: activitatea ı̂n familia substanţial extinsă a matematicienilor.
Lidia s-a născut la 30 octombrie 1930 ı̂n satul Ciutuleşti, judeţul Bălţi, de partea
mai zbuciumată a Prutului. Absolvă şcoala primară ı̂n satul natal şi continuă primele
patru clase de liceu la Liceul ”Regina Maria” din oraşul Bălţi. Nu considerăm nece-
sar să explicăm fracturile sociale care au determinat ca ultimele patru clase să şi le
desfăşoare la Liceul ”Ecaterina Teodoroiu” din Târgu Jiu.
Lidia devine studentă la Facultatea de matematică şi fizică la Universitatea ”Al.
I. Cuza” Iaşi. Ca recunoaştere a meritelor de la ı̂nceputul anului IV este numită
preparator. La absolvirea facultăţii, ı̂n anul 1952, a fost ı̂ncadrată ca asistent univer-
sitar la Catedra de geometrie, bine reputată inclusiv la nivel internaţional. Împreună
cu colegele şi prietenele Alice Corduneanu şi Ligia Papuc (deja căsătorite cu distinşi
matematicieni) se lansează ı̂n activităţi ştiinţifice şi didactice, devenind şi o harnică
secretară a Filialei Iaşi a SSM.
În septembrie 1959 i se desface contractul de muncă: originea socială, fiică de
preot, era privită drept ”necorespunzătoare”. Conducerea SSM se dovedeşte mai
puţin influenţabilă şi decide retribuirea fructuoasei sale activităţi de secretară de
filială. Este locul să spunem că Societatea preluase aproape integral sarcinile de
perfecţionare a profesorilor de matematică prin şcoli de vară; filiala ieşeană activă
ı̂n această directie extrem de susţinut şi materialele respective au fost incorporate ı̂n
patru volume de ”Probleme actuale de matematică”.
În septembrie 1960 este ı̂ncadrată şi ı̂n ı̂nvăţământul liceal din Iaşi, păstrându-şi
şi atribuţiile de ”secretară de filială model”: extrem de precisă găsea mereu zâmbetul
prietenesc stimulator.
A funcţionat la liceele ieşene ”Naţional”, ”Emil Racoviţă” şi ”Mihai Eminescu”.
Din perioada ı̂n care era ı̂ncadrată ca asistent universitar, Lidia ı̂ncepuse docu-
mentarea şi lucrul la o carte destinată ”modernizării predării geometriei”. Coordona
eforturile profesorul Adolf Haimovici, preşedinte al filialei Iaşi şi mai colaborau pri-
etenele mai sus numite Alice şi Ligia. Cartea Elemente de geometrie a planului a
apărut abia ı̂n 1968 la Editura Didactică şi Pedagogică.
Lidia s-a pensionat ı̂n septembrie 1987.
De multe ori se omite amintirea unor realizări ı̂n pretenţioasa activitate de soţie;
nu vom face aici această greşeală. Prin optimismul ei molipsitor şi prin tonusul ei
excelent, Lidia a potenţat neı̂ntrerupt activitatea soţului ei, distinsul profesor ieşean
Traian Cohal.
Unii oameni nu mor de tot.
Prof.dr. Dan BRÂNZEI
Preşedintele Filialei Iaşi a SSM

126
Concursul de matematică “Al. Myller”
Ediţia a VII-a, Iaşi, 28 martie 2009

Clasa a VII-a
1. Determinaţi n ∈ N∗ cu proprietatea că n!+3·2n = 6n−2 , unde n! = 1·2·3 . . . n.
Artur Bălăucă
2. Fie triunghiul ABC şi punctul D situat pe latura [BC]. Arătaţi că
AB · DC + AC · BD ≥ AD · BC.
***
3. Fie p un număr natural impar. Se ştie că oricare divizor al lui p are ultima
cifră diferită de 3 şi 7. Să se arate că numărul 5p + 1 nu este pătrat perfect.
Mircea Fianu
4. Fie triunghiul echilateral ABC şi punctul D situat pe latura (AC). Bisectoarea
unghiului ∠ABD intersectează paralela prin A la dreapta BC ı̂n punctul E. Arătaţi
că AE + DC = BD.
Cristian Lazăr

Clasa a VIII-a
1. Fie un tetraedru regulat cu muchia de lungime 3. Pe suprafaţa acestuia se
consideră 37 de puncte. Arătaţi că printre aceste puncte există două astfel ı̂ncât
distanţa dintre ele este cel mult egală cu 1.
***
2. Determinaţi perechile de numere (a, b) ∈ Z × Z care verifică egalitatea
2(a + b)2 + 3(a + b) + ab + 4 = 0.
Petru Răducanu
3. Fie a ≥ b ≥ c ≥ d ≥ 0 astfel ı̂ncât a2 + b2 + c2 + d2 = 1. Să se arate că
a + b ≥ 1 ≥ c + d.
Gheorghe Iurea
4. Numim piramidă Myller o piramidă SABCD cu baza ABCD, care are SA =
SB = SC = SD, ∠ASB ≡ ∠ASD şi ∠BSC ≡ ∠DSC, iar lungimile SA, AB, BC,
CD, DA, AC, BD sunt numere naturale nenule. Aflaţi piramida Myller de volum
minim.
Cristian Lazăr

Clasa a IX-a
1. Determinaţi n ∈ N∗ pentru care există o mulţime A ⊂ R cu n elemente, având
proprietatea a(b3 + 6) ≤ b(a3 + 6), ∀a, b ∈ A.
Gheorghe Iurea

127
2. Care este numărul minim de elemente care trebuie eliminate din mulţimea
{1, 2, 3, . . . , 100} astfel ı̂ncât ı̂n mulţimea rămasă să nu existe trei elemente x, y, z
pentru care xy = z?
***
3. Fie triunghiul ABC şi punctele M ∈ (AB), N ∈ (AC) astfel ı̂ncât BM +CN =
M N + BC. Notăm ρ raza cercului ı̂nscris ı̂n triunghiul AM N . Arătaţi că
√ È √ È
ρ( bc + (p − b)(p − c)) ≤ r( bc − (p − b)(p − c)).

Dan Brânzei
4. Numărul ı̂ntreg m are proprietatea că, pentru orice număr natural n, există
an ∈ Z astfel ı̂ncât |nm − 80an + 1| < 20. Arătaţi că 80 divide m.
Dinu Şerbănescu

Clasa a X-a
1. Determinaţi valorile lui n ∈ N∗ pentru care 41 are un multiplu de forma
a00
| {z. . . 0b} unde a, b sunt cifre zecimale nenule.
n+2 cifre
Mihai Bălună
2. Fie ABC un triunghi, k ∈ R\{1} şi punctele M ∈ BC, N ∈ CA, P ∈ AB
MB NC PA
astfel ı̂ncât = = = k. Se ştie că AM = BN = CP. Demonstraţi că
MC NA PB
triunghiul ABC este echilateral.
I. V. Maftei
3. Pentru x, y ∈ R definim f (x; y) = distanţa de la |x − y| la cel mai apropiat
ı̂ntreg, iar pentru o mulţime finită M ⊂ [0, 1] definim
X
s(M ) = f (x, y).
x,y∈M,x<y

Determinaţi valoarea maximă pentru s(M ), când M parcurge familia mulţimilor cu


4 elemente.
Dinu Şerbănescu
4. Fie a un număr real. Arătaţi că şirul de termen general xn = (−1)[na] , n ≥ 0,
este periodic dacă şi numai dacă a este raţional.
Gheorghe Iurea

Clasa a XI-a
n
1. Fie şirul (xn )n≥1 de numere reale definit prin x1 = 1 şi xn+1 = xn − ,
n+1
oricare ar fi n ≥ 1. Să se arate că şirul este divergent.
Paul Georgescu, Gabriel Popa
2. a) Să se determine două matrice A, B ∈ M2 (C) astfel ı̂ncât A2 + B 2 = I2 şi
matricea AB − BA este inversabilă.

128
b) Fie n un număr natural impar şi matricele A, B ∈ Mn (C) astfel ı̂ncât A2 +B 2 =
In . Să se arate că det(AB − BA) = 0.
Andrei Ciupan
3. Fie P, Q : R → R două funcţii polinomiale neconstante cu proprietatea că
există şi este finită lim ([P (x)] − [Q(x)]). Să se arate că există n ∈ Z astfel ı̂ncât
x→∞
P (x) − Q(x) = n, oricare ar fi x ∈ R.
Gheorghe Iurea
4. Fie A ∈ Mn (R) o matrice simetrică cu elementele de pe diagonala principală
egale cu 1 şi cu suma modulelor elementelor de pe fiecare linie mai mică sau egală cu
2. Să se arate că detA ≤ 1.
Cosmin Pohoaţă

Clasa a XII-a
1. Fie f, g două funcţii polinomiale reale, de acelaşi grad, ambele având coeficien-
tul dominant egal cu 1. Dacă g nu are rădăcini reale pozitive, calculaţi
Z 1  ‹
f (nx)
lim n x − 1 dx.
n→∞ 0 g(nx)
Radu Gologan
2. Fie A un inel. a) Arătaţi că dacă x ∈ A este nilpotent (există k ∈ N∗ cu
k
x = 0), atunci 1 + x este inversabil.
b) Dacă A este finit, numărul elementelor inversabile este un număr prim iar
elementele neinversabile sunt nilpotente şi numărul elementelor neinversabile este mai
mare sau egal cu numărul elementelor inversabile, arătaţi că A are 4 elemente.
***
3. Să se determine funcţiile continue f : R → R care verifică egalitatea f (arctg x) =
(1 + x2 )f (x), pentru orice x ∈ R.
Al. Gabriel Mı̂rşanu
4. Fie p, p > 2, un număr prim şi f un polinom cu coeficienţi ı̂ntregi, de grad p−1,
cu proprietatea că pentru orice a, b numere ı̂ntregi pentru care p divide f (a) − f (b),
rezultă că p divide a − b. Arătaţi că f are coeficientul dominant divizibil cu p.
Marian Andronache

Răspunsul la ”recreaţia” de la pag. ... este:

Numărul maxim de operaţii cu care se poate scrie 2009 este 4017 :


2009 = 10 : 10 + 10 : 10 + · · · + 10 : 10 (2009 termeni).
Numărul minim de operaţii este 209:
2009 = (10 + 10 + · · · + 10) + (10 : 10 + 10 : 10 + · · · + 10 : 10)
( 200 termeni ı̂n prima paranteză şi 9 ı̂n cea de-a doua).

129
Concursul de matematică ”Florica T. Câmpan”
Etapa judeţeană, 21 februarie 2009

Clasa a IV-a (Subiect elaborat de Dumitru Pârâială şi Cătălin Budeanu)


1. Fiica, tatăl şi bunica au ı̂mpreună 90 de ani. Peste doi ani, tatăl va avea de opt
ori vârsta fiicei, iar bunica de două ori vârsta actuală a tatălui. Aflaţi vârsta fiecăruia
ı̂n prezent.
2. Un blocnotes are 100 de pagini, numerotate de la 1 la 100. Se rup din acesta,
la ı̂ntamplare, 30 de foi, apoi se face suma numerelor ce marchează paginile rămase.
Este posibil ca această sumă să fie egală cu 800? Justificaţi!
3. O cantitate de 120 kg de prune a fost ı̂mpărţită ı̂n mod egal ı̂n mai multe
lăzi. În vederea transportului, pentru a se evita pierderile, s-a transferat câte un sfert
din cantitatea de prune din fiecare ladă, obţinâdu-se astfel o nouă ladă cu tot atâtea
kilograme de prune câte au rămas ı̂n fiecare din lăzile iniţiale.
a) Câte lăzi au fost la ı̂nceput?Justificaţi!
b) Câte kilograme de prune erau iniţial ı̂n fiecare ladă?
c) Câţi lei s-au ı̂ncasat din vânzarea prunelor din lada nou formată, dacă un
kilogram de prune de calitatea ı̂ntâi costă 4 lei, un kilogram de prune de calitatea a
doua costă 3 lei, iar la fiecare 2 kilograme de prune de calitatatea ı̂ntâi există câte
3kg de prune de calitatea a doua?

Clasa a V-a (Subiect elaborat de Gheorghe Iurea şi Andrei Nedelcu)


1. Notăm cu a1 , a2 , ....., a2009 numerele 1, 2, . . . , 2009, scrise ı̂n altă ordine.
Arătaţi că numărul N = (1 + a1 ) · (2 + a2 ) · . . . · (2009 + a2009 ) este par. Este posibil
ca N să nu fie divizibil cu 4 ?
2. Într-un pătrat 7 × 7 sunt scrise 49 numere naturale diferite. De pe fiecare
linie se consideră cel mai mic număr, iar cel mai mare dintre aceste şapte numere se
notează cu x. De pe fiecare coloană se consideră cel mai mare număr, iar cel mai mic
dintre aceste şapte numere se notează cu y. Justificaţi că x ≤ y.
3. a) Arătaţi că numărul 3 · 102009 − 8 nu este pătrat perfect.
b) Arătaţi că numărul 221506 are cel puţin 2009 cifre.

Clasa a VI-a (Subiect elaborat de Claudiu Ştefan Popa şi Ciprian Baghiu)
a c
1. a) Fie proporţia = , cu toţi termenii numere naturale, astfel ı̂ncât d este
b d
divizibil şi cu b, şi cu c, iar b şi c sunt prime ı̂ntre ele. Demonstraţi că bcd este pătrat
perfect.
b) Dacă un extrem al unei proporţii cu toţi termenii numere naturale nenule este
divizibil cu fiecare din mezii ei, arătaţi că suma tuturor termenilor este divizibilă cu
celălalt extrem.
Claudiu Ştefan Popa

130
2. Din cauza crizei economice, preţul unei mărfi se reduce săptămânal cu 50%,
timp de n săptămâni, unde n ∈ N, n ≥ 2.
a) Considerăm că preţul iniţial al mărfii este de 32 de lei. Ştiind că, atunci când
este cazul, se procedează la rotunjirea preţului după o anumită reducere, pentru ca
el să poată fi plătit cu unităţile monetare existente şi că ı̂n a n-a săptămână preţul
afişat este acelaşi cu cel din săptămâna n – 1, să se afle n.
b) Presupunând acum că preţurile succesiv obţinute sunt exprimate fără rotunjire
şi că preţul iniţial al mărfii este a lei, arataţi că suma ı̂ntre preţurile mărfii din primele
n – 1 săptămâni şi dublul preţului mărfii din a n-a săptămână este a lei.
Claudiu Ştefan Popa
3. Considerăm triunghiul isoscel ABC, cu AB = AC şi m(]BAC) < 900 . Punctele
M şi N sunt ı̂n aşa fel ı̂ncât AB separă C şi M, AC separă B şi N, m(]M AB) =
m(]N AC) = 900 , iar AM = AN = AB. Arătaţi că:
a) M C = N B;
b) M B ̸= N B;
c) dacă, ı̂n plus, triunghiul MNB este isoscel, calculaţi m(]BAC).

Clasa a VII-a (Subiect elaborat de Vasile Nechita şi Ionel Nechifor )



1. a) Fie a, b, c numere reale strict pozitive astfel ı̂ncât 5a + 3c = 4b şi 2b= 15ac.
Să se arate că numerele a, b, c sunt direct
√ proporţionale cu numerele 6, 15 şi 10.
b) Determinaţi x ∈ N pentru care x2 + 9x + 13 ∈ N.
2. a) Pe o masă sunt 20 monede. Radu şi Bogdan joacă următorul joc: ia fiecare,
pe rând, de pe masă, câte 1, 2 sau 3 monede; câştigătorul este cel care ia ultima
monedă. Cine va câştiga jocul? (justificaţi!) Dar dacă numărul monedelor de pe
masă este 21?
b) Fiind dat un paralelogram ABCD şi o dreaptă d, care taie dreptele AB, BC,
M A N B P C QD
CD, DA ı̂n punctele M, N, P, respectiv Q, demonstraţi că · · · = 1.
M B N C P D QA
3. Andrei, Bogdan, Costel şi Dan cumpără trei torturi de ı̂ngheţată de formă
pătrată, care au fiecare aceeaşi grosime şi compoziţie. Ştim că perimetrele torturilor
1 1 1
sunt invers proporţionale cu numerele , , , iar suma laturilor celor trei
36 160 164
torturi este 90cm. Explicaţi cum vor ı̂mpărţi copiii cele trei torturi, pentru a lua
fiecare porţii egale (fără cântărire).

Clasa a VIII-a (Subiect elaborat de Gabriel Popa şi Cristian Lazăr )


1. a) Determinaţi numerele ı̂ntregi a şi b cu proprietatea că
a2 b + ab2 + a + b + 1 = 0.
Recreaţii Matematice, 1/2000
b) Fie n un număr natural dat. Determinaţi numerele reale strict pozitive x, y, z
pentru care xn = yz, y n = zx şi z n = xy.
Neculai Hârţan , Recreaţii Matematice, 2/2003

2. Despre un număr natural n vom spune că√are proprietatea (P ) dacă 24n + 1 ∈
Q şi vom spune că are proprietatea (Q) dacă 24n + k ∈ R\Q, ∀k ∈ {2, 3, 4, ..., 99} .

131
a) Daţi exemplu de un număr natural care are proprietatea (P ), dar nu are pro-
prietatea (Q).
b) Daţi exemplu de un număr natural care are proprietatea (Q), dar nu are pro-
prietatea (P ).
c) Daţi exemplu de un număr natural care nu are nici proprietatea (P ), nici pro-
prietatea (Q).
d) Determinaţi (cu justificare) cel mai mic număr natural care are atât proprietatea
(P ), cât şi proprietatea (Q).
e) Găsiţi toate numerele naturale care au atât proprietatea (P ), cât şi proprietatea
(Q).
Cristian Lazăr
3. Un zmeu din carton, care are forma unui triunghi ABC, lasă pe pământ o
umbră având forma unui triunghi A′ B ′ C ′ , care este asemenea cu triunghiul ABC.
a) Dacă zmeul se află sub razele soarelui la amiază, demonstraţi că ∆ABC ≡
∆A′ B ′ C ′ .
b) Dacă zmeul se află sub becul unui stâlp de iluminat, mai rezultă ı̂n mod necesar
că ∆ABC ≡ ∆A′ B ′ C ′ ?
Constantin Cocea, Gabriel Popa

Etapa interjudeţeană, 21 martie 2009

Clasa a IV-a (Subiect elaborat de Dan Brânzei şi Cătălin Budeanu)


1. Aflaţi suma resturilor ı̂mpărţirii a 2009 numere naturale consecutive la 15,
ştiind că ultimul se ı̂mparte exact la 15.
2. Suma a două numere naturale este 26. Împărţind primul număr la al doilea şi
apoi pe al doilea la primul se obţine, de fiecare dată, aceeaşi sumă dintre cât şi rest,
aceasta fiind cu 8 mai mică decât unul dintre numere. Aflaţi numerele.
3. Fie numărul N = 510152025 . . . 725730735 . . . 20020052010.
a) Câte cifre are numărul N ?
b) Care este a 1000-a cifră a lui N ?
4. (facultativ ) Centrul unei pieţe are forma unui pătrat de latură 7 şi este ocupat
de o statuie cu un soclu pătrat de latură 1. Se poate acoperi suprafaţa rămasă cu
patru dreptunghiuri având dimensiunile 3 şi 4?

Clasa a V-a (Subiect elaborat de Gheorghe Iurea şi Andrei Nedelcu)


1. La un turneu de fotbal ı̂n sală participă patru echipe. Se acordă 2 puncte
pentru victorie, 1 punct pentru egal şi 0 puncte la ı̂nfrângere. Fiecare echipă joacă cu
fiecare câte un singur meci. În clasamentul final nu sunt două echipe la egalitate de
puncte. Care este numărul minim de puncte pe care ı̂l poate avea echipa câştigătoare?
Realizaţi o distribuţie a rezultatului meciurilor care să corespundă situaţiei de mai
sus.

132
2. Considerăm mulţimea A = {2a · 3b · 5c |a, b, c ∈ N}. Arătaţi că printre oricare 9
elemente ale mulţimii A există cel puţin două a căror produs este pătrat perfect.
3. Un pătrat cu latura 5 se ı̂mparte ı̂n pătrate cu latura 1, care se numerotează cu
numere de la 1 la 25. Se calculează sumele de pe fiecare linie şi de pe fiecare coloană.
Există o numerotare astfel ı̂ncât exact o sumă să fie număr par?
4. (facultativ ) Numărul 31 organizează o petrecere ı̂n ı̂mpărăţia Numerelor Nat-
urale. Dacă x ∈ N, atunci 9x – 2 şi 9x + 2 sunt părinţii lui x, iar 9x + 4 este bunicul
lui x. Se ştie că, dacă bunicul unui număr se află la petrecere, atunci este invitat
şi nepotul său. De asemenea, dacă un număr este invitat la petrecere, atunci sunt
invitaţi şi părinţii săi. Arătaţi că numărul 2009 este invitat la petrecere.

Clasa a VI-a (Subiect elaborat de Ionel Nechifor şi Ciprian Baghiu)


m + 2267 n + 5388
1. Să se arate că oricare ar fi m, n ∈ N, atunci 178
< .
m+3 n + 7291
2. Pe un teren de forma unui triunghi echilateral cu latura de 8m se plantează 5
brazi.
a) Demonstraţi că oricum am planta brazii, vor exista cel puţin doi având ı̂ntre
ei o distanţă nu mai mare de 4 metri.
b) În câte moduri se pot planta cei 5 brazi, astfel ı̂ncât să nu existe doi la distanţă
strict mai mică de 4 metri?
3. Fie a un număr natural nenul. Arătaţi că există b ∈ N∗ astfel ı̂ncât 9b − 3b să
fie divizibil cu a.
Ciprian Baghiu
4.(facultativ ) Determinaţi toate valorile numărului natural n, pentru care fracţia
2n2 + 5
este reductibilă.
3n2 + 5n + 10

Clasa a VII-a (Subiect elaborat de Vasile Nechita şi Claudiu Ştefan Popa)
1. Luându-se după harta care a aparţinut piratului Supernegru, pentru a găsi co-
moara fabuloasă (adică o supercomoară) ascunsă de acesta cândva, undeva ı̂n Caraibe,
patru supercăutători trebuie să caute ı̂n interiorului patrulaterului NESV situat pe
un teren plat şi obţinut astfel: din acelaşi punct O, fiecare dintre cei 4 merge, ı̂n linie
dreaptă; primul face b paşi spre nord până ı̂n N, al doilea c paşi spre est până ı̂n E, al
treilea a paşi spre sud până ı̂n S şi ultimul a paşi spre vest până ı̂n V (a,b,c numere
naturale). Ştiind că ı̂ntre N şi E sunt exact a paşi şi că lungimea pasului fiecăruia
dintre cei 4 este aceeaşi şi constantă, arătaţi că suprafaţa patrulaterului NESV are
aria exprimată printr-un număr natural (unitatea de măsură este pătratul cu latura
de 1 pas). Este posibil ca aria patrulaterului să fie egală cu aria unui pătrat de latură
6 paşi ?
Claudiu Ştefan Popa
2. a) Fie a, b, cnumere reale pozitive. Demonstraţi că a este media geometrică a
1 1 1
lui b şi c dacă şi numai dacă = + .
a a+b a+c

133
b) Fie O intersecţia diagonalelor patrulaterului convex ABCD. Demonstraţi că
1 1 1
AB|| CD dacă şi numai dacă = + .
AAOD AACD AABD
Claudiu Ştefan Popa
3. Rezolvaţi ı̂n mulţimea numerelor ı̂ntregi ecuaţia x2 (y − 1) + y 2 (x − 1) = 1.
Gabriel Mı̂rşanu

Clasa a VIII-a (Subiect elaborat de Gabriel Popa şi Cristian Lazăr )


1. Se dă mulţimea M = {1, 2, 3, 4} . Se formează toate sumele cu termeni distincţi
din M, luând ı̂n seamă inclusiv sumele cu un singur termen. Două sume se consideră
a fi distincte dacă diferă fie prin cel puţin un termen, fie prin ordinea termenilor (de
exemplu, sumele 4, 1 + 3, 3 + 1, 1 + 2 + 3 sunt distincte).
a) Câte sume diferite se pot forma?
b) Arătaţi că orice număr cuprins ı̂ntre 1 şi 10 poate fi obţinut ca rezultat ı̂n urma
calculării unei astfel de sume.
c) Două numere cuprinse ı̂ntre 1 şi 10 vor fi numite ı̂nrudite dacă se obţin de acelaşi
număr de ori ca rezultat al unor sume ca cele din enunţ. Determinaţi perechile de
numere ı̂nrudite.
Gabriel Popa
2. Fie VABCD şi SABCD două √piramide patrulatere regulate, având ca bază
comună pătratul ABCD de latură 12 2cm, vârfurile V şi S de o parte şi de alta a
planului (ABCD) şi ı̂n care ı̂nălţimile şi muchiile laterale se exprimă (ı̂n centimetri)
prin numere ı̂ntregi.
a) Demonstraţi că punctele V, A, S şi C sunt coplanare.
b) Aflaţi valorile posibile ale ı̂nălţimii VO a piramidei VABCD.
c) Arătaţi că patrulaterul VASC este circumscriptibil.
d) Dacă patrulaterul VASC este inscriptibil, determinaţi lungimea segmentului
VS.
Cristian Lazăr
3. Trei greieri ”sar capra”: un greiere, aflat ı̂n punctul A, sare peste un alt greiere,
aflat ı̂n B, şi ajunge ı̂n C, unde C este simetricul lui A faţă de B. Apoi, acelaşi greiere
sau un altul sare peste un partener de joacă şi tot aşa. Dacă iniţial cei trei greieri
se aflau ı̂n trei dintre vârfurile unui pătrat, se poate ca la un moment dat, ı̂n cursul
jocului, unul dintre greieri să ajungă ı̂n cel de-al patrulea vârf al acelui pătrat?
Mircea Ganga

Vizitaţi noua pagina web a revistei:

http://www.recreatiimatematice.ro

134
Soluţiile problemelor propuse ı̂n nr. 2/2008
Clasele primare
P.154. Dorina are 15 baloane roşii şi albastre. Câte baloane roşii poate avea, dacă
numărul acestora este mai mic decât numărul baloanelor albastre şi este cel puţin egal
cu 3?
(Clasa I ) Inst. Maria Racu, Iaşi
Soluţie. Numărul baloanelor roşii poate fi 3, 4, 5, 6 sau 7.
P.155. Dintr-o carte lipsesc câteva pagini, de la numărul 71 la numărul 94. Câte
foi lipsesc din această carte?
(Clasa I ) Ionela Bărăgan, elevă, Iaşi
Soluţie. Prima foaie care lipseşte are paginile 71 şi 72, iar ultima are paginile 93
şi 94. În total lipsesc 12 foi.
P.156. La concursul ”Desene pe asfalt”, elevii claselor I-IV de la Şcoala ”Otilia
Cazimir” au acumulat 50 de puncte şi cel puţin 2 premii din fiecare categorie. Care
este cel mai mare număr de premii pe care-l pot obţine elevii, dacă pentru premiul
I s-au acordat 10 puncte, pentru premiul al II-lea s-au acordat 6 puncte, iar pentru
premiul al III-lea s-au acordat 4 puncte?
(Clasa a II-a) Înv. Elena Porfir, Iaşi
Soluţie. Numărul maxim de premii se obţine ı̂n cazul: 2 premii I, 3 premii II şi
3 premii III, ı̂n total 8 premii.
P.157. Prin golirea unui singur vas, ales dintre cele de mai jos, putem face ca
restul vaselor să aibă cantităţi egale de lichid. Care vas trebuie golit?

(Clasa a II-a) Amalia Cantemir, elevă, Iaşi


Soluţie. Distribuim lichidul din vasul de 15 litri ı̂n primele 6 vase astfel: 5 l, 4 l,
3 l, 2 l, 1 l, 0 l. În acest mod, ı̂n fiecare vas vom avea 14 litri.
P.158. Aflaţi trei numere naturale ştiind că, adunându-le două câte două, obţinem
100, 89, respectiv 141.
(Clasa a III-a) Inst. Maria Racu, Iaşi
Soluţie. Obţinem că dublul sumei celor trei numere este 330, deci suma celor trei
numere este 330 : 2 = 165. Astfel, primul număr este 165 − 100 = 65, al doilea este
165 − 89 = 76, iar al treilea este 165 − 141 = 24.
P.159. Se consideră numerele: a = 1 + 4 + 7 + 10 + · · · + 2008, b = 2 + 5 + 8+
+ · · · + 2009, c = 3 + 6 + 9 + · · · + 2010. Arătaţi că suma a + b + c se ı̂mparte exact
la 3, fără să calculaţi această sumă.
(Clasa a III-a) Iuliana Moldovan, elevă, Iaşi

135
Soluţie. Putem scrie a + b + c = (1 + 2 + 3) + (4 + 5 + 6) + (7 + 8 + 9) + . . . +
(2008 + 2009 + 2010). În fiecare paranteză avem suma a trei numere consecutive, care
se ı̂mparte exact la 3, deci a + b + c se ı̂mparte exact la 3.
P.160. Numărul a este de forma xy0, iar numărul b este de forma uv. Să se afle
a şi b ştiind că a + b = 22 zeci.
(Clasa a III-a) Dragoş Toma, elev, Iaşi
Soluţie. Condiţia din problemă se scrie xy0 + uv = 220, din care se deduce
v = 0, iar xy + u = 22. Pentru perechea (a, b) avem posibilităţile: (210, 10); (200, 20);
(190, 30) . . . (130, 90).
P.161. Fie a şi b două numere naturale astfel ı̂ncât diferenţa lor este de 5 ori
mai mică decât suma lor. Să se arate că numărul cel mai mare se ı̂mparte exact la
3, iar cel mai mic se ı̂mparte exact la 2.
(Clasa a IV-a) Diana Tănăsoaie, elevă, Iaşi
Soluţie. Fie a şi b cele două numere. Din a + b = 5(a − b) obţinem 2a = 3b, deci
b trebuie să fie par. Înlocuind b = 2c, rezultă că a = 3c, prin urmare a se ı̂mparte
exact la 3.
P.162. Maria are 9 săculeţi cu monede. Cel puţin un săculeţ cântăreşte un
kilogram. În orice grupare de 5 săculeţi, cel puţin 3 săculeţi au aceeaşi masă, iar ı̂n
orice grupare de 6 săculeţi, cel mult 5 săculeţi au aceeaşi masă. Care este cel mai
mare număr de săculeţi de 1 kg pe care ı̂l poate avea Maria?
(Clasa a IV-a) Petru Asaftei, Iaşi
Soluţie. Nu putem avea mai mult de 5 săculeţi de 1 kg, deoarece am găsi o
grupare de 6 săculeţi de aceeaşi masă. Numărul maxim posibil de săculeţi de 1 kg
este 5 şi poate fi atins, de exemplu, dacă Maria are 5 săculeţi de 1 kg şi ı̂ncă 4 de o
altă masă, aceeaşi, caz ı̂n care ar fi ı̂ndeplinită şi prima condiţie (conform principiului
cutiei).
P.163. Jumătatea produsului a două numere naturale consecutive, ı̂mpărţită cu
3, nu poate da niciodată restul 2.
Recreaţii Ştiinţifice, Anul I (1883)- nr. 4, p.119
Soluţie. Dacă unul dintre cele două numere consecutive se ı̂mparte exact la
3, restul ı̂mpărţirii din enunţ este 0. În caz contrar, produsul celor două numere
consecutive, care este par, va da restul 2 la ı̂mpărţirea prin 3, iar jumătatea se va da
la ı̂mpărţirea prin 3 restul 1.
Clasa a V-a
V.95. Două numere naturale se scriu ı̂n baza 10 folosind doar cifrele 1, 4, 6 şi 9.
Poate fi unul dintre numere de 2008 ori mai mare decât celălalt?
Cătălin Budeanu, Iaşi
Soluţie. Dacă numerele A şi B se scriu doar cu ajutorul cifrelor 1, 4, 6 şi 9, atunci
U (2008A) ∈ {2, 8}, prin urmare 2008A ̸= B. Răspunsul la ı̂ntrebarea din enunţ este
negativ.
V.96. Determinaţi k, n ∈ N∗ astfel ı̂ncât (1 + 1 · n)+(2 + 2 · n)+· · ·+(k + k · n) =
3 · 4 · 5 · 6.
Petru Asaftei, Iaşi

136
Soluţie. Deoarece (1 + 1 · n) + (2 + 2 · n) + . . . + (k + k · n) = (1 + 2 + . . . +
k(k + 1)
k) + n(1 + 2 + . . . + k) = (1 + 2 + . . . + k)(n + 1) şi 1 + 2 + . . . + k = ,
2
ecuaţia dată este echivalentă cu k(k + 1)(n + 1) = 2 · 3 · 4 · 5 · 6. Atunci k şi k + 1
sunt divizori ai numărului 2 · 3 · 4 · 5 · 6, iar k(k + 1) ≤ 2 · 3 · 4 · 5 · 6. Găsim soluţiile
(k, n) ∈ {(1, 359); (2, 119); (3, 59); (4, 35); (5, 23); (8, 9); (9, 7); (15, 2)}.
V.97. Arătaţi că numărul N = 17n + 21n + 25n , n ∈ N, nu poate fi pătrat perfect.
Virginia Grigorescu, Craiova
Soluţie. Deoarece (M a+1)n = M a+1, deducem că 17n = M 4+1, 21n = M 4+1,
25n = M 4 + 1, deci N = M 4 + 3, prin urmare N nu poate fi pătrat perfect.
V.98. Fie n ∈ N∗ . Să se demonstreze că numărul N = 5050 . . . 505 (2n + 1 cifre)
se scrie ca sumă a 4n + 2 pătrate perfecte distincte.
Veronica Plăeşu şi Dan Plăeşu , Iaşi
Soluţie. Observăm că N = 5 · 102n + 5 · 102n−2 + . . . + 5 · 102 + 5. Cum 5 = 22 + 12 ,
iar 5 · 102k = 500 · 102k−2 = (162 + 122 + 82 + 62 ) · (10k−1 )2 = (16 · 10k−1 )2 + (12 ·
10k−1 )2 + (8 · 10k−1 )2 + (6 · 10k−1 )2 , cerinţa problemei este demonstrată.
V.99. Se consideră numărul N = 1 + 11 + 101 + 1001 + · · · + 100
| {z. . . 01}.
n cifre
a) Pentru n ∈ N, n ≥ 5, arătaţi că 5 | N ⇔ 5 | n.
b) Precizaţi care dintre propoziţiile ”3 | n ⇒ 3 | N ” şi ”3 | N ⇒ 3 | n” este
adevărată pentru orice n ≥ 3.
Temistocle Bı̂rsan, Iaşi
Soluţie. a) Cum N = 1+10+100+. . .+100 .
| {z }. . 0 + 1 + 1 + . . . + 1 = |11 .{z
. . 11} +(n−
| {z }
ncifre n−1 termeni ncifre
1) = 11
| .{z
. . 10} +n. Cum 5| 11
| .{z
. . 10}, rezultă cerinţa a).
ncifre n
b) Niciuna dintre implicaţii nu este adevărată. Într-adevăr, pentru n = 3 avem că
.
N = 1 + 11 + 101 = 113/.. 3. Apoi, pentru n = 5 avem că N = 1 + 11 + 101 + 1001 +
. .
10001 = 11115..3, ı̂n timp ce 5/.. 3.
V.100. Determinaţi numerele naturale nenule a şi b pentru care există n ∈ N
a 3n + 2
astfel ı̂ncât = şi 3a + 2b < 100.
b 7n + 5
Gheorghe Iurea, Iaşi
Soluţie. Dacă d = (3n + 2, 7n + 5), atunci d|3n + 2 şi d|7n + 5, de unde d|3(7n +
3n + 2
5) − 7(3n + 2), cu alte cuvinte d = 1, deci fracţia este ireductibilă. Deoarece
7n + 5
a 3n + 2
= , deducem că a = k(3n + 2) şi b = k(7n + 5), unde k ∈ N∗ , iar inegalitatea
b 7n + 5
3a + 2b < 100 devine k(23n + 16) < 100. Pentru n = 0 obţinem k ∈ {1, 2, 3, 4, 5, 6},
a = 2k şi b = 5k; pentru n = 1 avem k ∈ {1, 2}, a = 5k şi b = 12k. Dacă n = 2,
atunci k = 1, a = 8, b = 19, iar pentru n = 3, k = 1, a = 11, b = 26. Pentru n ≥ 4,
problema nu admite soluţii.

137
an + b
V.101. Considerăm fracţia , unde n, a, b, c, d ∈ N∗ , astfel ı̂ncât b şi d au
cn + d
parităţi diferite, iar a şi c au aceeaşi paritate. Arătaţi că, dacă ad − bc = 2k , k ∈ N∗ ,
atunci fracţia este ireductibilă.
Cosmin Manea şi Dragoş Petrică, Piteşti
Soluţie. Presupunem prin absurd că fracţia este reductibilă. Fie p un divizor
comun al numerelor an+b şi cn+d, p ∈ N∗ , p ≥ 2; rezultă că p|(a·n+b) şi p|(c·n+d),
de unde p|ad − bc, deci p|2k şi atunci p este par. Totodată, p|n(a + c) + b + d şi cum
n(a + c) + b + d este număr impar, rezultă că p este impar, contradicţie.
Clasa a VI-a
a1 a2
VI.95. Determinaţi numerele naturale nenule a1 , a2 ,. . . , a2008 , ştiind că =
1·2
a2 a3 a2007 a2088
= ··· = , iar a1 + a2008 = 2009.
2·3 2007 · 2008
Gheorghe Iurea, Iaşi
a1 a3
Soluţie. Considerând rapoartele două câte două, obţinem că = = ... =
1 3
a2007 a2 a4 a2008
, iar = = ... = . Astfel, relaţia a1 + a2008 = 2009 devine a1 +
2007 2 4 2008∗
1004a2 = 2009 şi cum a1 , a2 ∈ N , atunci a1 = 1, a2 = 2. Deducem că ai = i,
∀i = 1, 2008.
VI.96. Determinaţi p ∈ N pentru care numerele p, p + 12, p + 22, p + 52, p + 72,
p + 102 şi p + 132 sunt prime.
Damian Marinescu, Târgovişte
Soluţie. Pentru p = 7, numerele sunt 7, 19, 29, 59, 79, 109, 139 şi sunt toate prime.
Dacă p ∈ {2, 3, 5}, obţinem numerele compuse 2 + 12, 3 + 12, respectiv 5 + 22. Dacă
p ≥ 11, considerând p = M7 + r, cu r ∈ {0, 1, . . . , 6}, găsim ı̂n fiecare caz câte un
număr compus printre cele date. În concluzie, singura valoare convenabilă este p = 7.
VI.97. a) Dacă a, b, c, d, e, f ∈ N∗ sunt astfel ı̂ncât (a, b) = (c, d) = (e, f ) =
a c e
(b, d) = 1, iar t = − + ∈ N, arătaţi că f = bd.
b d f
4 1 7
b) Determinaţi a, b ∈ N pentru care − + ∈ N.
2a + 1 2b 6
Cosmin Manea şi Dragoş Petrică, Piteşti
e e
Soluţie. a) Deoarece t · bd = ad − bc + bd · ∈ N, rezultă că bd · ∈ N şi, cum
f f
c
(e, f ) = 1, deducem că f |bd. Pe de altă parte, din t · bf = af − bf · + ed ∈ N şi
d
a c a
t · df = df · − cf + de ∈ N, rezultă că bf · şi df · sunt numere naturale. Folosind
b d b
faptul că (b, a) = (d, b) = (d, c) = 1, obţinem că d|f şi b|f şi, cum (b, d) = 1, atunci
bd|f . Din f |bd şi bd|f , rezultă că f = bd.
b) Suntem ı̂n condiţiile punctului precedent; deducem că 6 = (2a + 1) · 2b , de unde
a = b = 1. Pentru aceste valori, vom avea t = 2 ∈ N.
VI.98. Determinaţi cel mai mic număr natural n cu proprietatea că numărul
zerourilor ı̂n care se termină numărul (n + 10)! este cu 2008 mai mare decât numărul
zerourilor ı̂n care se termină n! (unde n! = 1 · 2 · 3 · · · n).
Cătălin Budeanu, Iaşi

138
Soluţie. Ipoteza problemei revine la faptul că numărul (n + 1)(n + 2) . . . (n + 10)
se termină ı̂n 2008 zerouri, ceea ce se ı̂ntâmplă atunci când produsul (n + 1)(n +
2) . . . (n + 10) se divide cu 52008 şi nu se divide cu 52009 . Printre factorii produsului
precedent, există exact doi care sunt divizibili cu 5, iar dintre aceştia unul nu mai este
divizibil cu nicio altă putere a lui 5, deci celălalt se va divide cu 52007 . Cum dorim n
minim, atunci n + 10 = 52007 , prin urmare n = 52007 − 10.
VI.99. Un patrulater convex are două laturi opuse congruente şi diagonalele
congruente. Arătaţi că patrulaterul este trapez isoscel sau dreptunghi.
Ioan Săcăleanu, Hârlău
Soluţie. Presupunem că (AB) ≡ (CD) şi (AC) ≡ (BD). Din congruenţa tri-
Õ ≡ DCB,
unghiurilor ABC şi DCB, rezultă că ABC Õ iar din congruenţa triunghiurilor
ABD şi DCA, rezultă că BADÕ ≡ CDA.Õ Cum suma unghiurilor unui patrulater este
Õ Õ
360 , deducem că m(ABC) + m(BAD) = 180◦ , deci AD∥BC.

Dacă AB∥CD, atunci ABCD este un paralelogram cu diagonalele congruente,


deci este un dreptunghi. Dacă AB ∦ CD, atunc ABCD este un trapez cu diagonalele
congruente, adică un trapez isoscel.
VI.100. Fie △ABC cu m(A) b ≥ 90◦ . Să se arate că m(B) Ò = 2m(C) Ò dacă şi
numai dacă există M ∈ [BC] astfel ı̂ncât AB = AM = M C.
Petru Asaftei, Iaşi
Soluţie. Presupunem că m(B) Ò = 2m(C) Ò şi fie M intersecţia mediatoarei laturii
[AC] cu BC; cum m(A) b ≥ 90◦ , vom avea că M ∈ [BC]. A
Evident atunci că △M AC este isoscel cu M A = M C şi
Ö
cum AM B este unghi exterior, deducem că m(AM Ö B) =
Ò Ò
2m(C) = m(B). Astfel, △ABM va fi isoscel cu AB =
AM, ceea ce ı̂ncheie justficarea afirmaţiei directe. B M C
Reciproc, dacă există M ∈ [BC] cu AB = AM = M C, din triunghiurile isoscele
ABM şi M AC obţinem că m(B) Ò = m(AM Ö Ò
B) = 2m(C).
VI.101. Fie ABC un triunghi dreptunghic cu m(A) b = 90◦ şi CD bisectoarea
Ò
unghiului C, D ∈ (AB). Perpendiculara din D pe bisectoarea unghiului B Ò inter-
sectează ipotenuza BC ı̂n E. Dacă P este punctul de intersecţie a bisectoarelor unghi-
urilor triunghiului ABC, iar M este punctul de intersecţie dintre EP şi AC, arătaţi
Ö
că M PA ≡ P ÕBE.
Nela Ciceu, Bacău şi Titu Zvonaru, Comăneşti
Soluţie. Din enunţ rezultă că triunghiul BDE este isoscel şi că BP este medi-
atoarea segmentului DE. Totodată, m(CDE) Õ = m(CDB) Õ − C
Õ = 180 − m(C)◦ 1 Ò − m(B) 1
Ò − [180 − m(B)]
◦ Ò = 45 . ◦
m(EDB)
2 2
Prin urmare, triunghiul DP E este dreptunghic isoscel. Astfel,
CP ⊥ M E şi cum CP este bisectoarea unghiului C, Ò rezultă
că triunghiul CM E este isoscel. Deducem că m(M Ö P A) =
P E
◦ Ö ◦ ◦ ◦ Ö
180 − m(P M A) − 45 = 135 − 180 + m(CM E) = −45◦ + M
1 Ò = 1 [90◦ − m(C)]
Ò = 1 m(B) Ò = m(P
Õ
[180◦ − m(C)] BE). A D B
2 2 2

139
Clasa a VII-a
A M B
VII.95. Fie ABCD pătrat, M un punct oarecare pe (AB),
iar N ∈ (BC) este astfel ı̂ncât M N ⊥ M D. Arătaţi că AM ·
N
AB + CN · CB = DM . 2

Ovidiu Pop, Satu Mare


Gh. Szöllösy, Sighetul Marmaţiei
Soluţie. Fie a latura pătratului x = AM şi y = CN ; atunci D C
DM 2 +M N 2 = DN 2 ⇔ a2 +x2 +(a−x)2 +(a−y)2 = a2 +y 2 ⇔
2a2 − 2ax + 2x2 − 2ay = 0 ⇔ a2 + x2 = ax + ay ⇔ DM 2 = AM · AB + CN · CB.
VII.96. Fie [AD] mediană ı̂n △ABC, M mijlocul lui [AD], F
{E} = BM ∩ AC, iar punctul F pe dreapta AB este astfel ı̂ncât
CF ∥ AD. Demonstraţi că punctele D, E şi F sunt coliniare.
Mirela Marin, Iaşi A
Soluţie. Cum ADCF este trapez, M este mijlocul bazei mici,
. P
E
iar B este punctul de intersecţie a laturilor neparalele, ı̂nseamnă
.
.
că {P } = BM ∩ CF este mijlocul lui [CF ]. Cu reciproca teoremei
M
liniei mijlocii se arată că A este mijlocul lui [BF ], prin urmare
CA şi BP sunt mediane ı̂n △BCF , iar E va fi centrul de greutate
B D C
al acestui triunghi. Coliniaritatea dorită rezultă observând că a treia mediană a
triunghiului este F D.
VII.97. Fie C1 (O1 , r1 ) şi C2 (O2 , r2 ), r1 < r2 , două cercuri tangente exterior.
Considerăm punctele A′ ∈ C1 , B ′ ∈ C2 , de aceeaşi parte a dreptei O1 O2 , astfel ı̂ncât
A′ O1 ∥ B ′ O2 . Dacă AB este tangentă comună exterioară a cercurilor (A ∈ C1 ,
B ∈ C2 ), demonstraţi că dreptele AB, A′ B ′ şi O1 O2 sunt concurente.
Romanţa Ghiţă şi Ioan Ghiţă, Blaj
Soluţie. Notăm {P } = AB ∩ O1 O2 , {P ′ } = A′ B ′ ∩ O1 O2 . Din asemănările
P O1 P ′ O1 r1
△P O1 A ∼ △P O2 B şi △P ′ O1 A′ ∼ △P ′ O2 B ′ obţinem că = ′ = < 1.
P O2 P O2 r2
B
A
O2
O1

B A P
. .
P
Astfel, punctele P şi P ′ se află de aceeaşi parte cu O1 pe dreapta O1 O2 şi ı̂mpart
segmentul [O1 O2 ] ı̂n acelaşi raport, prin urmare P = P ′ şi de aici rezultă concurenţa
dorită.
VII.98. Să se determine numerele naturale nenule a şi b, ştiind că sunt direct
proporţionale cu b − 6 şi a şi invers proporţionale cu a + 12 şi b.
Constantin Apostol, Rm. Sărat
a b
Soluţie. Din datele problemei obţinem că = şi a(a + 12) = b2 . Din prima
b−6 a
ecuaţie rezultă că a2 − b2 = −6b, iar din a doua a2 − b2 = −12a, deci −6b = −12a.

140
Atunci b = 2a şi, folosind prima ecuaţie, găsim că a = 4 şi apoi b = 8.
VII.99. Fie a, b ∈ Z şi numerele A = 119a5 + 5b3 − 4a şi B = 119b5 + 5a3 − 4b.
Să se arate că A se divide cu 120 dacă şi numai dacă B se divide cu 120.
Dan Nedeianu, Drobeta-Tr. Severin
Soluţie. Avem că A + B = 120(a5 + b5 ) − (a5 − 5a3 + 4a) − (b5 − 5b3 + 4b). Cum
.
a5 − 5a2 + 4a = (a − 2)(a − 1)a(a + 1)(a + 2) se divide la 120, rezultă că A + B ..120
şi concluzia de impune.
VII.100. Arătaţi că 2a2 + 15b2 + 7c2 ≥ 10ab − 6ac + 20bc, ∀a, b, c ∈ R.
Alexandru Negrescu, student, Iaşi
Soluţie. Inegalitatea se scrie: 4a2 − 4a(5b − 3c) + 30b2 + 14c2 − 40bc ≥ 0 ⇔
(2a − 5b + 3c)2 + 30b2 + 14c2 − 40bc − (5b − 3c)2 ≥ 0 ⇔ (2a − 5b + 3c)2 + 5(b − c)2 ≥ 0,
evident adevărat. Egalitate avem pentru a = b = c.
VII.101. Pentru n ∈ N∗ , notăm cu d (n) numărul divizorilor primi ai lui n.
a) Determinaţi cardinalul mulţimii A = {n ∈ N∗ | n ≤ 208, d (n) = 3}.
b) Aflaţi cel mai mic şi cel mai mare element al mulţimii

B = {k ∈ N | ∃n ∈ N∗ , n ≤ 2008, a.ı̂. d (n) = k} .


Gabriel Popa, Iaşi
Soluţie. a) Observăm că n ∈ A dacă şi numai dacă n = pα 1 · p2 · p3 , cu
1 α2 α3

p1 < p2 < p3 numere prime, iar α1 , α2 , α3 ∈ N . Cum 5 · 7 · 11 = 385 > 208, ı̂nseamnă
că p1 ∈ {2, 3}, prin urmare
(p1 , p2 , p3 ) ∈ {(2, 3, 5); (2, 3, 7); (2, 3, 11); (2, 3, 13); (2, 3, 17); (2, 3, 19); (2, 3, 23);
(2, 3, 29); (2, 3, 31); (2, 5, 7); (2, 5, 11); (2, 5, 13); (2, 5, 17); (2, 5, 19);
(2, 7, 11); (2, 7, 13); (3, 5, 7); (3, 5, 11); (3, 5, 13)}.
Pentru tripletele subliniate, ı̂n produsul n = pα 1 · p2 · p3 vom avea obligatoriu
1 α2 α3

α1 = α2 = α3 = 1; obţinem astfel 13 elemente ale lui A.


Dacă (p1 , p2 , p3 ) = (2, 3, 5), ı̂n n = 2α1 · 3α2 · 5α3 putem avea α3 = 1, (α1 , α2 ) ∈
{(1, 1); (2, 1); (3, 1); (1, 2); (2, 2)} sau α3 = 2, α1 = α2 = 1; găsim ı̂ncă 6 elemente din
A. Dacă (p1 , p2 , p3 ) = (2, 3, 7), ı̂n n = 2α1 · 3α2 · 7α3 este obligatoriu ca α3 = 1, iar
(α1 , α2 ) ∈ {(1, 1); (2, 1); (3, 1); (1, 2)}; obţinem 4 noi elemente ale lui A.
Dacă (p1 , p2 , p3 ) = (2, 3, 11), atunci α3 = 1, (α1 , α2 ) ∈ {(1, 1); (2, 1); (1, 2)}, deci
ı̂ncă 3 elemente. Dacă (p1 , p2 , p3 ) ∈ {(2, 3, 13); (2, 3, 17); (2, 5, 7)}, vom avea ı̂n fiecare
caz (α1 , α2 , α3 ) ∈ {(1, 1, 1); (2, 1, 1)}, adică ı̂ncă 3 × 2 = 6 elemente.
În total, |A| = 13 + 6 + 4 + 3 + 6 = 32.
b) Evident că min B = 0, atins pentru n = 1.
Arătăm că max B = 4: dacă ar exista n ≤ 2008 cu d(n) ≥ 5, atunci n ≥
p1 p2 p3 p4 p5 ≥ 2 · 3 · 5 · 7 · 11 = 2310 > 2008, absurd, iar pentru n = 2 · 3 · 5 · 7
avem d(n) = 4.
Clasa a VIII-a
a b c a c b
VIII.95. Pentru a, b, c ∈ R∗ , notăm α = + + , β = + + . Calculaţi
b c a c b a
a3 b3 c3
numărul x = + + ı̂n funcţie α şi β.
b3 c3 a3
Elena Nicu, Malu-Mare (Dolj)

141
a b c
Soluţie. Fie x = , y = , z = ; atunci xyz = 1, x + y + z = α, iar
b c a
xy + yz + zx 1 1 1
= + + = β, de unde xy + xz + yz = β. Folosind identitatea
xyz x y z

x3 + y 3 + z 3 − 3xyz = (x + y + z)3 − 3(x + y + z)(xy + xz + yz),

rezultă că x3 + y 3 + z 3 = α3 − 3αβ + 3.


VIII.96. Rezolvaţi ı̂n numere naturale ecuaţia x2 + y 2 + xy = x2 y 2 .
Mihail Bencze, Braşov
Soluţie. Putem presupune că x ≤ y. Dacă x = 0, obţinem că y = 0. Dacă x = 1,
deducem că y = −1 ∈ / N. Pentru x = 2, găsim că 3y 2 − 2y − 4 = 0, ecuaţie care nu are
soluţii naturale. În cazul ı̂n care x ≥ 3, atunci y ≥ 3 şi vom avea că x2 − 1 > x + 1,
y 2 −1 > y+1, inegalităţi care, ı̂nmulţite, conduc la (x2 −1)(y 2 −1) > (x+1)(y+1). Însă
(x+1)(y+1) > xy+1, prin urmare (x2 −1)(y 2 −1) > xy+1, adică x2 +y 2 +xy < x2 y 2 .
În concluzie, singura soluţie a ecuaţiei date este (x, y) = (0, 0).
Soluţia 2. Vom rezolva ecuaţia ı̂n mulţimea numerelor ı̂ntregi. Fie (x, y) o soluţie.
Scriind ecuaţia sub forma (x + y)2 = xy(xy + 1), deducem că xy(xy + 1) este pătrat
perfect. Cum pentru xy ≥ 1, (xy)2 < xy(xy + 1) < (xy + 1)2 , iar pentru xy ≤ −2
(xy + 1)2 < xy(xy + 1) < (xy)2 , nu avem soluţii ı̂n aceste cazuri. Rezultă că xy = 0
sau xy = −1. Obţinem soluţiile: (0, 0); (1, −1) şi (−1, 1).
Soluţia 3 (Gheorghe Iurea). Fie (x, y) o soluţie cu x, y ∈ Z. Ecuaţia este
echivalentă cu (x + y)2 = (xy)2 + xy ⇔ 4(x + y)2 = (2xy + 1)2 − 1 ⇔ [2xy + 1 − 2(x +
y)][2xy + 1 + 2(x + y)] = 1, deci 2xy + 1 − 2(x + y) = 2xy + 1 + 2(x + y) ∈ {−1, 1}.
Obţinem soluţiile (0, 0); (1, −1) şi (−1, 1).
VIII.97. Fie d1 , d2 , d3 , d lungimile diagonalelor feţelor, respectiv diagonalei
2d2 d2
unui paralelipiped dreptunghic. Dacă d21 = 2 2 32 , să se arate că paralelipipedul are
√d2 + d3
d 3
o muchie de lungime cel puţin egală cu .
3
Gheorghe Molea, Curtea de Argeş
Soluţie. Fie d21 = a2 + b2 , d22 = b2 + c2 , d23 = a2 + c2 , d2 = a2 + b2 + c2 , unde a, b, c
sunt lungimile muchiilor paralelipipedului. Relaţia din ipoteză se scrie succesiv:

2d22 d23 2(b2 + c2 )(c2 + a2 )


d21 = ⇔ a2
+ b2
= ⇔ a4 + b4 = 2c4 .
d22 + d23 b2 + c2 + c2 + a2

Însă 2(a4 + b4 ) ≥ (a2 + b2 )2 , deci


√ 4c ≥ (a + b ) ⇔ 2c ≥ a + b ⇔ 3c ≥
4 2 2 2 2 2 2 2

d 3
a2 + b2 + c2 ⇔ 3c2 ≥ d2 , deci c ≥ . Avem egalitate pentru a = b = c, deci ı̂ntr-un
3
cub.
VIII.98. Fie VABCD piramidă patrulateră regulată. Notăm u = m((VÛ
BC),(ABC)),
v = m((V ÛBC) , (V CD)) şi t = m((V Û
BC) , (V AD)). Arătaţi că u + v + t > 180◦ .
Claudiu Ştefan Popa, Iaşi
Soluţie. Fie M şi N mijloacele laturilor [BC], respectiv [AD], iar P proiecţia

142
V
lui B pe V C; se arată că u = m(VÖ Õ
M N ), v = m(BP D), iar
Ö
t = m(M V N ). În △V M N avem că u + u + t = 180◦
şi atunci concluzia problemei rezultă dacă am arăta că
v > u. Evident că u < 90◦ , prin urmare dacă v ≥ 90◦ ,
demonstraţia este ı̂ncheiată. Presupunem că v < 90◦ ;
BD · P O VO . P
atunci v > u ⇔ sin v > sin u ⇔ 2
> . Cum D
BP VM C
V O · BD V M · BC
PO = , iar BP = , ultima inegali- . . . M
2V C VC N O
BD · V C
2
A B
tate revine la > 1. Însă BD2 = 2BC 2 , iar
2BC 2 · V M
V C > V M şi astfel soluţia problemei este completă.

VIII.99. Pentru n ∈ N∗ , considerăm A = 12 , 22 , 32 , . . . , n 2
√ . Determinaţi n,

ştiind că există o funcţie f : A → A astfel ı̂ncât f (x) − f (y) = x − y, ∀x, y ∈ A.
Cristian Lazăr, Iaşi
√ √
Soluţie.
√ Avem că f (x) − x = f (y) − √ y, ∀x, y ∈ A, deci ∃k ∈ R astfel ı̂ncât
f (x) − x = k, ∀x ∈ A, de unde f (x) = x + k, ∀x ∈ A. Deducem că, pentru
x < y din A, avem că f (x) < f (y) şi atunci, cum A este finită, rezultă că f (12 ) = 12 ,
f (22 ) = 22 , . . . , f (n2 ) = n2 , de unde 1 + k = 1 şi n + k = n2 . Deci, k = 0 şi n = 1.
VIII.100. Rezolvaţi ı̂n ı̂n N2 ecuaţia x2 − 8n + 1287 = 0.
Mihai Crăciun, Paşcani
Soluţie. Dacă n este impar, atunci 8n = (9 − 1)n = M9 − 1. Cum x2 dă la
.
ı̂mpărţirea prin 9 unul dintre resturile 0, 1, 4 sau 7, iar 1287..9, cantitatea din membrul
stâng al ecuaţiei dă la ı̂mpărţirea prin 9 unul dintre resturile 1, 2, 5 sau 8 şi se ajunge
la o contradicţie.
Dacă n = 2k, ecuaţia se scrie: (8k − x)(8k + x) = 32 · 11 · 13. Analizând pe rând
cazurile care se obţin, găsim soluţie doar când 8k − x = 11, 8k + x = 32 · 13, când vom
avea k = 2, x = 53. Deci, singura soluţie a ecuaţiei date este (n, x) = (4, 53).
VIII.101. Se calculează suma cifrelor pentru fiecare dintre numerele de la 1
la n, n > 10. Pentru fiecare sumă dintre cele n se calculează din nou suma cifrelor,
repetându-se această operaţie până când obţinem n numere formate din câte o singură
cifră. Să se afle n, ştiind că ı̂n mulţimea astfel obţinută cifrele 1, 2, 3 şi 4 se repetă
de câte 101 ori fiecare, iar cifrele 5, 6, 7, 8 şi 9 de câte 100 ori fiecare.
Mihai Haivas, Iaşi
Soluţie. Restul ı̂mpărţirii unui număr prin 9 este egal cu restul ı̂mpărţirii sumei
cifrelor sale prin 9. Atunci, cifra 1 se obţine din acele numere care dau restul 1
la ı̂mpărţirea prin
h n9,i adică din numerele 1, 10, 19 . . . . Pentru a obţine 1 de 101 ori,
trebuie să avem + 1 = 101, deci n ∈ {900, 901, . . . , 904}. Se verifică faptul că
9
doar n = 904 satisface şi celelalte condiţii din ipoteză.
Clasa a IX-a
2
IX.91. Fie a, b, c, p ∈ R, p > 0. Dacă ax + bx + c ≤ p, ∀x ∈ [−1, 1], atunci
cx2 + bx + a ≤ 2p, ∀x ∈ [−1, 1].
Dorin Mărghidanu, Corabia

143
Soluţie. Fie f (x) = ax2 + bx + c, g(x) = cx2 + bx + a, x ∈ [−1, 1]. Din ipoteză
se obţine că |a + b + c| = |f (1)| ≤ p, |a − b + c| = |f (−1)| ≤ p, iar |c| = |f (0)| ≤ p.
Folosind inegalitatea modulului, avem:
x+1 1−x
|g(x)| = |cx2 + bx + a| = |c(x2 − 1) + (a + b + c) · + (a − b + c) · |≤
2 2
|x + 1| |1 − x|
≤ |c| · |x2 − 1| + |a + b + c| · + |a − b + c| · ≤
2 2
 ‹  ‹
|x + 1| |1 − x| x+1 1−x
≤ p · |x2 − 1| + + = p 1 − x2 + + =
2 2 2 2
= p(2 − x2 ) ≤ 2p, ∀x ∈ [−1, 1].
IX.92. Fie n ∈ N, n ≥ 3, iar α, β ∈ R astfel ı̂ncât n + α + β ̸= 0. Arătaţi că
(1 + α) · · · (n + α) X
n−2
i+1
− (1 + α) · · · (n − 1 + α) + (−1) (1 + α) · · · (n − i − 1 + α)×
n+α+β i=1

n n β (β + 1) · · · (β + n − 1)
×β (β + 1) · · · (β + i − 1) + (−1) β · · · (β + n − 2) = (−1) .
n+α+β
Gheorghe Costovici, Iaşi
Soluţie. Notăm cu Sn membrul stâng al egalităţii. Pentru n = 3, avem că
(1 + α)(2 + α)(3 + α)
S3 = − (1 + α)(2 + α) + (1 + α) · β −
3+α+β
β(β + 1)(β + 2)
−β(β + 1) = . . . = − ,
3+α+β
calculele fiind de rutină. Fie n ≥ 4, iar σn−2 = Sn − (−1)n β(β + 1) . . . (β + n − 2).
Ar fi suficient să dovedim că
1
(∗) σk = (−1)k−1 (1+α) . . . (n−k−1+α)β(β+1) . . . (β+k)· , ∀k ∈ 1, n − 2
n+α+β
odată demonstrată (∗), vom obţine că
Sn = σn−2 + (−1)n β(β + 1) . . . (β + n − 2) =
 ‹
1+α
= (−1)n β(β + 1) . . . (β + n − 2) 1 − =
n+α+β
β(β + 1) . . . (β + n − 1)
= (−1)n · ,
n+α+β
adică tocmai ceea ce trebuia demonstrat.
Justficăm (∗) prin inducţie după k:
(1 + α) . . . (n + α)
σ1 = − (1 + α) . . . (n − 1 + α) + (1 + α) . . . (n − 2 + α)β =
n+α+β
−β
= (1 + α) . . . (n − 1 + α) · + (1 + α) . . . (n − 2 + β) · β =
n+α+β
(1 + α) . . . (n − 2 + α)β(β + 1)
= ,
n+α+β

144
deci (∗) este adevărată pentru k = 1. Presupunem (∗) adevărată pentru k ∈ {1, 2, . . . , n−
3}; atunci

σk+1 = σk + (−1)k (1 + α) . . . (n − k − 2 + α)β(β + 1) · . . . · (β + k) =


 ‹
n−k−1+α
= (−1) (1 + α) . . . (n − k − 2 + α)β · . . . · (β + k) 1 −
k
=
n+α+β
1
= (−1)k (1 + α) . . . (n − k − 2 + α)β · . . . · (β + k + 1) · ,
n+α+β
deci (∗) este adevărată şi pentru k + 1 şi astfel soluţia problemei este completă.
IX.93. Fie △ABC dreptunghic cu m(A) b = 90◦ şi AB = 3 , iar D mijlocul lui
AC 2
[AC]. Notăm cu E punctul de intersecţie a cercurilor C1 (A, AD) şi C2 (B, BC), aflat
de aceeaşi parte a dreptei AB ca şi punctul C. Determinaţi măsura unghiului CAE. Õ
Cătălin Ţigăeru, Suceava
Soluţie. Fie {M,√ N } = AB ∩√ C 1 , cu N ∈ (AB), iar a = AD; atunci AC = 2a,
AB = 3a, BC = a 13, BD = a 10, iar BM = 4a. C
Cum BD < BC < BM, ı̂nseamnă că cercurile C1 şi
C2 sunt secante, iar 90◦ < m(BAE)Õ < 180◦ . Aplicăm
D
teorema cosinusului ı̂n △ABE : BE 2 = AB 2 +AE 2 − E ..
Õ ⇔ 13a2 = 9a2 + a2 − 2 · 3a · a ·
2AB · AE · cos(BAE)
Õ de unde obţinem cos(BAE)
cos(BAE), Õ = − 1 , adică M
. A
.
N B
2
Õ = 120◦ . Deducem că m(CAE)
m(BAE) Õ = 120◦ − 90◦ = 30◦ .
IX.94. În △ABC, I este centrul cercului ı̂nscris, iar {M } = AI∩BC. Demonstraţi
că bisectoarea unghiului AMÖ C, BI şi AC sunt trei drepte concurente dacă şi numai
b
dacă m(A) = 120 . ◦

Vlad Emanuel, student şi Andrei Cozma, elev, Bucureţi


Soluţie. Notăm cu a, b, c lungimile laturilor şi fie {P } = BI ∩ AC; avem că
2bc A ab AP c
AM = ·cos , M C = , iar = . Atunci: A
b+c 2 b+c PC a
BI, AC şi bisectoarea unghiului AM Ö C sunt concurente P
Ö AP AM
⇔ M P este bisectoarea lui AM C ⇔ = ⇔ I
PC MC
c 2c A A 1 b = 120◦ .
= cos ⇔ cos = ⇔ m(A)
a a 2 2 2 B M C
IX.95. Dacă xi ∈ [0, a], i = 1, n şi xn+1 = x1 , demonstraţi că
X
n
na2
xi+1 (a − xi ) < π.
i=1 4 sin2
n
Gigel Buth, Satu Mare
Soluţie. Considerăm poligonul regulat de latură a, A1 A2 . . . An , ı̂nscris ı̂n cercul
C(O, r). Luăm pe fiecare latură [Ai Ai+1 ] câte un punct Pi astfel ı̂ncât Ai Pi = xi , i =
1, n (cu convenţia An+1 = A1 ). Dacă Si este aria triunghiului Pi Ai+1 Pi+1 , iar S este

145
X
n
xi+1 (a − xi ) 2π na2 π
aria poligonului, atunci Si < S, iar Si = · sin ,S= π · cos .
i=1
2 n 4 sin n
n
Obţinem că
π 1 X
n
π na2 π
2 sincos · · xi+1 (a − xi ) < π · cos ,
n n 2 i=1 4 sin n
n
de unde rezultă concluzia problemei.
Clasa a X-a
X.91. Arătaţi că
 2  
1 2  3 2 π 2 1 4  3 4 π4
arctg + arctg + = 2 arctg + arctg + .
7 4 16 7 4 256

D. M. Bătineţu-Giurgiu, Bucureşti
Soluţie. Se demonstrează relativ simplu identitatea

[x2 + y 2 + (x + y)2 ]2 = 2[x4 + y 4 + (x + y)4 ],

cunoscută sub numele de identitatea lui G. Candido. Concluzia problemei rezultă


1 3 tg x + tg y
observând că, dacă x = arctg şi y = arctg , atunci tg(x + y) = = 1,
7 4 1 − tg x tg y
π π
deci x + y = (deoarece x + y < ).
4 2
X.92. Fie a, b ∈ C. Demonstraţi că ecuaţia z − az
2
+ b = 0 are ambele soluţii de
modul 1 dacă şi numai dacă |b| = 1 şi |a| + a − 4b = 4. (În legătură cu X.77 din
2 2

RecMat - 1/2007.)
Marian Tetiva, Bârlad
Soluţie. Putem proceda ca ı̂n soluţia problemei X.77, care poate fi consul-
tată ı̂n RecMat 1/2008, pp. 65. În cele ce urmează, vom prezenta o soluţie care
foloseşte scrierea trigonometrică a numerelor complexe. Presupunem ı̂ntâi că ecuaţia
are soluţiile z1 , z2 de modul 1, deci zk = cos tk + i sin tk‹ , tk ∈ [0, 2π), k = 1, 2. Atunci
t1 − t2 t1 + t2 t1 + t2
a = z1 + z2 = 2 cos cos + i sin , b = z1 z2 = cos(t1 + t2 ) +
2 2 2
t1 − t2
i sin(t1 + t2 ), iar a2 − 4b = −4 sin2 (cos(t1 + t2 ) + i sin(t1 + t2 )). Evident că
 2 ‹
t1 − t2 t1 − t2
|b| = 1, iar |a|2 + |a2 − 4b| = 4 cos2 + sin2 = 4.
2 2
Reciproc, fie |b| = 1 şi |a|2 + |a2 − 4b| = 4. Atunci |a2 | + |4b − a2 | = 4 = 4|b| =
|a + 4b − a2 |, prin urmare există t ≥ 0 astfel ı̂ncât 4b − a2 = ta2 sau avem a2 = 0. În
2

al doilea caz, ecuaţia devine z 2 + b = 0 şi, cum |b| = 1, ambele soluţii vor fi de modul √
1±i t
1. În primul caz, discriminantul este ∆ = a − 4b = −ta , de unde z1,2 = a ·
2 2
,
É Ê 2
1+t (1 + t)a
2 È
cu modulul |z1 | = |z2 | = |a| · = =
|b| = 1, ceea ce trebuia
4 4
demonstrat.

146
X.93. Dacă a1 , a2 , . . . , an ∈ (0, 1) sau a1 , a2 , . . . , an ∈ (1, ∞), iar f, g : {1, 2, . . . , n}
→ {1, 2, . . . , n} sunt funcţii injective, să se arate că
X
n ‹ X
n ‹
logak af (k)
ak ≥ n2 .
ag(k)
k=1 k=1

Dan Popescu, Suceava


Soluţie. Se ı̂nmulţesc membru cu membru următoarele
Ì două inegalităţi, care
X
n Y
n X
n
logak af (k)
rezultă din inegalitatea mediilor: ak ≥ n · n
ak şi ≥
agk
k=1 k=1 k=1
Ï
Q
n
logak af (k)
k=1 n n
n· n
Q
n = Ê = Ê (am ţinut seama de faptul că funcţiile
Q
n Q
n
ag(k) n
ag(k) n
ak
l=1 k=1 k=1
f şi g sunt chiar bijective).
X.94. a) Să se arate că
p √ p
x2n + y 2n + xn y n + x2n + z 2n + xn z n ≥ y 2n + z 2n + y n z n , ∀x, y, z ∈ R∗ , ∀n ∈ N.
b) Demonstraţi că, dacă n este par, inegalitatea este strictă, iar dacă n este impar,
atunci există x, y, z ∈ R pentru care se atinge egalitatea.
Bogdan Victor ‚ Grigoriu,
√ Œ Fălticeni
n n
y y 3
Soluţie. a) Considerând ı̂n plan punctele A(xn , 0), B − , şi
2 2
‚ √ Œ
zn zn 3 p √
C − ,− , observăm că AB = x2n + y 2n + xn y n , AC = x2n + z 2n + xn z n ,
2 2
p
iar BC = y 2n + z 2n + y n z n . Concluzia rezultă aplicând inegalitatea triunghiului.
b) Dacă n este par, atunci numerele xn , y n şi z n sunt strict pozitive, prin urmare
B se va situa ı̂n cadranul II, iar C ı̂n cadranul III, ı̂n timp ce A se va afla pe semiaxa
pozitivă Ox. În acest caz, punctele A, B şi C nu pot fi coliniare, prin urmare ine-
galitatea de la a) este strictă.
√ Dacă n este impar, egalitatea se atinge, de exemplu,
pentru x = 1, y = z = − n 2.
X.95. Considerăm funcţia f : R3 → R,

f (x, y, z) = sin x + sin y + sin z + sin (x − y) + sin (y − z) + sin (z − x) .


Determinaţi maximul şi minimul funcţiei f .
Cătălin Calistru, Iaşi
Soluţie. Cu substituţiile x = x1 + x2 , y = x2 + x3 , z = x3 + x1 , avem:

f (x, y, z) = sin(x1 + x2 ) + sin(x2 + x3 ) + sin(x3 + x1 ) + sin(x1 − x3 )+


+ sin(x2 − x1 ) + sin(x3 − x2 ) = 2 sin x1 cos x3 + 2 sin x2 cos x1 + 2 sin x3 cos x2 .

Prin urmare, 3+f (x, y, z) = (sin x1 +cos x3 )2 +(sin x2 +cos x1 )2 +(sin x2 +cos x2 )2 ≥ 0
şi 3 − f (x, y, z) = (sin x1 − cos x3 )2 + (sin x2 − cos x1 )2 + (sin x3 − cos x2 )2 ≥ 0, deci

147
π π π π π π
−3 ≤ f (x, y, z) ≤ 3, ∀(x, y, z) ∈ R3 . Cum f ( , , ) = 3, iar f (− , − , − ) = −3,
2 2 2 2 2 2
rezultă că max f = 3 şi min f = −3.

Clasa a XI-a
XI.91. Fie matricele A, B, C, D ∈ Mn (R) astfel ı̂ncât AC + BD = In , iar
AD = BC. Demonstraţi că CA + DB = In şi DA = CB.
I. V. Maftei, Bucureşti şi Mihai Haivas, Iaşi
Soluţie. Din datele problemei obţinem că (A−iB)(C +iD) = In ; atunci matricele
A − iB şi C + iD sunt una inversa celeilalte, prin urmare (C + iD)(A − iB) = In .
Deducem că (CA + DB) + i(DA − CB) = In şi, cum CA + DB, DA − CB ∈ Mn (R),
atunci CA + DB = In şi DA − CB = On .
 ‹
1 1
XI.92. Determinaţi matricele X ∈ M2 (R) pentru care X + X = 2
.
1 1
Adrian Reisner, Paris
 ‹
a b
Soluţie. Fie X = ∈ M2 (R); atunci X 2 = (a + d)X − (ad − bc)I2 şi
c d
 ‹
1 1
ecuaţia devine X(a + d + 1) − (ad − bc)I2 = . Obţinem sistemul:
1 1

8
> a(a + d + 1) − (ad − bc) = 1 (1)
<
b(a + d + 1) = 1 (2)
>
:
c(a + d + 1) = 1 (3)
d(a + d + 1) − (ad − bc) = 1 (4)

Din ecuaţiile (1) şi (4) rezultă că (a + d + 1)(a − d) = 0 şi cum a + d + 1 ̸= 0,
1 1
atunci d = a. Din ecuaţiile (2) şi (3) obţinem că b = c = = .
a+d+1 2a + 1
1
Folosind acum ecuaţia (1), avem că a(2a + 1) − (a2 − ) = 1, echivalent cu
(2a + 1)2
3 1
a(a + 1)(4a2 + 4a − 3) = 0, deci a ∈ {− , , −1, 0}. Găsim soluţiile
2 2
 ‹  ‹  ‹  ‹
0 1 −1 −1 1 1 1 1 3 1
X1 = , X2 = , X3 = , X4 = − .
1 0 −1 −1 2 1 1 2 1 3
un + 1
XI.93. Studiaţi convergenţa şirului (un )n≥1 definit prin u1 ≥ 0, un+1 = ,
u2n + 1
∀n ∈ N∗ .
Gheorghe Costovici şi Adrian Corduneanu, Iaşi
Soluţie. Vom arăta că (un ) este convergent către 1. Dacă u1 ∈ {0, 1}, afirmaţia
este imediată. Cazurile u1 ∈ (0, 1) şi u1 ∈ (1, ∞) tratându-se asemănător, ne vom
fixa atenţia asupra primului. Evident că un > 0, ∀n ∈ N şi se arată uşor prin
inducţie că u2n−1 < 1 şi u2n > 1, ∀n ∈ N∗ . Vom demonstra acum că u2n+2 < u2n şi

148
u2n+1 > u2n−1 , ∀n ∈ N∗ ; efectuăm calculele doar pentru prima afirmaţie:
 ‹ Á  ‹2 
u2n+1 + 1 u2n + 1 u2n + 1
u2n+2 < u2n ⇔ 2 < u2n ⇔ +1 +1
u2n+1 + 1 u22n + 1 u22n + 1
u42n + u32n + 3u22n + u2n + 2
< u2n ⇔ < u2n ⇔ u52n − u42n + 2u22n −
u42n + 3u22n + 2u2n + 2
−u22n + u2n − 2 > 0 ⇔ (u2n − 1)(u42n + 2u22n + u2n + 2) > 0,

adevărat. În concluzie, subşirul (u2n−1 )n≥1 este crescător şi mărginit, iar subşirul
(u2n )n≥1 este descrescător şi mărginit inferior de 1; ı̂nseamnă că ambele sunt con-
vergente spre α, respectiv β. Trecând la limită ı̂n relaţia de recurenţă, obţinem că
β+1 α+1
α= 2 , iar β = 2 . Înlocuind, găsim pentru α ecuaţia α5 − α4 + 2α3 − α2 +
β +1 α +1
α − 2 = 0, deci (α − 1)(α4 + 2α2 + α + 2) = 0 şi, cum α > 0, ı̂nseamnă că α = 1.
Deducem apoi că β = 1, prin urmare lim un = 1.
n→∞

Notă. În aceeaşi manieră, autorii problemei au stabilit că pentru orice a, b ∈
uan + b
(0, ∞), şirul (un )n≥1 definit prin u1 ≥ 0, un+1 = a+1 este convergent către 1.
un + b
XI.94. Să se demonstreze că pentru orice n ∈ N∗ , există numerele distincte
 4 n
x1 , x2 , . . . , xn ∈ (1, 2), aşa ı̂ncât x1 x2 · · · xn = .
e
Dan Plăeşu , Iaşi
Soluţia 1 (a autorului). Considerăm funcţia f : [1, 2]•→ R, f (x) = x ln˜x − x.
k−1 k
Aplicând teorema lui Lagrange funcţiei f pe intervalele 1 + ,1 + ,k =
 ‹ n n  ‹
k−1 k k
1, 2, . . . , n, determinăm xk ∈ 1 + ,1 + , k = 1, 2, . . . , n, astfel ı̂ncât f 1 + −
 ‹ n n n
k−1 1
f 1+ = f ′ (xk ), k = 1, 2, . . . , n. Rezultă că f ′ (x1 ) + f ′ (x2 ) + . . . + f ′ (xn ) =
n n
Pn   k
‹ 
k−1
‹‹
n· f 1+ −f 1+ = f (2) − f (1). Folosind faptul că f ′ (x) = ln x
k=1 n n
4
şi că f (2) − f (1) = ln , din ultima relaţie obţinem concluza problemei.
e
Soluţia 2 (Gabriel Popa). Construcţia şirului (xn )n≥1 se poate face inductiv:
4
luăm x1 = ∈ (1, 2), iar dacă presupunem existenţa numerelor x1 , x2 , . . . , xn ∈ (1, 2)
e  ‹n
4 4
pentru care x1 x2 . . . xn = , atunci numerele x1 , . . . , xn−1 , kn xn şi , unde
e  ‹n+1n
k e
4
(kn )n≥1 este un şir de numere din (1, 2) cu limita 1, au produsul şi pot fi
e
făcute distincte, alegând convenabil kn (posibil, ı̂ntrucât orice vecinătate la dreapta
a lui 1 este nenumărabilă).
 ‹ Soluţia 3 (Gheorghe
 ‹ Iurea). Căutăm numerele xk , k = 1, n de forma xk =
4 ak 3
cu ak ∈ 0, , k = 1, n. Condiţia problemei devine a1 + a2 + . . . + an = n.
e 2

149
1 1 1 1 1
Pentru n = 2p putem alege numerele 1 − , 1 + , 1 − , . . . , 1 − ,1+ , iar
3 3 4 p+2 p+2
1 1 1 1 1
pentru n = 2p + 1 putem alege numerele 1, 1 − , 1 + , 1 − , . . . , 1 − , 1+ .
3 3 4 p+2 p+2
 
1   1  2α  1  nα
1 1

XI.95. Calculaţi lim 1 + α + 1 + α + ··· + 1 + α − n , unde


n→∞ n n n
α ≥ 1 este fixat. (În legătură cu L83 din RecMat-1/2005.)
Marius Olteanu, Rm. Vâlcea
 ‹ 1
1 kα
Soluţie. Notăm cu (xn )n≥1 şirul a cărui limită o căutăm. Cum 1 + α >
n
1, ∀k = 1, n, deducem că xn > 0, ∀n ∈ N∗ . Din inegalitatea lui Bernoulli, obţinem că
 ‹ 1  ‹
1 kα 1 1 1 1 1
1+ α < 1 + α · α , deci xn < α 1 + α + . . . + α , ∀n ∈ N∗ . Din teo-
n n k  n ‹2 n
1 1 1 1 1
rema Cesarò-Stolz, lim α 1 + α + . . . + α = lim · .
n→∞ n 2 n n→∞ (n + 1) α (n + 1)α − nα
Dacă vom dovedi că această din urmă limită este 0, conform criteriului cleştelui rezultă
că lim xn = 0.
n→∞
Aplicând teorema lui Lagrange funcţiei f : [n, n+1] → (0, ∞), f (x) = xα , deducem
1 1 1
că (n + 1)α − nα = α · cα−1 , cu c ∈ (n, n + 1). Atunci · < ·
α (n + 1)2α−1 (n + 1)α
1 1 1
< · , ceea ce, prin trecere la limită, conduce la faptul
(n + 1)α − nα α (n + 1)α · nα−1
1 1
că lim · = 0.
n→∞ (n + 1)α (n + 1)α − nα

Clasa a XII-a Z 1
x
XII.91. Prove that (1 + x) ex(1+e ) dx = ee − 1.
0
Zdravko Starc, Vrs̆ac, Serbia
Z 1
x
Soluţie. Facem substituţia xex = t; atunci (x+1)ex dx = dt şi (1+x)ex(1+e ) dx =
Z e e 0
t
e dt = e = e − 1.
t e
0 0

XII.92. Fie b > a > 0, iar f : [a, b] → R o funcţieZ continuă pe [a, b] şi derivabilă
c
pe (a, b); să se arate că există c ∈ (a, b) astfel ı̂ncât b f (x) dx = c (b − c) f (c).
a
Dan Nedeianu, Drobeta Tr.Z Severin
x−b x
Soluţie. Aplicăm teorema lui Rolle funcţiei g : [a, b] → R, g(x) = f (t)dt
Z x x a
b x−b
şi ţinem cont că g ′ (x) = 2 f (t)dt + f (x).
x a x
Z π
sin x 2 2 cos 1
XII.93. Demonstraţi că există c ∈ (2, π) pentru care dx ≤ .
1 x c
Constantin Micu, Melineşti (Dolj)

150
h πi
Soluţie. Se constată uşor că funcţia f : 1, → R, f (x) = sin x este strict
h πi 2
1
crescătoare, iar g : 1, → R, g(x) = este strict descrescătoare; atunci, conform
2 x
inegalităţii lui Cebı̂şev,
Z π Z π Z π
1 2 sin x 1 2 2 1
· ≤€ Š2 · sin xdx · dx,
π
−1 x x
2 −1
2 1 π 1 1

Z π
2sin x ln π − ln 2
de unde dx ≤ 2 cos 1 · . Aplicând teorema lui Lagrange funcţiei
1 x π−2
1 ln π − ln 2
h : [2, π] → R, h(x) = ln x, găsim c ∈ (2, π) pentru care = şi astfel
c π−2
soluţia problemei este completă.
Z 2n
xa + b
XII.94. Calculaţi lim n √ dx, unde a ∈ (0, ∞) şi b ∈ R.
n→∞ n x2a+4 + 1
Liviu Smarandache,
Z x Craiova
xa + b ta + b
Soluţie. Fie f : [0, ∞) → R, f (x) = √ şi F (x) = √ dt,
Z x a x2a+4 + 1  0
2a+4 + 1

t +b 1 1 b 1 1
x ≥ 0. Cum 0 ≤ F (x) ≤ a+2
dt = − + a+1
− , obţinem
0 t x 2x a + 1 x (2x)a+1
că lim F (x) = 0. Atunci
x→∞

Z 2x
ta + b F (2x) − F (x)
lim x √ dt = lim x(F (2x) − F (x)) = lim 1 =
x→∞ t 2a+4 +1 x→∞ x→∞
x x
2f (2x) − f (x) 1 1 1
= lim = lim (x2 f (x) − (2x)2 f (2x)) = 1 − = ,
x→∞ − x21 x→∞ 2 2 2

xa+2 + bx2 1
deoarece lim x2 f (x) = lim √ = 1. Prin urmare, limita cerută este .
x→∞ x→∞ x 2a+4 +1 2
XII.95. Fie (A, +, ·) un inel ı̂n care 0 ̸= 1 şi 1 + 1 + 1 + 1 + 1 = 0. Să se arate
că, dacă x3 y 2 = y 2 x3 , ∀x, y ∈ A, atunci inelul este comutativ.
I.V. Maftei, Bucureşti şi Mihai Haivas, Iaşi
Soluţie. Fie x, y ∈ A, arbitrare. Din x3 y 2 = y 2 x3 şi x3 (y + 1)2 = (y + 1)2 x3 ,
găsim că x3 (y + y) = (y + y)x3 . Cum 1 + 1 + 1 + 1 + 1 = 0, rezultă că 5y = 0, deci
x3 (5y) = (5y)x3 . Deducem că x3 (3y) = (3y)x3 , prin urmare x3 y = yx3 . Folosind
această relaţie pentru x şi x + 1, obţinem x3 y = yx3 şi (x + 1)3 y = y(x + 1)3 , de
unde (3x2 + 3x + 1)y = y(3x2 + 3x + 1). Înlocuind pe x cu x + 1 şi ţinând cont că
3(x+1)2 +3(x+1)+ 1−(3x2 +3x+1) = 6x+6 = x+1, rezultă că (x+ 1)y = y(x+1),
deci xy = yx. Cum x, y sunt arbitrare din A, concluzia problemei se impune.

151
Soluţiile problemelor pentru pregătirea
concursurilor propuse ı̂n nr. 2/2008

A. Nivel gimnazial
G146. Fie x, y, z ∈ (0, ∞) astfel ı̂ncât xyz = 1. Arătaţi că

xy 3 yz 3 zx3
+ + ≥ 1.
x4 + y + z y4 + z + x z4 + x + y
Liviu Smarandache şi Lucian Tuţescu, Craiova
Soluţie. Avem că x3 + y 3 ≥ xy(x + y), cu egalitate pentru x = y. Atunci
X xy 3 X xy 3 X y3 X x3
= = ≥ = 1.
x4 + y + z x4 + xyz(y + z) x3 + yz(y + z) x3 + y 3 + z 3
Egalitatea se atinge pentru x = y = z = 1.
G147. Fie n ∈ N, n ≥ 2, fixat, iar a, b,h c sunt numere naturale astfel ı̂ncât
n − 1i
na + (n + 1) b + 2nc = n + 1. Arătaţi că n −
2
≤ a + b + c ≤ n.
2
Gheorghe Iurea, Iaşi
.
Soluţie. Cum b − 1 = n(n − a − b − 2c), ı̂nseamnă că b − 1..n. Însă b < n, altfel
na + (n + 1)b + 2nc > n2 + 1 şi atunci b − 1 = 0, deci b = 1. Condiţia din enunţ devine
na + n + 1 + 2nc = n2 + 1 ⇔ a + 2c = n − 1 ⇔ a + b + c = n − c. Suma a + b + c
este maximă când c este minim, adică pentru c = 0; deducem că (a + b + c)max = n,
maxim atins când a = n •− 1, b = ˜ 1, c = 0. Suma a + b + c este minimă • când
˜ c este
n−1 n−1
maxim, deci pentru c = ; obţinem că (a + b + c)min = n − , minim
• 2 ˜ • ˜ 2
n−1 n−1
atins când a = n − 1 − 2 , b = 1, c = .
2 2
G148. Fie a1 a2 . . . ap ∈ N. Să se arate că orice număr natural are un multiplu
de forma a1 a2 . . . ap a1 a2 . . . ap . . . a1 a2 . . . ap 0 . . . 0.
Marian Panţiruc, Iaşi
Soluţie. Fie n ∈ N; considerăm numerele: a1 a2 . . . ap , a1 a2 . . . ap a1 a2 . . . ap , . . . ,
a1 a2 . . . ap a1 a2 . . . ap . . . a1 a2 . . . ap , ı̂n număr de (n + 1). Prin ı̂mpărţirea acestora la
n obţinem (n + 1) resturi şi, cum resturile posibile sunt 0, 1, . . . , n − 1, rezultă că
cel puţin două resturi sunt egale. Fie a = a1 a2 . . . ap . . . a1 a2 . . . ap (format din k1
numere a1 a2 . . . ap şi b = a1 a2 . . . ap . . . a1 a2 . . . ap (format din k2 numere a1 a2 . . . ap )
două dintre numerele de mai sus, care dau acelaşi rest la ı̂mpărţirea cu n. Diferenţa
acestora se divide cu n, deci a−b = a1 a2 . . . ap a1 a2 . . . ap . . . a1 a2 . . . ap 00 . . . 0 verifică
cerinţa problemei.
G149. a) Determinaţi două numere prime p, q astfel ı̂ncât p < q, iar p2 − 1 are
mai mulţi divizori naturali decât q 2 − 1.
b) Determinaţi toate numerele prime p pentru care p2 − 1 are exact opt divizori
naturali.
Dan Popescu, Suceava

152
Soluţie. a) De exemplu, putem lua p = 19, q = 23.
b) Dacă p ̸= 2, atunci p2 ≡ 1 (mod 8). Dacă p ̸= 3, atunci p2 ≡ 1 (mod 3).
Cum (3, 8) = 1, ı̂nseamnă că dacă p ∈
/ {2, 3}, atunci p2 ≡ 1 (mod 24), prin urmare
.
p2 − 1..24. Pentru p ≥ 7, avem, că p2 − 1 > 24 şi, cum 24 are opt divizori naturali,
p2 − 1 va avea mai mult de opt divizori. Pentru p ∈ {2, 3}, p2 − 1 are mai puţin de opt
divizori. Dacă p = 5, atunci p2 − 1 = 24 are exact opt divizori, deci singurul număr
care satisface condiţiile din enunţ este 5.
G150. Fie m şi n numere naturale nenule cu proprietatea că m ≤ 1 + 2 + · · · + n.
Să se arate că m poate fi scris ca suma câtorva numere distincte dintre 1, 2,. . . , n.
Marian
§
Tetiva, Bârlad ª
n(n + 1)
Soluţia 1 (Dan Mocanu, elev, Iaşi). Acoperim mulţimea 1, 2, 3, . . . ,
2
cu următoarele sume având termenii distincţi: 1, 2, 3, . . . , n, n+1, n+2, . . . , n+(n−1),
n + (n − 1) + 1, n + (n − 1) + 2, . . . , n + (n − 1) + (n − 2), . . . , n + (n − 1) + . . . + 1;
concluzia problemei este acum imediată.
Soluţia 2 (a autorului). Demonstrăm prin inducţie după n. Pentru n = 1, deci
m = 1, nu avem nimic de demonstrat; de asemenea se verifică uşor cazul n = 2 (deci
m = 1, 2 sau 3). Vom presupune mai departe n ≥ 2 şi că afirmaţia este adevărată
pentru n − 1 şi o demonstrăm pentru n.
Dacă m este unul dintre numerele 1, 2, . . . , n nu avem ce arăta. Dacă m ≥ n + 1,
avem că 1 ≤ m − n ≤ 1 + 2 + . . . (n − 1) şi, conform ipotezei de inducţie, m − n este
suma unor termeni distincţi din mulţimea {1, 2, . . . , (n − 1)}. Rezultă că m este suma
dintre aceşti termeni şi n, ceea ce ı̂ncheie demonstraţia.
G151. Bazele unei prisme sunt poligoane cu 2008 vârfuri. Numerotăm cu 1,
2,. . . , 2008 vârfurile bazei inferioare şi, corespunzător, cu a1 , a2 ,. . . , a2008 vârfurile
bazei superioare, unde {a1 , a2 , . . . , a2008 } = {1, 2, . . . , 2008}.
a) Demonstraţi că putem găsi o numerotare pentru baza superioară astfel ı̂ncât
.
i + ai .. 8, ∀i ∈ {1, 2, . . . , 2008}.
b) Demonstraţi că nu putem găsi o numerotare pentru baza superioară astfel ı̂ncât
.
i + ai .. 9, ∀i ∈ {1, 2, . . . , 2008}.
Gabriel Popa şi Gheorghe Iurea, Iaşi
Soluţie. a) De exemplu, putem lua a1 = 2007, a2 = 2006, . . . ,a2007 = 1, a2008 =
2008.
.
b) Dacă ar exsta o numerotare pentru care i + ai ..9, ∀i ∈ {1, 2, . . . , 2008}, atunci
. .
(1 + a1 ) + (2 + a2 ) + . . . + (2008 + a2008 )..9, prin urmare 2(1 + 2 + . . . + 2008)..9. Am
.
obţine astfel că 2008 · 2009..9, contradicţie.
G152. În triunghiul isoscel ABC (AB = AC) notăm cu B ′ , C ′ picioarele ı̂nălţi-
milor din B, respectiv C. Dacă AB = 2 B ′ C ′ , să se determine unghiurile triunghiului.
Nela Ciceu, Bacău şi Titu Zvonaru, Comăneşti
Soluţie. Vom analiza două cazuri, după cum unghiul A b este ascuţit sau obtuz.
a) Fie M mijlocul laturii [AB]; atunci [B M ] va fi mediană ı̂n △ABB ′

153
1 1
A
dreptunghic şi deducem că B ′ M = AB. Avem şi că B ′ C ′ = AB,
2 2
prin urmare B ′ C ′ = B ′ M , deci BÖ′M B ≡ B Ö
′ C ′ A. Însă BÖ ′C ′A ≡

Ò (deoarece B ′ C ′ ∥BC) şi atunci m(B)


B Ò = m(B × ′ M C ′ ) = 180◦ −
M
Ö ′ ◦ ◦ b b
2m(M BB ) = 180 −2[90 −m(A)] = 2m(A), relaţie care ı̂mpreună
C B
b
cu m(A)+2m( Ò = 180◦ conduce la m(A)
B) b = 36◦ , m(B) Ò = m(C) Ò =
72◦ .
b este obtuz, atunci △B ′ M A va fi isoscel şi B C
b) Dacă A B C
cu un raţionament asemănător celui de mai sus obţinem A
Ò = m(B
că m(B) × ′ C ′ M ) = 180◦ − 2m(BÖ ′ AB) = 180◦ − 2 ·

2m(B) Ò = 180◦ − 4m(B), Ò prin urmare m(B) Ò = m(C) Ò = 36◦ , M


b
iar m(A) = 108 .◦
B C
G153. În triunghiul ABC, M este mijlocul laturii [BC], m(ABC) Õ = 30◦ şi
Õ ◦
m(ACB) = 105 . Perpendiculara din C pe AM taie AB ı̂n Q. Calculaţi valoarea
QA
raportului .
QB
Neculai Roman, Mirceşti (Iaşi)
Soluţie. Ducem CE⊥AB, E ∈ (AB). Folosind triunghiul CEM echilateral
şitriunghiul AEC dreptunghic isoscel, găsim că M E = EC = EA, deci triunghiul
AEM este isoscel cu m(AEM ) = 150◦ . Atunci m(EAM Ö ) = 15◦ , deci m(M Ö AC) = 30◦
Ö
şi m(AM C) = 45◦ . Prin urmare, m(BCQ) Õ = 45◦ şi m(ACQ) Õ = 60◦ . Obţinem că
QA AACQ BC · CQ sin BCQ AC sin 60◦
= = = .
QB ABCQ AC · CQ Õ BC sin 45◦
√sin ACQ √
√ BC 2 QA 3
Dar AC = EC 2 = şi atunci = .
2 QB 2
G154. Fie D mijlocul laturii [BC] a triunghiului echilateral ABC de latură 1,
iar P un punct mobil pe [CD]. Notăm cu M şi N proiecţiile pe AP ale punctelor B,
respectiv C. Aflaţi aria locului geometric descris de segmentul [M N ].
Marius Olteanu, Rm. Vâlcea
Soluţie. Observând că patrulaterele ABDM A
şi ACN D sunt inscriptibile, deducem că punctele
M şi N se află pe cercurile C1 (circumscris tri-
unghiului ABD), respectiv C2 (circumscris tri-
unghiului ACD). Dacă E este mijlocul laturii F E
[AC], se constată că M parcurge arcul mic DE ÷ al
cercului C1 , ı̂n timp ce N parcurge arcul mic CD ÷ M P
C2
al cercului C2 . Astfel, locul geometric măturat de
[M N ] este suprafaţa haşurată ı̂n figură. Observăm B C D N C
că segmentele de disc mărginite de C1 şi [DE], re- 1
spectiv de C2 şi [CD], sunt congruente; atunci √ aria locului geometric va fi egală cu
3
aria triunghiului echilateral CDE, adică .
16

154
G155. Fie C cercul circumscris △ABC ascuţitunghic. Notăm cu P punctul de
intersecţie al tangentelor duse la cerc ı̂n B şi C, {D} = AP ∩ C, iar M şi N sunt
⌢ ⌢
mijloacele arcului mic BC, respectiv arcului mare BC. Să se arate că dreptele AM ,
DN şi BC sunt concurente.
Gabriel Popa, Iaşi
Soluţie. Fie {T } = AM ∩BC. Cum AM este bisectoarea unghiului BAC,Õ rezultă
că
BT AB N
(1) = . A
TC AC
Vom arăta că DT este bisectoare pentru BDC;Õ atunci C
D, T, N vor fi coliniare şi de aici concluzia problemei.
Cum BAPÕ ≡ P Õ BD, rezultă că △P AB ∼ △P BD,
PB AB T
deci = . Analog se arată că △P AC ∼ △P CD, .
PD BD B C
PC AC AB
de unde = . Însă P B = P C, prin urmare =
PD CD BD ..
AC AB BD D M
⇔ = . Ţinând cont de (1), obţinem că
CD AC CD
BT BD Õ ceea ce
= , adică DT este bisectoarea lui BDC,
TC DC
ı̂ncheie rezolvarea. P

B. Nivel liceal
L146. În plan se consideră dreptele d1 , d2 ,. . . , dn+1 , oricare două neparalele.
Notăm cu αk = m(d Ø ◦
k , dk+1 ), αk ≤ 90 , k = 1, n. Pe d1 se consideră un seg-
ment de lungime 2 care se proiectează pe d2 , apoi segmentul obţinut se proiectează
pe d3 şi tot aşa, până când pe d se obţine un segment de lungime 1. Ştiind că
€ ¦ ©Š È √ n+1
tg min αi | i = 1, n = n
4 − 1, determinaţi unghiurile αk , k = 1, n.
Cristian Săvescu, student, Bucureşti
Soluţie. Fie A1 B1 segmentul de lungime 2 de pe d1 , iar Ak = prdk Ak−1 ,
Bk = prdk Bk−1 , k = 2, n + 1. Cum Ak+1 Bk+1 = Ak Bk · cos αk , atunci An+1 Bn+1 =
1
A1 B1 cos α1 cos α2 . . . cos αn , de unde cos α1 cos α2 . . . cos αn = . Fie αp = min{αi |i =
 πi 2
1, n}; cum cosinusul este descrescător pe 0, , avem cos αp ≥ cos αk , ∀k = 1, n şi
2 È√
1 1 È n
4−1
astfel cosn αp ≥ , deci cos αp ≥ √ n
, apoi sin αp = 1 − cos2 αp ≤ √
n
, prin
2È 2 2

urmare tg αp ≤ n
4 − 1. Conform ipotezei, rezultă că se atinge egalitatea; acest
lucru are loc pentru cos α1 = cos α2 . . . = cos αn , deci când α1 = α2 = . . . = αn =
1
arccos √n
.
2
L147. Se consideră un poligon convex cu n laturi, n ≥ 4, având proprietatea că
oricare două diagonale nu sunt paralele şi oricare trei nu sunt concurente ı̂n puncte
diferite de vârfurile poligonului. Se notează cu ni numărul punctelor de intersecţie a

155
diagonalelor interioare poligonului şi cu ne cel al punctelor de intersecţie exterioare
poligonului.
a) Să se arate că există exact opt poligoane care verifică relaţia ni > ne .
b) Să se arate că există exact trei poligoane pentru care ni + ne = kn2 , k ∈ N∗ .
Mihai Haivas, Iaşi
Soluţie. Fiecare punct interior de intersecţie a diagonalelor este unic determinat
de cele două diagonale ce-l conţin, deci de patru vârfuri ale poligonului; rezultă că
n(n − 3)
ni = Cn4 . Cum sunt diagonale, care se intersectează ı̂n C 2n(n−3) puncte,
2 2
2
fiecare vârf al poligonului fiind numărat de Cn−3 ori (se obţine ca intersecţie a oricare
două diagonale care trec prin acel vârf), obţinem că numărul total de intersecţii, fără
n(n − 3)(n2 − 7n + 14)
vârfuri, este ni + ne = C 2n(n−3) − nCn−3 2
= . Prin urmare,
2 8
n(n − 3)(n − 4)(n − 5)
nl = .
12
a) Din condiţia ni > ne rezultă că n3 −15n+38 < 0, cu soluţiile n ∈ {4, 5, 6, . . . , 11}.
b) Condiţia ni + ne = kn2 este echivalentă cu n3 − 10n2 + (35 − 8k)n − 42 = 0,
deci n ∈ {7, 14, 12}, cărora le corespund valorile k ∈ {1, 11, 33}.
Notă. Într-o manieră asemănătoare a rezolvat problema dl. Daniel Văcaru,
Piteşti.
L148. Pe latura (AB) a triunghiului ABC considerăm punctul D astfel ı̂ncât
AB = 4 AD. De aceeaşi parte a laturii AB ca şi punctul C, luăm un punct P
Õ
astfel ı̂ncât P DA ≡ ACB Õ şi P B = 2 P D. Demonstraţi că patrulaterul ABCP este
inscriptibil.
Nela Ciceu, Bacău şi Titu Zvonaru, Comăneşti
Soluţia 1 (a autorilor). Considerăm punctul Q astfel ı̂ncât AB separă P
Õ ≡ BP Õ AQ
şi Q, AB = 2AQ şi QAB D. Atunci = A
 ‹ AB
PD 1
= şi obţinem că △AQB ∼ △P DB, de unde P
PB 2
Õ = P
AQB Õ DB. Rezultă că m(AQB) Õ + m(ACB) Õ = Q D
m(PÕ DB) + m(P ÕDA) = 180◦ , ceea ce asgură inscriptibil-
itatea patrulaterului AQBC. Din AB = 2AQ şi AB =
AQ AD B C
4AD deducem că AQ2 = AD · AB, astfel scris = , prin urmare △AQD ∼
AB AQ
△ABQ. Obţinem că ADQ Õ ≡ AQB Õ şi cum avem şi AQB
Õ ≡P ÕDB, rezultă că punctele
Q, D şi P sunt coliniare. Ne amintim că QAB Õ ≡ QP Õ B, deci patrulaterul AQBP
este inscriptibil. Inscriptibilitatea patrulaterelor AQBC şi AQBP arată că punctele
A, Q, B, C, P sunt conciclice, de unde concluzia problemei.
Soluţia 2 (Daniel Văcaru, Piteşti). Folosind relaţia lui Stewart ı̂n △P AB,
BD AD 3 1
obţinem că P A2 · − P D2 + P B 2 · = AD · DB, deci P A2 · − P D2 + P B 2 · =
AB AB 4 4
3 3 3 1
AB 2 şi, cum P B = 2P D, deducem că P A2 · = AB 2 , adică P A = AB.
16 4 16 2
Aplicăm acum teorema cosinusului ı̂n △ADP : P A2 = AD2 +DP 2 −2AD ·DP ·cos C,

156
1 3
de unde DP 2 − AB · DP · cos C − AB 2 = 0, relaţie pe care o vom privi ca ecuaţie
2 16
de gradul II ı̂n necunoscuta DP. Singura soluţie pozitivă a acestei ecuaţii este DP =
1 √ P A2 + P B 2 − AB 2
· AB · (cos C + 3 + cos2 C). Cum cos AP Õ B= , ı̂nlocuind P A
4 2P A · P B
Õ
şi P D cu valorile găsite anterior şi făcând calculele, obţinem cos AP Õ
B = cos(ACB),
prin urmare AP Õ Õ ceea ce arată că patrulaterul ABCP este inscriptibil.
B ≡ ACB,
L149. Să se determine poziţia punctului P pe directoarea parabolei P, astfel ı̂ncât
aria triunghiului P T1 T2 să fie minimă, unde T1 şi T2 sunt punctele de contact cu
parabola ale tangentelor duse din P la P.
Adrian Corduneanu, Iaşi
Soluţia 1 (a autorului). Raportăm parabola la un reper canonic, având originea
ı̂n vârful O al parabolei şi drept axă a absciselor, perpendiculara dusă din O pe
directoare. Ecuaţia parabolei este P : y 2 = 2px(p > 0), iar cea a directoarei d : x =
p
− . Se ştie (sau se verifică!) faptul că tangentele P T1 şi P T2 , duse la parabolă din
2
1
punctul P ∈ d, sunt ortogonale; prin urmare, SP T1 T2 = P T1 · P T2 .
2
Fie T1 (x0 , y0 ), cu y02 = 2px0 , y0 > 0. Ecuaţia tangentei P T1 este yy0 = p(x+x0 ) şi
  2
p p2 y0 p 2 p2 y02
cum xp = − , obţinem yp = − 2
+ , deci P T1 = x0 + + y0 + − =
2 2y0 2 2 2y0 2
1 y0
(y 2 + p2 )3 . Tangenta P T2 trece prin P şi are panta − ; ecuaţia sa va fi y =
4p2 y02 0 p
y0 p2
− x− . Intersectând această tangentă cu parabolă, găsim coordonatele punc-
p 2y0
 3   3 2  2
p p2 p p p2 p2 y0
tului T2 , anume , − . Astfel, P T 2
2 = + + − + − =
2y02 y0 2y02 2 y0 2y0 2
1
(y 2 + p2 )3 .
4y04 0
Pătratul ariei triunghiului P T1 T2 se poate scrie acum ı̂n funcţie de parametrul y0 :
 ‹2
1 1 (y02 + p2 )3
S(y0 ) = · P T1 · P T2 = · . Derivata acestei funcţii este S ′ (y0 ) =
2 8p y03
3
(y 2 + p2 )2 (y02 − p2 ) şi se anulează doar ı̂n y0 = p (deoarece y0 a fost considerat
8py04 0
pozitiv). Obţinem că S are un minim egal cu Smin = p2 , care se atinge când y0 = p,
p
deci pentru punctul P − , 0 .
2
Soluţia 2 (Gheorghe Costovici, Iaşi). Vom rezolva problema ı̂ntr-un caz ceva
mai general, când P nu aparţine numaidecât directoarei parabolei, ci unei drepte
perpendiculare pe axa de simetrie a acesteia şi care nu are niciun punct comun cu
parabola. Fie y 2 = ax, a > 0, ecuaţia parabolei, iar Ti (xi , yi ), cu yi2 = axi , x1 ̸= x2 ,
1
xi ̸= 0. Tangenta ı̂n Ti la parabolă are ecuaţia (y +yi ) = axxi , i = 1, 2. Intersectând
2 x + x 
1 2
cele două tangente, obţinem coordonatele punctului P , anume P , ax1 x2 .
2
Fie y = k, k < 0, ecuaţia dreptei pe care se află P ; atunci k = ax1 x2 < 0, deci

157
putem alege x1 > 0, x2 < 0. Aria triunghiului P T1 T2 va fi

x1 + x2  ‹  
1 1 2
k
1 k a k2
SP T1 T2 = 1 x1 y1 = − x1 k − x21 + 2 2 .
2 2 ax1 2 a x1
1 x2 y2

Gândim această arie ca o funcţie ı̂n nedeterminata x1 , derivata acestei funcţii É este
 ‹ ‹2
3 a k k −k
· 4 · x21 + x21 − , iar punctele critice ale funcţiei sunt x′1 = şi
4 x1 É a a a
−k
x′′1 = − . Studiind semnul derivatei, se observă că x′1 este punct de minim
a
pentru arie şi obţinem că xp = 0. În concluzie, punctul P căutat este intersecţia
dreptei date cu Ox.
L150. Fie tetraedrul A1 A2 A3 A4 , iar P un punct ı̂n interiorul său. Notăm
cu Aij ∈ (Ai Aj ) proiecţiile ortogonale ale lui P pe muchiile Ai Aj ale tetraedrului.
Demonstraţi că
1
VP A12 A13 A23 + VP A12 A14 A24 + VP A13 A14 A34 + VP A23 A24 A34 ≤ VA1 A2 A3 A4 .
4
Când se atinge egalitatea?
Marius Olteanu, Rm. Vâlcea
Soluţie. Fie P1 = P r(A2 A3 A4 ) A1 ; din reciproca teoremei celor trei perpendiculare,
obţinem că P1 A23 ⊥ A2 A3 , P1 A24 ⊥ A2 A4 , P1 A34 ⊥ A3 A4 , prin urmare A23 A24 A34
este triunghiul podar al punctului P1 ı̂n raport cu △A2 A3 A4 . Aria s1 a acestui
triunghi este cel mult un sfert din aria S1 a △A2 A3 A4 . Dacă mai notăm h1 = A1 P1 ,
VP A23 A24 A34 x1 s1 1 x1
x1 = P P1 , atunci = ≤ · . Introducem analog x2 , x3 , x4
VA1 A2 A3 A4 h1 S 1 4 h1
şi h2 , h3 , h4 ; putem scrie ı̂ncă trei inegalităţi analoage celei precedente. Concluzia
problemei se obţine adunând cele patru ‹ inegalităţi şi ţinând seama de relaţia lui
x1 x2 x3 x4
Gergonne + + + =1 .
h1 h2 h3 h4
Deoarece 4si = Si doar atunci când Pi coincide cu centrul cercului circumscris
feţei care se opune vârfului Ai , rezultă că egalitatea se atinge când P este centrul
sferei circumscrise tetraedrului.
L151. Să se demonstreze că nu există numere naturale n şi k astfel ı̂ncât
h€ √ Š2n+1 i h€ √ Šk i
2+ 3 = 4 + 15 .

√ Cosmin Manea şi Dragoş√ Petrică, Piteşti


Soluţie. Avem că (2 + 3)2n+1 = 22n+1 + C2n+1 1
22n 3 + C2n+1
2
22n−1 · 3 +
√ 2n+1 √ √ 2n+1 √
2n+1
. . . + C2n+1 ( 3) = an + bn 3, cu an , bn ∈ N, iar (2 − 3) = an − bn 3.
√ 2n+1 √ 2n+1 √ 2n+1 √ 2n+1
Astfel, [(2 + 3) √] = [(2 + 3) + (2 − 3) − 1√+ 1 − (2 − 3) ] =
[2an − 1 + 1 − (2√ − 3)2n+1 ] = 2a√
n − 1, deoarece
√ 1 − (2 − 3)√ 2n+1
∈ (0, 1). La fel
√ că (4 + 15) = ck + dk 15, (4 + 15) = ck − dk 15, ck , dk ∈ N, iar
k k
stabilim
[(4 + 15)k ] = 2ck − 1. Dacă presupunem că există n şi k cu proprietăţile cerute,

158
avem an = ck , deci 22n+1 + M3 = 4k + M15 ; rezultă că 3|2 · 4n − 4k . Dar 2 · 4n − 4k =
2(M3 + 1) − (M3 + 1) = M3 + 1 şi obţinem contradicţia 3|1, prin urmare nu există n
şi k cu proprietatea cerută.
L152. Pentru a, b, c ∈ R şi x ∈ R+ , demonstraţi inegalitatea
2 4
9 3 (x + 1) (a + b + c) 1 1 1
2 2 2
≤” 2
—
2
≤ 2+ 2+ 2.
a +b +c 2 2 2 2
3(x + 1)(a + b + c ) + 2x(a + b + c) (ab + bc + ca) a b c

I. V. Maftei şi Dorel Băiţan, Bucureşti


2
XSoluţie. Utilizând
X sumarea
X ciclică, inegalitatea
X din
Xstânga se scrie 3[3(x +
1)( 2
a ) + 2x( a) ] · (
2
ab) ≤ (x + 1)(
2 2 4
a) ( 2
a ) şi aceasta se obţine
adunând inegalităţile
P P P P P
(1) 9( ab)2 ≤ ( a)4 ; 3( ab)2 ≤ ( a)2 ( a2 ),
X X
prima multiplicată prin (x2 + 1)( a2 ), iar a doua prin 2x( a)2 .
Inegalitatea din dreapta se scrie sub forma
X X X X 2x X 2
(x + 1)2 a2 b2 c2 ( a)4 ≤ ( a2 b2 )( ab)2 [(x2 + 1)( a2 ) + ( a) ]
3
şi se poate obţine adunând inegalităţile
P P P P P 1 P 2 2 P
(2) a2 b2 c2 ( a)4 ≤ ( a2 b2 )( ab)2 ( a2 ); a2 b2 c2 ( a)2 ≤ ( a b )( ab)2 ,
3
X
prima multiplicată prin (x2 + 1), iar a doua prin 2x( a)2 .
X
Justificarea inegalităţilor (1) şi (2) revine la demonstrarea inegalităţilor 9( ab)2 ≤
X X X X X X X
( a)4 ; ( a)2 ≤ 3( a2 ); 3abc( a) ≤ ( ab)2 ; abc( a) ≤ a2 b2 , care sunt
relativ uzuale.
L153. Găsiţi toate funcţiile f : R → R cu proprietatea că
€ Š
f x2 + xy + yf (y) = xf (x + y) + f 2 (y) , ∀x, y ∈ R.

Adrian Zahariuc, student, Princeton


Soluţie. Căutăm un z ̸= x astfel ı̂ncât x2 +xy = z 2 +zy; găsim z = −x−y. Atunci
xf (x + y) + f 2 (y) = f (x2 + xy + yf (y)) = f (z 2 + zy + yf (y)) = zf (z + y) + f 2 (y) =
(−x − y)f (−x) + f 2 (y). Rezultă că xf (x + y) = −(x + y)f (−x), pentru orice x, y ∈ R.
f (x)
Deducem că (x ̸= 0) este constantă, deci f (x) = cx, c ∈ R. Cum pentru x = 0
x
şi y ̸= 0 obţinem f (0) = 0, rezultă că f (x) = cx, c ∈ R, pentru orice x ∈ R.
Se verifică uşor că toate aceste funcţii au proprietatea dorită.
Notă. O soluţie corectă, dar ceva mai laborioasă, a fost primită din partea d-lui
Daniel Văcaru, Piteşti.
L154. Fie P ∈ R [X] un polinom de gradul n şi p : R → R funcţia polino-
mială asociată. Ştiind că mulţimea {x ∈ R | p (x) = 0} are k elemente (distincte), iar

159
funcţia f : R → R, f (x) = |p (x)| este derivabilă pe h nR,
i arătaţi că numărul maxim de
rădăcini complexe nereale ale lui P este egal cu 2 − 2k.
2
Vlad Emanuel, student, Bucureşti
Soluţie. Studiem derivabilitatea funcţiei f . Dacă x0 ∈ R şi p(x0 ) ̸= 0, atunci p
are semn constant ı̂ntr-o vecinătate a lui x0 . Pe această vecinătate f (x) = p(x) (sau
f (x) = −p(x). Rezultă că f este derivabilă ı̂n x0 . Dacă x0 ∈ R şi P (x0 ) = 0, atunci
|P (x)| − |P (x0 )| P (x) − P (x0 )

fs (x0 ) = x→x
lim lim
= − x→x = −|P ′ (x0 )| = −|P ′ (x0 )| şi

0 x − x0 x<x 0
0 x − x 0
s
x<x0
|P (x)| − |P (x0 )| P (x) − P (x0 )
fd′ (x0 ) = x→x
lim lim
= x→x = |P ′ (x0 )| = |P ′ (x0 )|.

x>x
0 x − x 0 x>x
0 x − x0
d
0 0

În concluzie, f este derivabilă ı̂n x0 dacă şi numai dacă −|P ′ (x0 )| = |P ′ (x0 )| ⇔

P (x0 ) = 0.
Prin urmare, f este derivabilă pe R dacă şi numai dacă orice rădăcină a lui P este
cel puţin dublă. Cum P are k rădăcini reale distincte, rezultă că P are cel puţin 2k
rădăcini reale.
Pentru n = 2p, deducem că P are cel mult 2p − 2k rădăcini complexe nereale, iar
pentru n = 2p + 1, P are cel mult 2p + 1 − (2k + 1) rădăcini complexe nereale h n i (am
folosit faptul că rădăcinile complexe sunt perechi). În final avem cel mult 2 − 2k
2
rădăcini complexe nereale.
Cum pentru n = 2p, P = (X − 1)2 . . . (X − k)2 (X 2 + 1)p−k verifică ipotezele
problemei şi are 2p − 2k rădăcini complexe nereale, iar pentru n = 2p + 1, P =
(X − 1)3 (X − 2)2 . . . (X − k)2 (X 2 + 1)p−k verifică ipotezele
h n i şi are 2p − 2k rădăcini
nereale, conclucionăm că numărul căutat este egal cu 2 − 2k.
2
L155. Fie A, B ∈ M2 (C) două matrice astfel ı̂ncât matricea AB − BA să fie
−1
inversabilă. Să se arate că urma matricei (I2 + AB) (AB − BA) este egală cu 1.
Florina Cârlan şi Marian Tetiva, Bârlad
Soluţie. Considerăm plinomul f (x) = det[I2 +AB +x(BA−AB)], care are gradul
doi (coeficientul lui x2 este egal cu det(BA − AB) = det(AB − BA) ̸= 0). Observăm
1
că f (0) = f (1), ceea ce arată că punctul de extrem al funcţiei f este x = .
2
Pe de altă parte, f (x) = det(BA − AB)det[(I2 + AB)(BA − AB)−1 + xI2 ] =
det(BA − AB)(x2 + xtr(P ) + detP ), unde P = (I2 + AB)(BA − AB)−1 , ceea ce
1 1
ı̂nseamnă că punctul de extrem este − tr(P ) = − tr[(I2 + AB)(BA − AB)−1 ]. Prin
2 2
1
urmare, − 2 tr[(I2 + AB)(BA − AB) ] = , de unde tr[(I2 + AB)(AB − BA)−1 ] = 1.
1 −1
2
Notă. Soluţie corectă a dat dl. Daniel Văcaru, Piteşti.

160
Probleme propuse1

Clasele primare
P.174. Mirela are un măr, o pară şi o portocală. Mama ı̂i spune să aşeze fructele
pe două farfurii astfel ı̂ncât pe fiecare farfurie să fie cel mult două fructe. În câte
moduri poate aşeza Mirela cele trei fructe?
(Clasa I ) Inst. Maria Racu, Iaşi
P.175. Scrieţi toate numerele mai mici ca 27 care se pot des-
compune sub forma indicată alăturat.
(Clasa I ) Diana Tănăsoaie, elevă, Iaşi a a+1 a+2 a+3

P.176. Într-o bombonieră sunt cinci bomboane cu fructe şi şapte bomboane cu
ciocolată. Care este cel mai mic număr de bomboane pe care trebuie să-l luăm din
bombonieră, fără să ne uităm, pentru a avea cel puţin două bomboane cu ciocolată?
(Clasa a II-a) Alexandru Dumitru Chiriac, elev, Iaşi
P.177. Cum măsurăm 1litru de apă folosind două vase negradate, unul de 5litru,
iar celălalt de 8litri?
(Clasa a II-a) Mariana Nastasia, elevă, Iaşi
P.178. Arătaţi că, dacă restul este o cincime din scăzător, atunci descăzutul se
ı̂mparte exact la 6. Care este cel mai apropiat descăzut de numărul 100 cu această
proprietate?
(Clasa a III-a) Mirela Cucoranu, elevă, Iaşi
P.179. Se dau produsele: a × b = 60, a × c = 70, a × d = 95. Ştiind că b + c + d
este de 9 ori mai mare decât a, să se afle valoarea lui a.
(Clasa a III-a) Andreea Amarandei, elevă, Iaşi
P.180. Arătaţi că din şirul 7, 28, 31, 46, 61, 100 nu putem extrage patru numere a
căror sumă să se ı̂mpartă exact la trei.
(Clasa a III-a) Dragoş Iacob, elev, Iaşi
P.181. Un triunghi şi un pătrat au acelaşi perimetru, exprimat printr-un număr
natural. Care este cea mai mică valoare a perimetrului? Câte valori posibile ale
perimetrului sunt cuprinse ı̂ntre 100 şi 200?
(Clasa a IV-a) Andreea Alexa, elevă, Iaşi
1 2 3 k
P.182. Aflaţi cea mai mică valoare a lui k astfel ı̂ncât + + + ··· +
21 21 21 21
să fie un număr natural.
(Clasa a IV-a) Ionela Bărăgan, elevă, Iaşi
P.183. Se consideră nouă numere naturale a, b, c, . . . , i. Media aritmetică a nu-
merelor a şi b este 1, media numerelor c, d şi e este 5, iar media numerelor f, g, h şi i
este 11. Aflaţi media aritmetică a numerelor a, b, c, . . . , i şi 9.
(Clasa a IV-a) Ionel Nechifor, Iaşi
1 Se primesc soluţii până la data de 31 iunie 2010.

161
Clasa a V-a
V.109. Aflaţi câtul şi restul ı̂mpărţirii numărului 3 · 22009 la 5 · 22007 .
Damian Marinescu, Târgovişte
V.110. Determinaţi patru numere naturale x, y, z, t cu proprietatea că 2x−1
+ 3 · 22y+1 + 5 · 23z+2 + 11 · 25t+1 = 2009.
Cătălina Drăgan, Galaţi
V.111. Demonstraţi că numărul 20200 are 261 de cifre la scrierea ı̂n baza 10.
Geanina Hăvârneanu, Iaşi
§ ª
3n + 4
V.112. Demonstraţi că mulţimea A = x = n ∈ N, 1000 ≤ n ≤ 2009 are
4n + 3
1010 elemente.
Daniela Munteanu, Iaşi
1 1 1 13
V.113. Dacă S = 1 + + + . . . + , demonstraţi că S > .
2 3 2009 2
Al. Gabriel Mı̂rşanu, Iaşi
V.114. Se consideră ı̂n plan cinci drepte distincte, care ı̂mpart planul ı̂n mai
multe regiuni. Arătaţi că oricum am alege 2009 puncte din plan, vor exista cel puţin
126 de puncte dintr-o aceeaşi regiune.
Nicolae Ivăşchescu, Craiova
V.115. O mulţime de numere naturale A = {a1 , a2 , . . . , a9 } are elementele
aranjate strict crescător; media aritmetică a primelor două elemente este 1, media
următoarelor trei este 5, iar media ultimelor patru este 11. Câte astfel de mulţimi
există?
Ionel Nechifor şi Gabriel Popa, Iaşi

Clasa a VI-a
VI.109. Determinaţi a, b, c, d ∈ N pentru care 2a + 3b + 5c + 7d = 87, dacă:
a) a, b, c, d sunt numere prime;
b) a, b, c, d sunt pătrate perfecte.
Nicolae Ivăşchescu, Craiova
VI.110. Determinaţi perechile de numere naturale care au suma 2009 şi produsul
multiplu al numărului 2009.
Dan Popescu, Suceava
 ‹
1 1
VI.111. Demonstraţi că numărul A = 40! · 1 + + . . . + este natural,
2 40
divizibil cu 2009 · 72 (unde 40! = 1 · 2 · . . . · 40).
Mihai Haivas, Iaşi
VI.112. Fie a, b, c, d ∈ N∗ astfel ı̂ncât ad + bc = bd. Demonstraţi că

a2009 a2008 c a2 c ac c
2009
+ 2008
+ . . . + 2
+ + ∈ N.
b b d b d bd d
Cătălin Budeanu, Iaşi

162
VI.113. După două reduceri succesive, preţul unui frigider scade de la 2000 lei la
1620 lei. Ştiind că cele două reduceri sunt proporţionale cu preţurile rămase ı̂n urma
lor, aflaţi preţul frigiderului după prima reducere.
Ciprian Baghiu, Iaşi
VI.114. Pe laturile [BC], [AC], [AB] ale triunghiului isoscel ABC (AB = AC)
Õ = 2m(EDC)
considerăm punctele D, E, respectiv F , astfel ı̂ncât m(BAD) Õ şi m(DAC)
Õ
Õ
= 2m(F DB). Demonstraţi că △AEF este isoscel.
Doru Buzac, Iaşi
VI.115. Dreptele a şi b sunt perpendiculare pe segmentul [AB] ı̂n A, respectiv
ı̂n B. Considerăm punctele C ∈ (AB), M ∈ a, N, P ∈ b astfel ı̂ncât ı̂ntre oricare
două dintre triunghiurile ACM, BCN şi BCP există câte o congruenţă. Ştiind că
m(BPÕ C) = 25◦ , determinaţi măsurile unghiurilor triunghiului M N P.
Andrei Nedelcu, Iaşi

Clasa a VII-a
VII.109. Fie ABCD dreptunghi, O mijlocul lui [AC], M ∈ (AO), N ∈ (OC),
{P } = BM ∩ AD şi {Q} = BN ∩ CD. Demonstraţi că O este centrul de greutate al
1
triunghiului BP Q dacă şi numai dacă OM = ON = AC.
6
Petru Asaftei, Iaşi
VII.110. Măsurile unghiurilor A, √ B şi C
√ ale triunghiului ABC sunt direct propor-
ţionale cu 5, 4 şi 3, iar BC = (2 + 2 2 + 2 3) cm. Demonstraţi că perimetrul şi aria
triunghiului sunt numeric egale.
Constantin Apostol, Rm. Sărat
VII.111. Fie ABC un triunghi şi punctele D ∈ (AC), E ∈ (AB), {P } = BD ∩
DA PC PD
CE. Dacă = k, demonstraţi că k − (k + 1) = 1.
DC PE PB
Neculai Roman, Mirceşti (Iaşi)
VII.112. Fie ABCD trapez cu baza mare [AB], {E} = AD ∩ BC, {O} =
AC ∩ BD, iar OP ∥AB, cu P ∈ (AD). Demonstraţi că CP şi CE sunt bisectoarele
Õ dacă şi numai dacă AB = AC.
(interioară, respectiv exterioară) unghiului ACD,
Claudiu Ştefan Popa, Iaşi

√ 3 · bc
VII.113. a) Demonstraţi că b2 − bc + c2 ≥ √ , ∀b, c ∈ R∗+ .
b + bc + c2
2

b) Considerând un triunghi ABC cu m(A) b = 120◦ , AB = c, AC = b, interpretaţi


geometric inegalitatea de la a).
Dan Mocanu, elev, Iaşi
VII.114. Demonstraţi că produsul a două numere naturale nenule consecutive
nu poate fi egal cu produsul altor patru numere naturale consecutive.
Mihai Crăciun, Paşcani
1 1 1
VII.115. Demonstraţi că + + ··· + k > k, ∀n, k ∈ N∗ .
n+1 n+2 4 ·n
Cosmin Manea şi Dragoş Petrică, Piteşti

163
Clasa a VIII-a
VIII.109. Fie ABCDA′ B ′ C ′ D′ un cub de muchie a. Notăm cu E, F, G, H, K, L
mijloacele muchiilor AB, BC, CC ′ , C ′ D′ , D′ A′ , respectiv A′ A. Calculaţi volumul poli-
edrului B ′ EF GHKL.
Adrian Corduneanu, Iaşi
VIII.110. Fie V ABCD piramidă patrulateră regulată. Notăm cu
u = m((VÛ BC), (ABC)), v = m((V Û BC), (V CD)) şi t = m((V Û BC), (V AD)). Stabiliţi
dacă printre numerele u, v, t pot exista perechi de numere egale. (În legătură cu
VIII.98 din RecMat 2/2008.)
Claudiu Ştefan Popa, Iaşi

VIII.111. Fie ABC un triunghi de laturi a, b, c, astfel ı̂ncât b + c = a 2.
Demonstraţi că triunghiul‚ √este ascuţitunghic
√ Œ dacă şi numai dacă b şi c sunt distincte
a 2 3a 2
şi se află ı̂n intervalul , .
4 4
Romanţa Ghiţă şi Ioan Ghiţă, Blaj
x È

VIII.112. Fie x, y ∈ R astfel ı̂ncât xy, şi y x2 + (x + 1)2 + x2 (x + 1)2 sunt
y
toate numere raţionale. Demonstraţi că x şi y sunt tot numere raţionale.
Dan Nedeianu, Drobeta Tr. Severin
 ‹
1 1 1 1 1 1
VIII.113. Dacă a, b, c ∈ R∗+ , demonstraţi că + + = 2 + +
a b c a+b b+c c+a
dacă şi numai dacă a = b = c.
D.M. Bătineţu-Giurgiu, Bucureşti
VIII.114. Demonstraţi că oricare ar fi numerele naturale impare m, n cu m >
n + 2, există numere naturale x, a, b astfel ı̂ncât x = a(a + m) = b(b + n).
Titu Zvoranu, Comăneşti
VIII.115. Demonstraţi că 5(a2 + b2 )2 ≤ 4a4 b4 + (a + b)4 , ∀a, b ∈ [1, +∞).
Lucian Tuţescu şi Ion Vişan, Craiova

Clasa a IX-a
IX.101. Prin inducţie matematică se arată că are loc inegalitate a lui Bernoulli

(1) (1 + x)n ≥ 1 + nx,


∀n ∈ N, n ≥ 2 şi ∀x ∈ [−1, ∞), egalitatea fiind atinsă pentru x = 0. Arătaţi că:
a) dacă n = 2k, k ∈ N∗ , atunci (1) are loc ∀x ∈ R;
b) dacă n = 2k + 1, k ∈ N∗ , atunci (1) are loc ∀x ∈ [−2, ∞);
c) dacă n = 3, atunci (1) are loc ∀x ∈ [−3, +∞), cu egalitate când x ∈ {−3, 0},
iar pentru x ∈ (−∞, −3), (1) are loc cu sens contrar.
Dorin Dutkay, Orlando (U.S.A.) şi Florin Popovici, Braşov
IX.102. Rezolvaţi ı̂n R3 sistemul:
1 1 1 1 1 1 1 1 1
x+y+z = 2− − − ; x2 +y 2 +z 2 = 6− 2 − 2 − 2 ; x3 +y 3 +z 3 = 2− 3 − 3 − 3 .
x y z x y z x y z
Vasile Chiriac, Bacău

164
IX.103. Fie x, y, z ∈ R cu 0 ≤ x ≤ y ≤ z. Dacă α ∈ R este astfel ı̂ncât
αx + (1 − α)z ≥ 0, demonstraţi că αx + (1 − α)y ≥ 0 şi αy + (1 − α)z ≥ 0.
Ovidiu Pop, Satu Mare
IX.104. Fie A, B, C, D patru puncte ale cercului C(O, r), {M } = AB ∩ CD,
N şi P mijloacele coardelor [AB], respectiv [CD], iar Ω cel de-al patrulea vârf al
paralelogramului N M P Ω.
−−→ −−→ −−→ −−→ −−→
a) Arătaţi că M A + M B + M C + M D = 2M Ω.
b) Paralelele prin C şi D la AB şi paralelele prin A şi B la CD se taie două câte
două ı̂n patru puncte ce determină un paralelogram de centru Ω.
c) Ω = O dacă şi numai dacă AB ⊥ CD.
Diana Vrânceanu, elevă şi Dumitru Mihalache, Bârlad
IX.105. Într-un triunghi, cu notaţiile uzuale, demonstraţi echivalenţa condiţiilor:
(i) R = ra ; (ii) cos A = cos B + cos C.
Temistocle Bı̂rsan, Iaşi
Clasa a X-a
X
n arctg √ 1
2k+1
X.101. Calculaţi suma S = √ .
2k+1
k=1 arcsin k+1
Bencze Mihály, Braşov
 ‹
1 7 7
X.102. Rezolvaţi ecuaţia + log2 x − + = 0.
2x 4 4
Eugen Jecan, Dej
−→
X.103. Fie S, U, A trei puncte distincte. Rotind vectorul SA ı̂n jurul lui S, cu
−→
un arc α ∈ (−π, π), obţinem punctul S ′ ; rotind apoi U A ı̂n jurul lui U , cu un arc
−−→ −−−→
β ∈ (−π, π), obţinem U ′ , U ′ ̸= S ′ . Fie M ∈ S ′ U ′ astfel ı̂ncât S ′ M = k · M U ′ , unde
k ∈ R\{0, −1}. Demonstraţi că poziţia punctului M nu depinde de A atunci şi numai
atunci când k = 1, β = α ± π * .
Diana Vrânceanu, elevă şi Dumitru Mihalache, Bârlad
X.104. Fie p, la , lb , lc semiperimetrul, respectiv lungimile bisectoarelor unui tri-
unghi.√Determinaţi numerele reale α şi β, ı̂n funcţie de p, ştiind că soluţiile ecuaţiei
x3 − p 3 · x2 + αx − β = 0 sunt la , lb şi lc .
Cătălin Calistru, Iaşi
X.105. Determinaţi cel mai mare număr real α astfel ı̂ncât inegalitatea

sin x + sin y sin x+3y + sin 3x+y x+y


≤α· 4 4
+ (1 − α) sin
2 2 2
să fie adevărată pentru orice x, y ∈ [0, π].
Marian Tetiva, Bârlad
Clasa a XI-a  a x 
XI.101. Pentru a ∈ R∗+ , calculaţi lim x 1+ − ea .
x→∞ x
D.M. Bătineţu-Giurgiu, Bucureşti
* Generalizare a problemei comorii din insulă a lui G. Gamow, din One, Two, Theree . . . Infinity.

165
XI.102. Determinaţi funcţiile continue f : R → R cu proprietatea că f (x2 ) −
f (y 2 ) = (x + y)f (x − y), ∀x, y ∈ R.
Gheorghe Iurea, Iaşi

XI.103. Fie (xn )n≥1 ⊂ R+ astfel ı̂ncât lim n(xn+1 − xn ) = a ∈ (1, +∞); definim
n→∞
n ln n
şirul (yn )n≥1 prin yn = . Calculaţi lim (yn )n .
x1 + x2 + . . . + xn n→∞
Cosmin Manea şi Dragoş Petrică, Piteşti
 5 
1 25 35 n5
XI.104. Calculaţi lim + 2 + 3 + ... + n .
n→∞ 51 5 5 5
Neculai Roman, Mirceşti (Iaşi)
XI.105. Considerăm matricele Ak , Bk ∈ Mk (C), k ∈ N\{0, 1}, astfel ı̂ncât
detAk = α ∈ C∗ , ∀k ∈ N\{0, 1}. Studiaţi convergenţa şirului (an )n≥2 definit prin
Xn
det(Ak x + Bk )
an = lim .
x→∞ k! · xk
k=2
Cătălin Calistru, Iaşi
Clasa a XII-a
XII.101. Rezolvaţi ecuaţia x2 + x + 1 = 0 ı̂n Z13 şi ı̂n Z19 , apoi deduceţi că 247
divide (37 −1 − 1)(74 −1 − 1), oricare ar fi n ∈ N.
n n

Mihai Haivas, Iaşi şi I.V. Maftei, Bucureşti


 π cos 2x
XII.102. Determinaţi primitivele funcţiei f : 0, → R, f (x) = .
4 (tg x + ctgx)2009
Nicoleta Bran, Craiova
 π
XII.103. Demonstraţi că există c ∈ 0, pentru care
4
Z π
4 π 2 etg c
(etg x − 1)dx ≤ · .
0 32 cos2 c
Bogdan Victor Grigoriu, Fălticeni
XII.104. Determinaţi funcţiile derivabile f : [0, 1] → R pentru care f (0) = 0, iar
f ′ (x) = f (ax), ∀x ∈ [0, 1], cu a ∈ [0, 1] fixat.
Gheorghe Iurea, Iaşi
x + y  f (x) + f (y)
XII.105. Fie f : (0, ∞) → R o funcţie semiconvexă (f ≤ ,
2 2
∀x, y ∈ (0, ∞)).
• a) Demonstraţi
‹ că˜ pentru orice x ∈ (0, ∞), şirul (fn (x))n≥1 definit prin fn (x) =
1
n f x+ − f (x) , este monoton.
n

b) Deduceţi
‹
că pentru orice x ∈ (0, ∞), şirul (en (x))n≥1 definit prin en (x) =
1 n
1+ , este crescător.
nx
Dan Ştefan Marinescu şi Viorel Cornea, Hunedoara

166
Probleme pentru pregătirea concursurilor
A. Nivel gimnazial
G166. Demonstraţi că următoarele propoziţii sunt adevărate.
a) ∀n ∈ N, n ≥ 2, ∃x1 , x2 , . . . , xn ∈ N∗ astfel ı̂ncât x1 x2 + x2 x3 + . . . + xn x1 =
x1 x2 . . . xn .
b) ∀n ∈ N, n ≥ 5, @x1 , x2 , . . . , xn ∈ 2N∗ astfel ı̂ncât x1 x2 + x2 x3 + . . . + xn x1 =
x1 x2 . . . xn .
c) ∃x1 , x2 , . . . , xn ∈ 2N + 1 astfel ı̂ncât x1 x2 + x2 x3 + . . . + xn x1 = x1 x2 . . . xn ⇔
n ∈ 2N∗ + 1.
Dan Popescu, Suceava
G167. Fie 1 = d1 < d2 < . . . < dk = n toţi divizorii pozitivi ai numărului natural
n. Dacă există i, j cu j > i > 13 şi d27 + d2i = d2j , arătaţi că n este multiplu de 8.
Titu Zvonaru, Comăneşti
G168. Pentru x, y, z ∈ R∗+ , demonstraţi că are loc inegalitatea
 
x(y + z) y(x + z) z(x + y) x2 y2 z2
+ + ≤2 + + .
x + yz y + xz z + xy x + yz y + xz z + xy
Ştefan Gavril, Piatra Neamţ
G169. Demonstraţi că există o infinitate de numere iraţionale α cu proprietatea
că α3 şi α2 + α sunt, de asemenea, iraţionale.
Gabi Ghidoveanu şi Dumitru Mihalache, Bârlad
G170. O mulţime A ⊂ R, de cardinal 2009, are proprietatea că fiecare element
al ei este mai mare decât o zecime din suma celor 2008 numere rămase. Arătaţi că A
conţine cel puţin 12 numere negative.
Andrei Nedelcu, Iaşi
G171. Punctele planului care au, ı̂n raport cu un reper y
cartezian, ambele coordonate numere naturale, le parcurgem
ı̂n sensul indicat de săgeţi ı̂n figură, pornind din origine.
Notăm cu an,k poziţia punctului de coordonate (n, k) ı̂n şirul
obţinut (de exemplu, a0,0 = 1, a0,2 = 4, a2,2 = 13 etc.).
Exprimaţi numărul an,k ı̂n funcţie de n şi de k.
Lucian Georges Lăduncă, Iaşi x
0
G172. O tablă dreptunghiulară m × n, m, n ≥ 2, are pătrăţelele unitate de la
intersecţiile liniilor de ordin impar cu coloanele de ordin impar colorate ı̂n negru,
restul pătrăţelelor fiind albe. A recolora o linie (coloană) ı̂nseamnă a schimba culorile
tuturor pătrăţelelor acelei linii (coloane). Arătaţi că tabla nu poate fi transformată
ı̂ntr-una complet albă prin recolorarea câtorva linii şi coloane.
Răzvan Ceucă, elev, Iaşi
G173. Notăm cu T (a, b, c) triunghiul care are laturile de lungimi a, b şi c. Demon-
straţi că triunghiurile T (b, 2c, 2mb ) şi T (c, 2b, 2mc ) pot fi confecţionate (pe rând)
dintr-o aceeaşi bucată de carton, fără pierdere de material.
Petru Asaftei, Iaşi

167
b = 40◦ . Să se arate că
G174. Se consideră triunghiul ABC isoscel cu m(A)
nu există puncte P ∈ Int ABC pentru care m(PÕ Õ
AB) = 30◦ , m(P BC) = 10◦ şi
m(PÕCA) = 35 .◦

Gabriel Popa şi Paul Georgescu, Iaşi


G175. Fie ABCD un patrulater ı̂nscris ı̂n cercul de rază R. Demonstraţi că
AB · AD + CB · CD ≤ 4R2 .
Gheorghe Costovici, Iaşi

B. Nivel liceal
L166. Fie ABCD un dreptunghi, iar C un cerc prin A, care intersectează (AB),
(AC) şi (AD) ı̂n M, N , respectiv P . Arătaţi că AM · AB + AP · AD = AN · AC.
Gheorghe Iurea, Iaşi
L167. Fie ABC un triunghi cu AB > AC. Cercul ı̂nscris ı̂n triunghi este tangent
laturilor BC şi AC ı̂n D, respectiv E. Considerăm T un punct pe latura [BC] şi notăm
cu J centrul cercului ı̂nscris ı̂n △ABT. Dacă DE trece prin mijlocul segmentului [CJ],
demonstraţi că triunghiul AT C este isoscel.
Titu Zvonaru, Comăneşti
L168. Demonstraţi că ı̂n orice triunghi, cu notaţiile uzuale, are loc inegalitatea
a b c 11p2 − 15r2 − 60Rr 3
+ + ≥ ≥ .
b+c c+a a+b 6p2 − 6r2 − 24Rr 2
Marius Olteanu, Rm. Vâlcea
L169. Care este probabilitatea ca razele cercurilor exı̂nscrise unui triunghi ales
aleator, să fie laturile unui nou triunghi?
Petru Minuţ, Iaşi
L170. Fie n ∈ N, n ≥ 2 şi a1 , a2 , . . . , an ∈ R+ cu a1 +a2 +. . .+an = S. Considerăm
k ∈ N, 1 ≤ k ≤ n − 1 şi α1 , α2 ∈ R+ cu α1 + α2 = 1. Demonstraţi inegalitatea
X k α1 (n − k)α2 k
(ai1 + ai2 + . . . + aik )α1 (S − ai1 − . . . − aik )α2 ≤ Cn S.
n
1≤i1 <...<ik ≤n

(În legătură cu 6117 din R.M.T. 1/1987 )


Paul Georgescu şi Gabriel Popa, Iaşi
L171. Pentru x, y ∈ R∗+ , demonstraţi că are loc inegalitatea
√ È √ p p
x+3 2(x + y) + y ≤ 2( 3x + y + x + 3y).

Marian Tetiva, Bârlad


L172. Fie P ∈ Q[X], cu grP = n ≥ 1. Dacă P admite o rădăcină complexă a,
având ordinul de multiplicitate m, cu n < 2m, demonstraţi că a ∈ Q.
Adrian Reisner, Paris
L173. a) Există funcţii f : (a, b) → R cu proprietatea că |f (x) − f (y)| ≥ c,
∀x, y ∈ (a, b), unde c este o constantă pozitivă?

168
b) Există funcţii f : (a, b) ∩ Q → R cu proprietatea că |f (x) − f (y)| ≥ c, ∀x, y ∈
(a, b) ∩ Q, unde c este o constantă pozitivă?
Geanina Hăvârneanu, Iaşi
!2
X
n

L174. Fie a1 , a2 , . . . , an , b1 , b2 , . . . , bn numere reale pozitive şi a = ai ,
i=1
!2 " #
n √
X X
n
p √
b= bi . Arătaţi că există x0 > 0 astfel ı̂ncât ai x + bi − [ ax + b] ∈
i=1 i=1
{0, 1}, ∀x > x0 .
Marian Tetiva, Bârlad
L175. Arătaţi că
n
[2]
X
Cn2k Ωk = 2n Ωn , n ∈ N,
k=0

(2k − 1)!!
unde Ωk = , k ∈ N∗ (se convine ca Ω0 = 1).
(2k)!!
Gheorghe Costovici, Iaşi

Training problems for mathematical contests

A. Junior highschool level


G166. Prove that the following assertions are true:
a) ∀n ∈ N, n ≥ 2, ∃ x1 , x2 , . . . , xn ∈ N∗ such that x1 x2 +x2 x3 +xn x1 = x1 x2 . . . xn .
b) ∀n ∈ N, n ≥ 5, @ x1 , x2 , . . . , xn ∈ 2 N∗ such that x1 x2 + x2 x3 + xn x1 =
x1 x2 . . . xn .
c) ∃ x1 , x2 , . . . , xn ∈ N∗ such that x1 x2 +x2 x3 +xn x1 = x1 x2 . . . xn ⇔ n ∈ 2 N∗ +1.
Dan Popescu, Suceava
G167. Let d1 < d2 < . . . dk = n be all the positive divisors of the natural number
n. Assuming that the subscripts i, j with j > i > 13 exist such that d27 + d2i = d2j ,
show that n is a multiple of 8.
Titu Zvonaru, Comăneşti
G168. For any x, y, z ∈ R∗+ , prove that the following inequality holds :
 
x(y + z) y(x + z) z(x + y) x2 y2 z2
+ + ≤2 + + .
x+yz y + xz z + xy x+yz y + xz z + xy

Ştefan Gavril, Piatra Neamţ


G169. Prove that an infinity of irrational numbers α exist with the property that
α3 and α2 + α are irrational numbers as well.
Gabi Ghidoveanu and Dumitru Mihalache, Bârlad

169
G170. A subset A ⊂ R, of cardinal number 2009, has the property that each
of its elements is greater than one tenth of the sum of the remaining 2008 numbers.
Show that A contains at least 12 negative numbers.
Andrei Nedelcu, Iaşi
G171. The points in the plane whose both coordinates, y
in a Cartesian system of coordinates, are natural numbers,
visited along the path indicated by the arrows in the figure at
right, starting from the origin. We denote by an,k the position
of the point of coordinates (n, k) in the sequence thus obtained
(for instance, a0,0 = 1, a0,2 = 4, a2,2 = 13, etc.). Express the
number an,k in terms of n and k. x
0
Lucian Georges Lăduncă, Iaşi
G172. A rectangular board of size m × n, m, n ≥ 2, has unit squares at the
intersections of odd-order rows with odd-order columns, colored in black while the
other squares remain white. To re-color a row (column) means to change the color
of all the squares on that row (column). Show that the board cannot be turned to a
completely white board by recoloring a couple of rows and columns.
Răzvan Ceucă, school student, Iaşi
G173. We denote by T (a, b, c) the triangle whose side lengths are a, b, c. Prove
that the triangles T (b, 2c, 2mb ) and T (c, 2b, 2mc ) can be successively manufactured
from a single (the same) cardboard sheet without losses of material.
Petru Asaftei, Iaşi
b = 400 . Show that
G174. The isosceles triangle ABC is considered with m(A)
no points P ∈Int(ABC) exist such that m(P Õ 0
AB) = 30 , m(P Õ
BC) = 100 and
m(PÕCA) = 350 .
Gabriel Popa and Paul Georgescu, Iaşi
G175. Let ABCD be a quadrilateral inscribed in the circle of radius R. Prove
that AB · AD + CB · CD ≤ 4R2 .
Gheorghe Costovici, Iaşi

B. Highschool level
L166. Let ABCD be a rectangle, and C a circle passing through A which
intersects the lines (AB), (AC) and (AD) at the points M, N and P , respectively.
Show that AM · AB + AP · AD = AN · AC.
Gheorghe Iurea, Iaşi
L167. Let ABC be a triangle with AB > AC. The circle inscribed in the triangle
is tangent to the sides BC and AC at D and respectively E. We consider a point T
on the side [BC] and denote by J the center of the circle inscribed in △ABT . If
DE passes through the midpoint of the segment [CJ], prove that the triangle AT C
is isosceles.
Titu Zvonaru, Comăneşti

170
L168. Prove that in any triangle, with the usual notations, the following inequa-
lity holds :
a b c 11 p2 − 15 r2 − 60 R r 3
+ + ≥ ≥ .
b+c c+a a+b 6 p2 − 6 r2 − 24 R r 2
Marius Olteanu, Rm. Vâlcea
L169. What is the probability that the radii of the excircles to a randomly chosen
triangle be the sides of a new triangle?
Petru Minuţ, Iaşi
L170. Let n ∈ N, n ≥ 2 and a1 , a2 , . . . , an ∈ R+ with a1 + a2 + · · · + an = S.
We consider k ∈ N, 1 ≤ k ≤ n − 1 and α1 , α2 ∈ R with α1 + α2 = 1. Prove the
inequality
P α1 α2
(ai1 + ai2 + · · · + aik ) (S − ai1 − · · · − aik ) ≤
1≤ i1 < i2 <···< ik ≤ n
α
k α1 (n − k) 2 n
≤ k S.
n
(Connected with 6117 of R.M.T. 1/1987 ).
Paul Georgescu and Gabriel Popa, Iaşi

L171. For any x, yÈ ∈ R+ , prove that the following inequality holds :
√ √ €p p Š
x + 3 2(x + y) + y ≤ 2 3x + y + x + 3y .
Marian Tetiva, Bârlad
L172. Let P ∈ Q [X] with degP = n ≥ 1. If P admits a complex root a with
its multiplicity m such that n < 2 m, prove that a ∈ Q.
Adrian Reisner, Paris
L173. a) Do exist functions f : (a, b) → R with the property that |f (x) − f (y)| ≥
c for ∀x, y ∈ (a, b), where c is a positive constant?
b) Do exist functions f : (a, b) ∩ Q → R with the property that |f (x) − f (y)| ≥ c
for ∀x, y ∈ (a, b) ∩ Q, where c is a positive constant ?
Geanina Hăvârneanu, Iaşi
L174. Let a1 , a2 , . . . , an , b1 , b2 , . . . , bn be positive real numbers and let a =
!2 !2
X
n
√ X
n
p
ai , b= bi . Show that x0 > 0 exists such that
i=n " i=n #
X
n
p ”√ —
ai x + bi − ax + b ∈ {0, 1} , for ∀x > x0 .
i=n

Marian Tetiva, Bârlad


L175. Show that
[n]
X
2

Cn2k Ωk = 2n Ωn
k=0
(2k − 1)!!
where Ωk = , k ∈ N∗ ; by convention Ω0 = 1.
(2k)!!
Gheorghe Costovici, Iaşi

171
Pagina rezolvitorilor
COVASNA
Şcoala cu clasele I-VIII ”Avram Iancu”. Clasa a V-a (prof. LÁZÁR Emese).
BÂRLĂ Oana: P(161,163), V(95,96), VI.96; PĂTRÂNJEL Andrada-Maria: P(161,
163), V(95,96), VI.96; TIMARU Carmen-Ioana: P(161,163), V(95,96), VI.96; UŢĂ
Ioana: P(161, 163), V(95,96), VI.96; VLĂDEA Diana: P(161,163), V(95,96), VI.96.
CRAIOVA
Liceul Teoretic ”Tudor Arghezi”. Clasa a V-a (prof. DRĂCEA Dorina). VÎRLAN
Leonard: P.161, V(95-97).
IAŞI
Şcoala nr. 3 ”Al. Vlahuţă”. Clasa I (inst. MAXIM Gabriela). CUCURUZ
Raluca: P(154,155, 157, 164-167); DASCĂLU Lorena: P(154,155,157,164-167); PO-
PESCU Alexandru: P(154,157,164-167); ROBU Carmen: P(154,157,164-167); ŞER-
BĂNOIU Alexandru: P(154,157,164-167); TORAC George: P(154,157,164-167);
NICA Daniel: P(154,155,157,164-167). Clasa a II-a (inst. CRĂCIUN Marilena).
CREANGĂ Adrian: P(154,155,157,164-167); FILIP Vlad: P(154,155,157,164-167);
POPESCU Claudia: P(154,155,157,164-167). Clasa a II-a (inv. MĂRIUŢĂ Valen-
tina). ENEA Codruţ Alexandru: P(154, 155, 157, 164-167); GHEORGHIU Beatrice-
Elena: P(154,155,157,164-167); HERGHELEGIU Mădălina: P(154,155,157,164-167);
HREAPCĂ Alin: P(154,155,157,164-167); HUHU Paula: P(154,155,157,164-167);
POPOVICIU Teodor-Andrei: P(154,155,157,164-167); ROMILĂ Andreea-Maria:
P(154,155,157,164-167). Clasa a VI-a (prof. MARIN Mirela). MARCU Ana:
V(105,106), VI(102,104,105); RUSU Mădălina-Andreea: V(105,106), VI(102,104,105);
TIBA Ştefana-Alexandra: V(105,106), VI(102,104,105).
Şcoala nr. 13 ”Alexandru cel Bun”. Clasa I (inst. COJOCARIU Ana). ACA-
TRINEI Andra: P(154,164-167); PERDUN Patricia-Maria: P(154,164-167); PRISE-
CARU Alexandru-Julian: P(154,164-167), SAMSON Constantin-Cătălin: P(154,164-
167); ŞTEFAN Tudor: P(154,164-167); ZAHARIA Ştefan-Eusebiu: P(164,164-167).
Şcoala nr. 14 ”Gh. Mârzescu”. Clasa a IV-a (inst. NUŢĂ Elena). BACIU Tu-
dor: P(164-167,170); CHIRILUŢĂ George-Ştefan: P(164-167,169-171); NEDELEU
Iulia: P(164-167,170); POSTUDOR Georgiana-Mădălina: P(164-167,170); ŢÂMBA-
LARIU Ioana-Vasilica: P(164-167,170).
Şcoala nr. 22 ”B.P. Haşedeu”. Clasa I (inv. BLĂJUŢ Cristina). SAVA Cosmina
Ioana: P(164-167,169). Clasa a IV-a (inv. ŞTEFAN Liviu). BLĂJUŢ Cristin-
Marian: P(167-168).
Şcoala nr. 26 ”George Coşbuc”. Clasa a II-a (inst. VÂRLAN Elena). AMARIEI
Romeo: P(154-157,164-167); GHEBAN Andreea: P(154-157,164-167); PĂVĂLUC
Ana-Maria: P(154-157,164-167); PICHIU Cosmin: P(154-157,164-167); TĂTARU
Alice: P(154,157,164-167); ŢIPLEA Iulian: P(154-157,164-167). Clasa a III-a
(inv. BUCATARIU Rica). BARHAN Ştefana-Adina: P(164-166,168,169,171,172);
CHIRIAC Alexandra: P(164-166,168,169,171,172) CUPEŢ Valeria: P(164-166,168,
169,171,172); FRUNZĂ Andrei-David: P(164-166,168,169,171,172); FRUNZĂ Diana-
Mihaela: P(164-169,171,172); IVANOV Alexandra: P(164-166,168,169,171,172);

172
MÎNDRU Liana: P(164-169,171,172). Clasa a IV-a (inst. RACU Maria). APA-
CHIŢEI Aura-Georgiana: P(164-171); BURA Emma-Andreea: P(164-170); CRĂ-
CIUN Ioana-Daniela: P(164-171); LEŞOVSCHI Alexandra-Ioana: P(164-169); LUPU
Roxana-Elena: P(164-169).
Şcoala SAM ”M. Kogălniceanu”, Ţigănaşi. Clasa a II-a (ı̂nv. GALIA Paraschiva).
CAZADOI Ioana-Cristina: P(154,155,157,164-167); DUCA Cristina Mihaela: P(154-
155,157,164-167); SANDU Rebeca: P(154-155,157,164-167).
SFÂNTU GHEORGHE (Tulcea)
Şcoala generakă cu clasele I-VIII. Clasa a II-a (ı̂nv. GAVRILĂ Elena). HALCHIN
Ioana: P(154-158); SIDORENCU Adrian: P(154-158). Clasa a IV-a (ı̂nv. GAVRILĂ
Elena). BĂLAN Silviu: P(154-161); CLADIADE Bogdan-Robert: P(154-161); CUCU
Delia: P(154-161); EFIMOV Cosmin Alexandru: P(154-161). Clasa a VI-a (prof.
SĂILEANU Sorin). SIDORENCU Andrei: V(95,96), VI.96, VII(96,98), VIII.96.

Elevi rezolvitori premiaţi


Şcoala nr. 26 ”G. Coşbuc”, Iaşi
1. LEŞOVSCHI Alexandra-Ioana (cl. a IV-a): 2/2008(6pb), 1/2009(6pb), 2/2009
(6pb).
2. LUPU Roxana-Elena (cl. a IV-a): 2/2008(6pb), 1/2009(6pb), 2/2009(6pb).

Vizitaţi noua pagina web a revistei:

http://www.recreatiimatematice.ro

173
IMPORTANT
• În scopul unei legături rapide cu redacţia revistei, pot fi utilizate următoarele
adrese e-mail: t birsan@yahoo.com şi profgpopa@yahoo.co.uk . Pe
această cale colaboratorii pot purta cu redacţia un dialog privitor la ma-
terialele trimise acesteia, procurarea numerelor revistei etc. Sugerăm cola-
boratorilor care trimit probleme originale pentru publicare să le numeroteze
şi să-şi reţină o copie xerox a lor pentru a putea purta cu uşurinţă o discuţie
prin e-mail asupra acceptării/neacceptării acestora de către redacţia revistei.

• La problemele de tip L se primesc soluţii de la orice iubitor de matematici


elementare (indiferent de preocupare profesională sau vârstă). Fiecare dintre
soluţiile acestor probleme - ce sunt publicate ı̂n revistă după un an - va fi
urmată de numele tuturor celor care au rezolvat-o.

• Adresăm cu insistenţă rugămintea ca materialele trimise revistei


să nu fie (să nu fi fost) trimise şi altor publicaţii.

• Rugăm ca materialele tehnoredactate să fie trimise pe adresa redacţiei


ı̂nsoţite de fişierele lor (de preferinţă ı̂n LATEX).

• Pentru a facilita comunicarea redacţiei cu colaboratorii ei, autorii materi-


alelor sunt rugaţi să indice adresa e-mail.

174
Revista semestrială RECREAŢII MATEMATICE este editată de
ASOCIAŢIA “RECREAŢII MATEMATICE”. Apare la datele de 1 martie şi
1 septembrie şi se adresează elevilor, profesorilor, studenţilor şi tuturor celor
pasionaţi de matematica elementară.
În atenţia tuturor colaboratorilor
Materialele trimise redacţiei spre publicare (note şi articole, chestiuni de
metodică, probleme propuse etc.) trebuie prezentate îngrijit, clar şi concis; ele
trebuie să prezinte interes pentru un cerc cât mai larg de cititori. Se recomandă ca
textele să nu depăşească patru pagini. Evident, ele trebuie să fie originale şi să
nu fi apărut sau să fi fost trimise spre publicare altor reviste. Rugăm ca mate-
rialele tehnoredactate să fie însoţite de fişierele lor.
Problemele destinate rubricilor: Probleme propuse şi Probleme pentru
pregătirea concursurilor vor fi redactate pe foi separate cu enunţ şi demonstra-
ţie/rezolvare (câte una pe fiecare foaie) şi vor fi însoţite de numele autorului, şcoa-
la şi localitatea unde lucrează/învaţă.
Redacţia va decide asupra oportunităţii publicării materialelor primite.
În atenţia elevilor
Numele elevilor ce vor trimite redacţiei soluţii corecte la problemele din
rubricile de Probleme propuse şi Probleme pentru pregatirea concursurilor
vor fi menţionate în Pagina rezolvitorilor. Se va ţine seama de regulile:
1. Pot trimite soluţii la minimum cinci probleme propuse în numărul
prezent şi cel anterior al revistei; pe o foaie va fi redactată soluţia unei singure
probleme.
2. Elevii din clasele VI-XII au dreptul să trimită soluţii la problemele
propuse pentru clasa lor, pentru orice clasă mai mare, din două clase mai mici şi
imediat anterioare. Elevii din clasa a V-a pot trimite soluţii la problemele propuse
pentru clasele a IV-a, a V-a şi orice clasă mai mare, iar elevii claselor I-IV pot
trimite soluţii la problemele propuse pentru oricare din clasele primare şi orice cla-
să mai mare. Orice elev poate trimite soluţii la problemele de concurs (tip G şi L).
3. Vor fi menţionate următoarele date personale: numele şi prenumele,
clasa, şcoala şi localitatea, precum şi de numele profesorului cu care învaţă.
4. Plicul cu probleme rezolvate se va trimite prin poştă (sau va fi adus
direct) la adresa Redacţiei:

Prof. dr. Temistocle Bîrsan


Str. Aurora, nr. 3, sc. D, ap. 6,
700 474, Iaşi
Jud. IAŞI
E-mail: t_birsan@yahoo.com
CUPRINS
Către cititori dupǎ zece ani de apariţie a revistei .......................................................................... 85
NECULAI GHEORGHIU (1930-2009) ............................................................................................ 87

ARTICOLE ŞI NOTE
T. BÎRSAN, G. DOSPINESCU – Conjectura Beal pentru polinoame ..................................... 89
C.-L. BEJAN – Teorema lui Brouwer - un caz particular elementar........................................ 94
D. MĂRGHIDANU – Exponentul de triangularitate al unui triunghi...................................... 96
D. POPESCU – Asupra unui şir de integrale Riemann ............................................................. 98
A. VERNESCU – Dualitatea unor sume combinatoriale ........................................................ 101

NOTA ELEVULUI
D. M. MOCANU – Asupra problemei C.O: 5004 din G.M.................................................... 105

CORESPONDENŢE
A. REISNER – Matrices à coefficients dans un corps fini ..................................................... 107
CHESTIUNI METODICE
M. MIHEŢ – O metodǎ de rezolvare a problemelor ..............................................................111
S. BOGA – Metoda identificǎrii.............................................................................................115

CUM CONCEPEM ... CUM REZOLVĂM


D. MIHALACHE, M. TETIVA– Bisectoarele exterioare nu sunt ca bisectoarele interioare..117

MATEMATICA ÎN CLASELE PRIMARE


D.M. BĂTINEŢU-GIURGIU – Metoda falsei ipoteze - variante de utilizare.........................121

ŞCOLI ŞI DASCĂLI
Liceul Teoretic "Garabet Ibrăileanu"........................................................................................123
LIDIA COHAL (1930-2009) ...................................................................................................126

CONCURSURI ŞI EXAMENE
Concursul de matematică "Al. Myller", ed. a VII-a, 2009 .................................................. 127
Concursul de matematică "Florica T. Câmpan", 2009 ......................................................... 130

PROBLEME ŞI SOLUŢII
Soluţiile problemelor propuse în nr. 2/2008 .......................................................................... 135
Soluţiile problemelor pentru pregătirea concursurilor din nr. 2/2008 ................................ 152
Probleme propuse ...................................................................................................................... 161
Probleme pentru pregătirea concursurilor .............................................................................. 167
Training problems for mathematical contests ....................................................................... 169
Pagina rezolvitorilor ............................................................................................................... 172

ISSN 1582 – 1765 7 lei

S-ar putea să vă placă și